You are on page 1of 154

Join Online Coaching for IBPS, SBI Bank Exams

http://www.bankexamportal.com/elearning
1 EXAM PAPER IBPS Clerk Common Written Exam (Held on: 20-12-2015)

SOLVED PAPER
IBPS Clerk Common Written Exam

REASONING way based on their positions in the given


arrangement and so form a group. Which is
Directions (1-5): Study the following arrangement the one that does not belong to the group?
carefully and answer the given questions. (1) TW3 (2) @%7
(3) G9D (4) P3U
1. If all the symbols are dropped from the
(5) $9Q
arrangement, then which will be the eleventh
element from the right end of the given Directions (6-10): The questions is based on the five
arrangement? three-digit numbers given below.
(1) 9 (2) G 476 538 289 814 753
(3) D (4) 5 6. Which of the following is the second digit of
(5) P the three digit number obtained by
2. How many such symbols are there in the subtracting the lowest number from the
given arrangement each of which is highest number?
immediately followed by a letter and also (1) 2 (2) 3
immediately preceded by a number? (3) 4 (4) 6
(1) More than three (2) Two (5) 7
(3) Three (4) None 7. If 1 is added to the first digit in each number
(5) One and then the position of the first and the
3. Which of the following is sixth to the left of third digits are interchanged. Which of the
the fourteenth from the left end of the given following will be the third digit of the second
arrangement? highest number thus formed?
(1) b (2) * (1) 6 (2) 4
(3) U (4) Z (3) 2 (4) 8
(5) 4 (5) 7
4. What should come in place of the question 8. In which of these digits, the sum of all the
mark (?) in the following series based on the three digits is an even number?
given arrangement? (1) 753 (2) 538
UXT 3ZU 4PQ G5J ? (3) 269 (4) 476
(1) W 2 b (2) 6%8 (5) 814
(3) 6 % + (4) QXZ 9. Which of the following is the sum of the
(5) & 8 + second and third digits of the second lowest
5. Four of the following five are alike in a certain number? .

Buy Printed Study Material for IBPS, SBI Bank Exams


http://www.bankexamportal.com/study-kit
Join Online Coaching for IBPS, SBI Bank Exams
http://www.bankexamportal.com/elearning
EXAM PAPER IBPS Clerk Common Written Exam (Held on: 20-12-2015) 2
(1) 12 (2) 15 (1) Either conclusion I or II is true
(3) 91 (4) 11 (2) Only conclusion I is true
(5) 13 (3) Neither conclusion I nor II is true
10. If 1 is subtracted from third digit in the each (4) Only conclusion II is true
number and the position of the first and third (5) Both conclusions I and II are true
digits are interchanged. Which of the 14. Statements W # D, Z B, B $ H
following will be the first digit of the third Conclusions
highest number thus formed? I. H # Z II. B % W
(1) 3 (2) 5 (1) Either conclusion I or II is true
(3) 7 (4) 8 (2) Only conclusion I is true
(5) 9 (3) Neither conclusion I nor II is true
Directions (11-15): In these questions, the symbols (4) Only conclusion II is true
@, %, , $ and # are used with the following meaning (5) Both conclusions I and II are true
as illustrated below. 15. Statements F $ N, N @ D, D % B
Conclusions
P @ Q means P is neither smaller than nor
I. F @ D II. B @ N
equal to Q.
(1) Either conclusion I or II is true
P % Q means P is neither greater than nor (2) Only conclusion I is true
equal to Q. (3) Neither conclusion I nor II is true
P Q means P is not greater than Q. (4) Only conclusion II is true
P $ Q means P is not smaller than Q. (5) Both conclusions I and II are true
P # Q means P is neither smaller nor greater Directions (16-20): In these questions, two
than Q. statements followed by two conclusions numbered I and
II have been given. Decide which of the givenconclusions
Assuming the given statements to be true,
logically follows the given statements disregarding
find which conclusion is definitely true.
commonly known facts.
11. Statements H@K, K%M, MP
16. Statements All buses are cars.
Conclusions All scooters are buses.
I. H @ D II. K % D Conclusions
(1) Either conclusion I or II is true I. No scooter is a bus.
(2) Only conclusion I is true II. All cars are buses.
(3) Neither conclusion I nor II is true (1) Either conclusion l or II is true
(4) Only conclusion II is true (2) Only conclusion I is true
(5) Both conclusions I and II are true (3) Neither conclusion I nor II is true
12. Statements R % H, H T, T @ K (4) Only conclusion II is true
Conclusions (5) Both conclusions I and II are true
I. T R - II. K % H 17. Statements No auditorium is hall.
(1) Either conclusion I or II is true All theatres are halls.
(2) Only conclusion I is true Conclusions
(3) Neither conclusion I nor II is true I. No auditorium is a theatre.
(4) Only conclusion II is true II. All halls are theatres.
(5) Both conclusions I and II are true (1) Either conclusion I or II is true
13. Statements R D, D $ M, M # J (2) Only conclusion I is true
Conclusions (3) Neither conclusion I nor II is true
I. J # D II. J % D
Buy Printed Study Material for IBPS, SBI Bank Exams
http://www.bankexamportal.com/study-kit
Join Online Coaching for IBPS, SBI Bank Exams
http://www.bankexamportal.com/elearning
3 EXAM PAPER IBPS Clerk Common Written Exam (Held on: 20-12-2015)
(4) Only conclusion II is true 21. Four of the following five are alike in a certain
(5) Both conclusions I and II are true way and so form a group. Which is the one
18. Statements Some drugs are medicines. that does not belong to that group?
No medicine is a treatment. (1) L (2) R (3) 8
Conclusions (4) O (5) P
I. All treatments being drugs is a 22. Who sits third to the left of Q?
possibility. (1) M (2) S
II. All drugs can never be treatments. (3) R (4) N
(1) Either conclusion I or II is true (5) P
(2) Only conclusion I is true 23. How many people sit between M and S when
(3) Neither conclusion I nor II is true counted from the right hand side of S?
(4) Only conclusion II is true (1) One (2) Three
(5) Both conclusions I and II are true (3) None (4) Two
19. Statements Some cameras are photos. (5) Four
All cameras are snaps. 24. Which of the following is true regarding P?
Conclusions (1) P sits exactly between M and N
I. All snaps are photos. (2) O sits second to right of P
II. Some snaps are photos. (3) None of the given options is true
(1) Either conclusion I or II is true (4) L sits immediate right of P
(2) Only conclusion I is true (5) Q is an immediate neighbour of P
(3) Neither conclusion I nor II is true
(4) Only conclusion II is true 25. What is the position of P with respect to S?
(5) Both conclusions I and II are true (1) Fourth to the left
(2) Second to the right
20. Statements Some computers are tablets. (3) Second to tlie left
Some laptops are computers. (4) Third to the right
Conclusions (5) Third to the left
I. No laptop is a tablet.
II. All tablets being computers is a Directions (26-30): Study the informationand answer
possibility. the given questions.
(1) Either conclusion I or II is true In a certain code language
(2) Only conclusion I is true work never goes waste is written as rb mk
(3) Neither conclusion I nor II is true ni tj.
(4) Only conclusion II is true
(5) Both conclusions I and II are true never waste your time is written as ni ap sy
rb.
Directions (21-25): Study the following information
focus on your work is written as mk ap cn
carefully and answer the given questions. Eight friends
or.
L, M, N, O, P, Q. Rand S are sitting around a square table
in such a way that four of them sit at four corners of the focus goes with time is written as sy tj en
square while four sit in the middle of each of the four ke.
sides. The ones who sit at the four sides face outside while (All the codes are two letter codes only)
those who sit in the middle of the sides face the centre. 26. In the given code language, what does the
Only one person sits between Land Q. L sits at middle of code tj stand for?
oneof the series. O sits third to the right of Q. Both Rand (1) never (2) goes
S are immediate neighbours of L. M sits second to left of (3) on (4) work
N. N is neither an immediate neighbour of Q nor S. (5) waste
Buy Printed Study Material for IBPS, SBI Bank Exams
http://www.bankexamportal.com/study-kit
Join Online Coaching for IBPS, SBI Bank Exams
http://www.bankexamportal.com/elearning
EXAM PAPER IBPS Clerk Common Written Exam (Held on: 20-12-2015) 4
27. What is the code for focus in the given code 33. Four of the following five are alike in a certain
language? way based on the given seating arrangement
(1) ap (2) ni (3) sy and thus form a group. Which is the one that
(4) cn (5) mk does not belong to that group?
28. What is the code for time in the given code (1) H (2) I (3) W
language? (4) Y (5) X
(1) tj (2) sy (3) ni 34. Who amongst the following sits third to the
(4) cn (5) rb right of the person who faces X?
29. Which of the possibly means work on (1) G (2) F (3) J
projects? (4) I (5) H
(1) sy cn tj (2) gt cn or 35. Which of the following is true regarding V?
(3) mk gt or (4) mk cn gt (1) V faces I
(5) mk or sy (2) An immediate neighbour of V faces F
30. In the given code language, what does the (3) X is an immediate neighbour of V
code rb stand for? (4) W sits to immediate right of V
(1) Either never or waste (5) None of the given options is true
(2) goes (3) your Directions (36-40): These questions consist of a
(4) work (5) time question and two statements numbered I and II below
Directions (31-35): Study the following information it. You have to decide whether the data given in the
to answer the given questions. statements are sufficient to answer the questions. Read
the statements and choose the most appropriate option.
Ten people are sitting in two parallel rows
having five people each, in such a way that there is 36. In a straight line of eight people (all facing
an equal distance between adjacent person. In row North), what is the position of R from the left
l-V; W. X, Y and Z are seated (but not necessarily in end?
the same order) and all of them are facing North. In I. Y stands fourth from the right end of the
row 2-F, G, H, I and J are seated (but not necessarily line. Only two people stand between Y
in the same order) and all of them are facing South. and Z. R stands to the immediate right
Therefore, in the given seating arrangement, each of Z.
member seated ina row faces another member of the II. W stands fourth from the left end of the
other row. Y sits third to the left of W The one who line. R is an immediate nieighbour of W.
faces Y sits second to the right of F. Only one person (1) The data in statement I alone are
sits between F and 1. Hand J are immediate sufficient to answer the question while
neighbours of each other. J does not sit at any of the the data in statement II are not
extreme ends of the line. The one faces G sits to the sufficient to answer the question
immediate right of Z. X is not an immediate (2) The data in both statements I and II
neighbour of Z. together are necessary to answer the
question
31. Who amongst the following faces H?
(3) The data even in both statements I and
(1) Y (2) V (3) Z
II together are not sufficient to answer
(4) W (5) X
the question
32. Who amongst the following sits to the (4) The data either in statement I or II alone
immediate left of the person who sits exactly are sufficient to answer the question
in the middle of row 2? (5) The data in statement II alone are
(1) J (2) H (3) I sufficient to answer the question while
(4) G (5) F the data in statement I are not sufficient

Buy Printed Study Material for IBPS, SBI Bank Exams


http://www.bankexamportal.com/study-kit
Join Online Coaching for IBPS, SBI Bank Exams
http://www.bankexamportal.com/elearning
5 EXAM PAPER IBPS Clerk Common Written Exam (Held on: 20-12-2015)
to answer the question to answer the question
37. Among four friends W, X, Y and Z (each 39. How is P related to Q?
having different number of cookies), who has I. Q is the mother of T. M is the only sibling
the most number of cookies? of T. H is the daughter of M and P.
I. W has lesser number of cookies than Z. II. M is married to P. T is the brother of M.
Y does not have the most number of Q is the brother of T.
cookies. (1) The data in statement I alone are
II. W has more cookies than Y. X does not sufficient to answer the question while
have the most number of cookies. the data in statement II are not
(1) The data in statement I alone are sufficient to answer the question
sufficient to answer the question while (2) The data in both statements I and II
the data in statement II are not together are necessary to answer the
sufficient to answer the question question
(2) The data in both statements I and II (3) The data even in both statements I and
together are necessary to answer the II together are not sufficient to answer
question the question
(3) The data even in both statements. I and (4) The data either in statement I or II alone
II together are not sufficient to answer are sufficient to answer the question
the question (5) The data in statement II alone are
(4) The data either in statement I or II alone sufficient to answer the question while
are sufficient to answer the question the data in statement I are not sufficient
(5) The data in statement II alone are to answer the question
sufficient to answer the question while 40. How many students attended the cultural
the data in statement I are not sufficient fair of the college?
to answer the question I. The number of students attending the
38. In a code language my dear family is coded cultural fair was twice the number of
as 624. Which number stands for dear? female students.
I. In the same code language my small II. The number of female students
family is coded as 256. attending the cultural fair was 25 more
II. In the same code language, dear family than that in the previous year.
friend is coded as 647? (1) The data in statement I alone are
(1) The data in statement I alone. are sufficient to answer the, question while
sufficient to answer the question while the data in statement II are not
the data in statement II are not sufficient to answer the question
sufficient to answer the question (2) The data in both statements I and II
(2) The data in both statements I and II together are necessary to answer the
together are necessary to answer the question
question (3) The data even in both statements I and
(3) The data even in both statements I and II together are not sufficient to answer
II together are not sufficient to answer the question
the question (4) The data either in statement I or II alone
(4) The data either in statement I or II alone are sufficient to answer the question
are sufficient to answer the question (5) The data in statement II alone are
(5) The data in statement II alone are sufficient to answer the question while
sufficient to answer the question while the data in statement I are not sufficient
the data in statement I are not sufficient to answer the question
Buy Printed Study Material for IBPS, SBI Bank Exams
http://www.bankexamportal.com/study-kit
Join Online Coaching for IBPS, SBI Bank Exams
http://www.bankexamportal.com/elearning
EXAM PAPER IBPS Clerk Common Written Exam (Held on: 20-12-2015) 6

COMPUTER KNOWLEDGE 47. Which of the following cables can transmit


data at high speeds?
41. ...... refers to the unauthorized copying and (1) Flat cables
distribution of software. (2) Coaxial cable
(1) Hacking (3) Optic fibre cable
(2) Software piracy (4) Twisted pair cable
(3) Software literacy (5) UTP cable
(4) Cracking 48. The hardware device or software program
(5) Copyright that sends messages between networks is
42. Which of the following is an operating known as a
system? (1) bridge
(1) Umlx (2) Debugger (2) backbone
(3) Mozilla (4) Google Chrome (3) router
(5) Intel 8085 (4) gateway
43. A collection of programs that controls how (5) Other than those given as options
your computer system runs and processes 49. If you are performing Windows 98 operating
information is called system to Windows XP you are performing
(1) operating system a(n)
(2) computer (1) push up
(3) office (2) China
(4) compiler (3) patch
(5) interpreter (4) pull down
44. Which of the following statements is a false (5) update
conceiving file names? 50. . . . . . . . . . technology enables you to carry a
(1) Every file in the same folder must have powerful navigational aid in websites.
a unique name (1) Convergence (2) Locator
(2) The file name comes before the dot (.) (3) Sampling rate (4) Synchronization
(3) File extension is another name for the (5) Global Positioning System
type 51. Which of the following is not true about
(4) The file extension comes before the dot RAM?
(.) followed by the file name (1) RAM is the same as hard disk storage
(5) Files may share the same name or the (2) RAM is a temporary storage area
same extension, but not both at the (3) RAM is volatile
same time (4) RAM stands for Random Access Memory
45. Which is not a storage device? (5) Information stored in RAM is gone when
(1) Printer (2) CD you, turn the computer off
(3) Hard Disk (4) Floppy Disk 52. All of the following are examples of real
(5) DVD security and privacy risks EXCEPT
46. ........ tags, placed on animals body can be (1) spyware (2) spam
used to record and track in a database all of (3) hackers (4) identity theft
the animals movements. (5) viruses
(1) pas (2) RFID 53. Which of the following commands in Office
(3) PPS (4) GPS 2007, can be used to go to the first cell in the
(5) Other than those given as options current row?

Buy Printed Study Material for IBPS, SBI Bank Exams


http://www.bankexamportal.com/study-kit
Join Online Coaching for IBPS, SBI Bank Exams
http://www.bankexamportal.com/elearning
7 EXAM PAPER IBPS Clerk Common Written Exam (Held on: 20-12-2015)
(1) Tab 60. . . . . . . . .. can interpret voice data into words
(2) Shift + Tab that can be understood by the computer.
(3) Esc + Home (1) Speech input hardware
(4) Shift + Home (2) Speech recognition software
(5) Home (3) Word recognition software
54. Software such as Viruses, Worms and Trojan (4) Talking software
Horses that has a malicious content, is (5) Other than those given as options
known as 61. Oracle is a(n)
(1) malicious software (malware) (1) hardware
(2) adware (2) high level language
(3) shareware (3) operating system
(4) spyware (4) system software
(5) firewall (5) RDBMS
55. Chip is a common nickname for a(n) 62. Which of the following company developed
(1) transistor the microprocessor?
(2) resistor (1) Intel (2) IBM
(3) integrated circuit (3) Microsoft (4) Apple
(4) semi-conductor (5) Google
(5) Other than those given as options 63. What does ALU in computing denote?
56. What is extension of Microsoft Word (1) Application and Logic Unit
document? (2) Algorithm Logic Unit
(1) doc (2) ppt (3) Arithmetic Layered Unit
(3) dcc (4) bd (4) Arithmetic Legal Unit
(5) hml (5) Arithmetic Logic Unit
57. Which of the following enables to determine 64. Integrated Circuit (IC) chips used in
how often a user visited a website? computers are made with
(1) Hackers (1) copper
(2) Spammers (2) aluminium
(3) Phish (3) gold
(4) Identity thefts (4) silicon
(5) Cookies (5) silver
58. To move to the bottom of a document while 65. What does the computer abbreviation MB
working on MS Word, which command is used for?
used? (1) Mega Bit
(1) Home key (2) End key (2) Million Bytes
(3) Ctrl + Page Down(4) Insert key (3) Mega Bytes
(5) Ctrl key + End key (4) Million Bit
59. What is a person called who uses a computer (5) Micro Bytes
to cause harm to people or destroy critical 66. What is the full form of HTTP?
systems? (1) Hyper Text Transfer Protocol
(1) Cyber Terrorist (2) Hyper Text Transition Protocol
(2) Black-hat Hacker (3) Hyper Text Transfer Program
(3) Cyber Cracker (4) Hacktivist (4) Hyper Text Transition Program
(5) Other than those given as options (5) Hyper Text Trivial Protocol

Buy Printed Study Material for IBPS, SBI Bank Exams


http://www.bankexamportal.com/study-kit
Join Online Coaching for IBPS, SBI Bank Exams
http://www.bankexamportal.com/elearning
EXAM PAPER IBPS Clerk Common Written Exam (Held on: 20-12-2015) 8
67. Verification of a login name and password is Information Interchange
known as (4) American Standardisation Code for
(1) configuration Information Interchange
(2) accessibility (5) American Standardisation Code for
(3) authentication Information Interchanger
(4) logging in 74. Which of the following is not a valid formula
(5) Other than those given as options in Microsoft Excel?
68. Which of the following is software? (1) =A2+A1
(1) Keyboard (2) =A2+1
(2) Internet Explorer (3) = 1 + A2
(3) Scanner (4) =1A+2
(4) Mouse (5) =A1 +A2
(5) Printer 75. What does the acronym ISP stand for?
69. The term bit is short for (1) Internal Service Provider
(1) binary digit (2) International Service Provider
(2) binary number (3) Internet Service Provider
(3) binary language (4) Internet Service Providing
(4) small digit (5) Internet Service Provision
(5) one byte 76. What is the full form of WWW?
70. What does MICR stand for? (1) Work Wide Web
(1) Magnetic Ink Character Register (2) World Wide Web
(2) Magnetic Ink Code Reader (3) World With Web
(3) Magnetic Ink Code Register (4) World Wide Wet
(4) Magnetic Ink Character Recognition (5) World Weak Web
(5) Magnetic Ink Cases Reader 77. Which of the following is hardware?
71. RTGS stands for (1) Assembler
(1) Real Time Gross Settlement (2) Server
(2) Real Time General Settlement (3) Compiler (4) Interpreter
(3) Run Time Gross Settlement (5) Operating System
(4) Regular Time General Settlement 78. USB in data cables stands for
(5) Regular Time Gross Settlement (1) Unicode Smart Bus
72. A device that converts from decimal to binary (2) Universal Structural Bus
number is known as (3) Unicode Serial Bus
(1) instructor (4) Universal Smart Bus
(2) decoder (5) Universal Serial Bus
(3) AND gate 79. Which of the following is not a binary
(4) converter number?
(5) decoder (1) 11101 (2) 110
73. What does the acronym ASCI stands for? (3) 233 (4) 1001
(1) American Standard Code for (5) 100110
Information Interchanger 80. Ais approximately one billion bytes.
(2) American Standard Code for (1) megabyte (2) kilobyte
Information Interchange (3) terabyte (4) gigabyte
(3) American Standardised Code for (5) bit

Buy Printed Study Material for IBPS, SBI Bank Exams


http://www.bankexamportal.com/study-kit
Join Online Coaching for IBPS, SBI Bank Exams
http://www.bankexamportal.com/elearning
9 EXAM PAPER IBPS Clerk Common Written Exam (Held on: 20-12-2015)

ENGLISH LANGAUGE inflation and improve confidence in the


economy.
Directions (81-85): Choose the best word that fits in (1) achieve (2) get
the meaning of both the sentences to make them (3) end (4) finish
grammatically correct and coherent. (5) undertake
81. I. Governments in these countries should 85. 1. According to some experts, subsidising
create education systems since . good crop insurance to a large extent may be bad
schools the bulk of people entering the world for the environment since farmers may take
force will not have the skills they need. .......... such as farming on food plains or sleep
hills.
II. We cannot process colour, details of rapid
changes in our surroundings .... the cone II. In the 1990s, Germany was known as the
shaped cells packed around the centre of the sick man of Europe and had high
retina. unemployment but its success today is on
(1) lack (2) missing account of the huge ........ it took in reforming
(3) eficient (4) without the labour market
(5) absence (1) danger (2) chance
(3) possibility (4) risks
82. I. Japan is investing in strengthening bonds
(5) threats
with ASEAN countries and Japanese
countries recently won a $ 370 million Directions (86-90): Arrange the following five
contract to start. a new underground sentences A, B, C, D and E in the proper sequence to form
railway system in Jakarta. a meaningful paragraph and then answer the given
II. The cost ofspace vehicles is high no doubt, questions.
but the costs of complying with legal and A. If you are transitioning from products to
regulatory paperwork and rules is huge too. services or vice-versa, you have to know and
(1) construction (2) manufacture understand these differences to effectively
(3) launch (4) assembly promote and sell.
(5) building B. For example, a product is tangible, which
83. I. A troubled student and his PhD guide ........ means the customer can touch and see the
a way for websites to reduce the incidence of product before deciding to make a purchase
spam which troubled internet users in the and a service is intangible.
2000s. C. Understanding the different changes in
II. One of the biggest drawback of concrete is product and service marketing can help you
that it often develop cracks and scientists establish the right approach for this
have now . self-healing concrete. transition.
(1) create (2) devised D. Unless you understand the basic difference of
(3) develop (4) thought tangibility, it will be a challenge to promote
(5) invent and sell your product.
84. I. Politicians have promised to ....... the goal E. Companies that are marketing a product face
of eradicating extreme poverty many times different challenges compared to those that
but have failed because they cannot agree are promoting a service.
about what exactly counts as poverty and
how exactly to measure it. 86. Which of the following should be the FIFTH
(LAST) sentence after the rearrangement?
II. Reserve Bank hoped that a cut in its (1) A (2) E (3) D (4) C (5) B
benchmark rate will . a reduction in
Buy Printed Study Material for IBPS, SBI Bank Exams
http://www.bankexamportal.com/study-kit
Join Online Coaching for IBPS, SBI Bank Exams
http://www.bankexamportal.com/elearning
EXAM PAPER IBPS Clerk Common Written Exam (Held on: 20-12-2015) 10
87. Which of the following should be the FIRST 95. The trend of wearing boots in something that
sentence after the rearrangement? has most sure come around this season.
(1) A (2) B (3) C (4) D (5) E (1) most surely
88. Which of the following should be the (2) very certainly
SECOND sentence after the rearrangement? (3) very sure
(1) A (2) B (3) C (4) D (5) E (4) most certainly
(5) No correction required
89. Which of the following should be the THIRD
sentence after the rearrangement? Directions (96-105): Read the following passage
(1) A (2) B (3) C (4) D (5) E carefully and answer the questions given below it.
90. Which of the following should be the Do you ever feel there is a greater being inside
FOURTH sentence after the rearrangement? of you bursting to get out? It is the voice that
(1) A (2) B (3) C (4) D (5) E encourages you to really make something of your life.
When you act congruently with that voice, its like
Directions (91-95): Which of the phrases givenbelow
you are a whole new person. You are bold and
the sentence shouldreplacethe word/phrase given in bold courageous. You are strong. You are unstoppable.
in sentence to make it grammatically correct? If the But, then realitysets in and soonthose moments are
sentence is correct as it is given and no correction is history. It is not hard to put yourself temporarily
required, mark No correction required as the answer. into an emotionally motivated state Just listen to
91. In order to look taller, we should use pencil that motivational song for that matter. However,
heels instead block heels. this motivation does not stay forever. Your great
(1) instead of (2) despite ideas seem impractical. How many times have you
(3) in spite of (4) neither been temporarily inspired with an idea like, I want
(5) No correction required to start my own business. And then a week later its
92. This move is expected for mitigate the effects forgotten. You come up with inspiring ideas when
of a depleting water by making use of surface you are motivated. But you fail to maintain that
or canal water. motivation through the action phase.
(1) expects of The problem we ask ourselves is, why does this
(2) is expectation of happen? You can listen to hundreds of motivational
(3) is expected to (4) is expectant to speakers and experience an emotional yo-yo effect,
(5) No correction required but it does not last. The problem is that as we are
93. Do you know that dressing properly for an intellectually guided, we try to find logic in
interview can earns you those brownie extra emotional motivation and as we fail to find logic it
points which will help you get that dream eventually phases out. 1 used to get frustrated when
job? my emotional motivation fizzled out after a while.
(1) can earn (2) will earning Eventually, I realised that being guided by intellect,
(3) earned (4) well earns was not such a bad thing after all. 1 just had to learn
(5) No correction required to use my mind as an effective motivational tool. 1
figured that if1 was not feeling motivated to go after
94. The craze for private hands has caught the a particular goal, may be there was a logical reason
fancy of the citys residents. for it. I noted that when 1 had strong intellectual
(1) catch the fancily reasons for doing something, 1 usually did not have
(2) caught fancily trouble taking action. But when my mind thinks a
(3) catching fancy goal is wrong on some level. I usually feel blocked. I
(4) catch fancy eventually realised that this was my minds way of
(5) No correction required telling me the goal was a mistake to begin with.

Buy Printed Study Material for IBPS, SBI Bank Exams


http://www.bankexamportal.com/study-kit
Join Online Coaching for IBPS, SBI Bank Exams
http://www.bankexamportal.com/elearning
11 EXAM PAPER IBPS Clerk Common Written Exam (Held on: 20-12-2015)
Sometimes a goal seems to make sense on one level 97. Which of the following is/are true in the
but when you look further upstream, it becomes context of the passage?
clear that the goal is ill advised. Suppose you work A. Many a time, although we are motivated
in sales and you get a goal to increase your income at the start, we fail to keep up the
by 20% by becoming a more effective salesperson. motivation while working towards the
That seems like a reasonable and intelligent goal. But goal.
may be you are surprised to find yourself B. We can learn to use our mind as a
encountering all sorts of internal blocks when you motivational tool.
try to pursue it. You should feel motivated, but you C. Being guided by intellect is very bad.
just dont. The problem may be that ona deeper level (1) A and B (2) Band C
your mind knows you dont want to be working in (3) Only A (4) A and C
sales at all. You really want to be a musician. No (5) All of these
matter how hard you push yourself in sales career, 98. According to the passage, the key to
it will always be a motivational dead end. motivation is
Further when you set goals, that are too small A. setting challenging goals.
and too timid, you suffer a perpetual lack of B. not letting our fears pull us back.
motivation. You just need to summon the courage to C. changing our goals periodically.
acknowledge your true desires. Then you will have (1) A and B (2) Only C
to deal with the self-doubt and fear thats been (3) Only B (4) A and C
making you think too small. Ironically, the real key (5) All of these
to motivation is to set the goals that scare you. You 99. Which of the following is most nearly the
are letting fears, excuses and limiting beliefs hold same in meaning tothe word as used in the
you back. Your subconscious mind knows you are passage?
strong, so it wont provide any motivational fuel
until you step up, face your fears, and acknowledge Timid
your hearts desire. Once you finally decide to face (1) Sudden (2) Scared
your fears and drop the excuses, then you will find (3) Humble (4) Distant
your motivation turning on full blast. (5) Egoistic

96. What does the author want to convey when 100. Which of the following is/are not true in the
he says, When you look further upstream, it context of the passage?
becomes clear that the goal is ill advised? A. Although a goal may look intellectual, it
(1) When you analyse your goal closely you may not actually work for us.
realise that it is not what you thought it B. Our subconscious mind will not
would be motivate us unless we face our fears.
(2) When you work towards dealing with C. The only way to keep yourself motivated
your problems eventually they disappear throughout is by listening to many
(3) When you inspect the problem you motivational speakers.
realise that the solution is within our (1) Band C (2) Only C
reach (3) Only A (4) A and B
(4) When you devise a method of motivating (5) Only B
self you must keep analysing it 101. According to the passage, the author found it
periodically difficult to be motivated because
(5) When you face a problem you realise A. he was not intellectually motivated
your incapability of making good B. he wanted to take the easy way out.
decisions. C. he did not push himself hard enough.
(1) A and B (2) Only C

Buy Printed Study Material for IBPS, SBI Bank Exams


http://www.bankexamportal.com/study-kit
Join Online Coaching for IBPS, SBI Bank Exams
http://www.bankexamportal.com/elearning
EXAM PAPER IBPS Clerk Common Written Exam (Held on: 20-12-2015) 12
(3) Only A (2) choosing the colour, the actress creating
(4) A and C a fluffed
(5) Only B (3) with the mini cape that she wore
102. Which of the following is most nearly the (4) to match her dress
same in meaning to the word as used in the (5) No error
passage? 107. Winter is / the best / season to / explored the
Congruently outdoors.
(1) Periodically (2) In deluson (1) Winter is (2) the best
(3) In addition (4) Progressively (3) season to
(5) In agreement (4) explored the outdoors
(5) No error
103. What does the author mean when he says, It
eventually phases out? 108. The technology sepses / how a hand shakes
(1) We need continuous practice in phases / and makes instant adjustments / to stay
to retain it balanced.
(2) It reduces because of lack of practice (1) The technology senses
(3) We become self-motivated with time (2) how a hand shakes
(4) It goes after a period of time passes (3) and makes instant adjustments
(5) It improves after a certain period passes (4) to stay balanced
(5) No error
104. Which of the following can be an appropriate
title for the passage? 109. With the water project moving / a step closer
(1) Living Life With Success to feasibility, / city residents can soon /
(2) Why Dont We Feel Motivated? expect for sufficient water supply.
(3) Our Only Aim-Success (1) With the water project moving
(4) How to Achieve Intellectual Success (2) a step closer to feasibility,
(5) Feel Motivated-Feel Superior (3) city residents can soon
(4) expect for sufficient water supply
105. According to the passage, what does the (5) No error
author say about emotional motivation?
A. It tends to be temporary. 110. We are in the final stages / of resolve the
B. It promises our negativity. issue / which has been pending / for over two
C. It has the potential to inspire us. years.
(1) A and B (2) Only B (1) We are in the final stages
(3) Only A (4) A and C (2) of resolve the issue
(5) All of these (3) which has been pending
(4) for over two years
Directions (106-110): Read each of the following (5) No error
sentences to find out whether there is any grammatical
mistakes/error in it. The error, if any will be in one part Directions (111-120) In the given passage; there are
of the sentence. Mark the part with the error as your blanks, each of which hasbeennumbered These numbers
answer. If there is no error, mark No error as your are printed below the passage and against each, five
answer. (Ignore the errors of punctuation, if any) words are suggested, one of which fits the blank
appropriately. Find out the appropriate word in each
106. More than scoring points for / choosing the case.
colour, the actress creating a fluffed / with
the mini cape that she wore / to match her The emergence of a cloud based banking will
dress. affect banks big and small. Banks are expected to
(1) More than scoring points for spend almost $ 180 billion on IT this year. At

Buy Printed Study Material for IBPS, SBI Bank Exams


http://www.bankexamportal.com/study-kit
Join Online Coaching for IBPS, SBI Bank Exams
http://www.bankexamportal.com/elearning
13 EXAM PAPER IBPS Clerk Common Written Exam (Held on: 20-12-2015)
present, cloud based services make up a (111) GENERAL KNOWLEDGE
fraction of this amount but some estimates (112) by
financial services firms on the cloud will (113) $ 26 121. India has recently signed a deal to build a $ 1
billion in 2015. This increase should (114) barriers billion 900 MW hydro-power project on Arun
to entry for newcomers which can (115) modern IT river with
infrastructure at monthly fees of less than $ 10000 (1) Bhutan (2) Bangladesh
(116) having to invest tens of millions of dollars (3) Myanmar (4) Nepal
upfront (117) build their own secure data entries (5) China
and it should (118) enable big banks to become much 122. Indian player Anirban Lahiri is associated
more cost of (119). Small firms without traditional with the game of
computer systems to maintain are the fastest (1) rifle shooting (2) golf
movers. (120) can type documents, run (3) billiards (4) wrestling
spreadsheets and read e-mails in the cloud. Keeping (5) chess
track of clients, payments and loans can be done on
a cloud computing platform using a specially 123. The abbreviation ATS stand for
banking software. (1) Anti Terrorism Sensor
111. (1) largely (2) tiny (2) Anti Terrorist Set-up
(3) bit (4) part (3) Anti Terrorist System
(5) less (4) Anti Terrorism Squad
112. (1) paying (2) offering (5) Anti Terrorism Scheme
(3) buying (4) purchase 124. China has recently announced that it has
(5) spending completed a major hydro-power darn in Tibet
113. (1) sum (2) come over the river Yartung Zangbo known in
(3) account (4) costs India as
(5) total (1) Indus river
114. (1) fall (2) dropped (2) Brahmaputra rive
(3) lower (4) sank (3) Zanskar river
(5) maintain (4) Ranganadi rive
115. (1) rent (2) sold (5) Doyanq river
(3) hired (4) leased 125. According to which of the following acts, a
(5) used fixed deposit in a bank should not be paid in
116. (1) other (2) rather cash if it is Rs. 20000 and above?
(3) more (4) further (1) Wealth Tax Act
(5) compared (2) RBI Act
117. (1) that (2) for (3) Income Tax Act
(3) to (4) try (4) Banking Regulation Act
(5) on (5) Negotiation Instruments Act
118. (1) both (2) include
126. The Commonwealth Association for Public
(3) always (4) beside
Administration and Managements (CAP
(5) also
AM) International Innovation Awards for
119. (1) effect (2) efficient
2014 has recently been conferred upon the
(3) price (4) ceiling
Indian State of
(5) subsidy
120. (1) Employees (2) Who (1) Uttar Pradesh (2) Andhra Pradesh
(3) How (4) Worker (3) Gujarat (4) Karnataka
(5) Sub-ordinates (5) Maharashtra

Buy Printed Study Material for IBPS, SBI Bank Exams


http://www.bankexamportal.com/study-kit
Join Online Coaching for IBPS, SBI Bank Exams
http://www.bankexamportal.com/elearning
EXAM PAPER IBPS Clerk Common Written Exam (Held on: 20-12-2015) 14
127. In 2016, the Olympic Games shall be held in (4) ATMs, which are set-up by Reserve Bank
(1) Beijing (China) of India
(2) Tokyo (Japan) (5) ATMs, which are installed by group of
(3) Rio de Janeiro (Brazil) bankers
(4) Seoul (South Korea) 133. As per the recent press reports, RBI has
(5) Bangkok (Thailand) recently conveyed the in-principle approval
128. Banks borrow money from the RBI at which to open new bank. Which of the following
of the following rates? entity has received the permission?
(1) CRR (1) Tata Group
(2) Base Rate (2) L1C Housing Finance
(3) Repo Rate (3) Anil Ambani Group
(4) SLR (4) Videocon Group
(5) Reverse Repo Rate (5) IDFC
129. Which of the following is not an online travel 134. Which of the following is present Chief
safe? Economic Advisor of India?
(1) Goibibo.com (1) Pranjul Bhandari
(2) Thomascook.in (2) Dr. Prachi Mishra
(3) Zomato.com (3) Arvind Subramanian
(4) Arzoo.com (4) Dr. Kaushik Basu
(5) Expedia.co.in (5) Supriyo De
130. Under the PMJDY, the government is 135. By the year 2022, the Sardar Patel Urban
looking to open at least . . . . . . . .. basic bank Housing Mission aims to build as many as
accounts for unbanked tambles in the (1) 25 million houses
country. (2) 20 million houses
(1) 7.5 crore (3) 30 million houses
(2) 1.8 crore (4) 35 million houses
(3) 5.7 crore (5) 15 million houses
(4) 3 crore 136. The private sector banking space witnessed
(5) 4.5 crore its first consolidation move in four years.
131. In order to achieve financial inclusion goals, Which of the following bank merger took
RBI has permitted opening of USBs. What is place recently?
the full form of USB? (1) United Western Bank and IDBI Bank
(1) Urban Small Branch (2) Ganesh Bank of Kurundwad and Federal
(2) Unique Safety Branch Bank
(3) Ultra Small Branch (3) ING Vysya Bank and Kotak Mahindra
(4) Other than those given as options Bank
(5) United Smaller Branches (4) Bank of Rajasthan and ICICI Bank
132. As per cent press in India, White Label A TMs (5) State Bank of Indore and State Bank of
are installed. What is meant by a white label India
ATM? 137. Who bears the premium payable to the
(1) ATMs, which are set-up by non-banking insurance company under accidental
entities insurance cover under PMJDY scheme?
(2) ATMs, which are white in colour (1) Government of India
(3) ATMs, which are set-up by group of (2) 50% by account holder and rest by the
foreign banks operating in India bank

Buy Printed Study Material for IBPS, SBI Bank Exams


http://www.bankexamportal.com/study-kit
Join Online Coaching for IBPS, SBI Bank Exams
http://www.bankexamportal.com/elearning
15 EXAM PAPER IBPS Clerk Common Written Exam (Held on: 20-12-2015)
(3) NPCI 143. The Parliamentary Constituency in Uttar
(4) Account holder Pradesh represented in the 16th Lok Sabha
(5) RBI by Rajnath Singh, the Union Minister for
138. Buy Now Pay Now refers to which of the Home Affairs is
following? (1) Lucknow
(1) Affinity Card (2) Mirzapur
(2) Smart Card (3) Kanpur
(3) Business Card (4) Haidergarh
(4) Credit Card (5) Chandauli
(5) Debit Card 144. Balances in deposit accounts which are not
139. The government of India has decided to operated for at least years are to be
obser ve the birth anniversary of Sardar transferred to the RBI.
Vallabbhai Patel on October 31, as (1) 2
(1) Rashtriya Sankalp Diwas (National (2) 7
Resolve Day) (3) 12
(2) Rashtriya Shahidi Diwas (National (4) 5
Martyrs Day) (5) 10
(3) Rashtriya Sadbhavana Diwas (National 145. The 18th SAARC meet was held in
Solidarity Day) November, 2014 at
(4) Rashtriya Ekta Diwas (National Unity (1) Dhaka (Bangladesh)
Day) (2) Kathmandu (Nepal)
(5) Rashtriya Shiksha Diwas (National (3) Thirnphu (Bhutan)
Education Day) (4) Colombo (Sri Lanka)
140. The international day for Elimination of (5) New Delhi (India)
Violence against Women is observed across 146. Many times we read about SHGs in financial
the world on newspapers.
(1) December 15 (2) August 24 What is the full form of the term SHGs?
(3) June 24 (1) Self Help Groups
(4) November 25 (2) Other than those given as options
(5) September 24 (3) Small Help Groups
141. Time limit for exchanging of pre- 2005 Indian (4) Small Hope in Growths
currency notes is (5) Self Hope Groups
(1) April 1, 2015 147. Which of the following institutions provides
(2) December 31, 2014 secured depository services?
(3) January 1, 2015 (1) NSE
(4) June 1, 2015 (2) RBI
(5) September 1, 2015 (3) NSDL
142. Kangto also known as Kanggardo Rize is a (4) BSE
mountain in the Eastern Himalayas of India (5) Other than those given as options
and is highest peak in the State of 148. Which Indian woman sportsperson has
(1) Sikkim recently been appointed as the UN Goodwill
(2) Arunachal Pradesh Ambassador for the South Asian region?
(3) Jammu and Kashmir (1) Mary Kom
(4) Uttarakhand (2) Sania Mirza
(5) Himachal Pradesh (3) Sikha Tandon

Buy Printed Study Material for IBPS, SBI Bank Exams


http://www.bankexamportal.com/study-kit
Join Online Coaching for IBPS, SBI Bank Exams
http://www.bankexamportal.com/elearning
EXAM PAPER IBPS Clerk Common Written Exam (Held on: 20-12-2015) 16
(4) Jwala Gutta (1) Al Ahmadi
(5) Dipika Pallikal (2) Al Jahra
149. The United Nations Childrens Fund (a) Kuwait City
(UNICEF) (4) Al Salmiya
(1) Washington DC (USA) (5) Hawalli
(2) Vienna (Austria) 156. The Davis Cup is the premier international
(3) Paris (France) team event in
(4) Geneva (Switzerland) (1) Mens Tennis
(5) New York City (USA) (2) Mens Hockey
150. IRNSS is an independent regional navigation (3) Mens Soccer
satellite system being developed by (4) Womens Hockey.
(1) Japan (2) the USA (5) Womens Cricket
(3) Brazil 157. The Sans ad Adarsh Gram Yojana has been
(4) China launched on October 11, 2014, the birth
(5) India anniversary of
151. The first Commonwealth Science (1) Jai Prakash Narayan
Conference was recently held in (2) Shyam Prasad Mukherji
(1) Singapore (3) Lala Lajpat Rai
(2) Bangalore (India) (4) Deen Dayal Upadhyaya
(3) Glasgow (Scotland) (5) Balraj Madhok
(4) Kuala Lumpur (Malaysia) 158. The Minister of State (Independent Charge)
(5) Colombo (Sri Lanka) for Skill Development and
152. Typhoon Rammasun is Entrepreneurship in the revamped Union
(1) China Council of Ministers is
(2) Singapore (1) Mahesh Sharma
(3) India (2) Ravij Pratap Rudy
(4) Japan (3) JP Nadda
(5) Myanmar (4) Birender Singh
(5) Najma Heptulla
153. The currency of Sultanate of Oman is
(1) Omani Rand 159. The theme of the next Pravasi Bhartiya Divas
(2) Omani Rial to be held in the year 2015 is based on the
(3) Omani Dinar completion of the hundred years of Mahatma
(4) Omani Guilder Gandhis
(5) Omani Shilling (1) return to South Africa from India
(2) marriage with Kasturba Gandhi
154. Process of getting shares in an electronic (3) Quit India call to the British users
form of holding in a demat account is known (4) return to India leaving South Africa
as (5) founding the Natal Indian Congress
(1) Rematerialisation
(2) Other than those given as options 160. According to the 2011 census, the highest
(3) Dematerialisation literacy rate is recorded by
(4) Materialisation (1) Delhi
(5) Mutilsation (2) Tripura
(3) Mizoram
155. Which of the following is the Capital of the (4) Kerala
State of Kuwait? (5) Goa

Buy Printed Study Material for IBPS, SBI Bank Exams


http://www.bankexamportal.com/study-kit
Join Online Coaching for IBPS, SBI Bank Exams
http://www.bankexamportal.com/elearning
17 EXAM PAPER IBPS Clerk Common Written Exam (Held on: 20-12-2015)

NUMERICAL ABILITY 3
166. Two pipes A and B can fill tank in 3 h when
7
161. In an examination, a student scores 6 marks
for every correct answer and loses 4 marks opened simultaneously. If B alone can takes
for every wrong answer. If he attempted 80 2 h less than A alone takes to fill the tank
questions and obtained 310 marks, how compleletely. How much does A alone take to
many questions did he attempted correctly? fill the tank?
(1) 59 (2) 67 (1) 8 h (2) 12 h
(3) 63 (4) 65 (3) 4 h (4) 6 h
(5) 61 (5) 10 h
162. The diameter of a wheel is 49 m. How many 167. A man can now 10.2 km downstream in 18
revolutions will it make to cover a distance of min. If the speed of the stream in 3.5 km/h,
3200 m? how much time (in h) he would take to cover
(1) 17 (2) 27 121.5 km upstream?
(3) 24 (4) 22 1
(5) 18 (1) 4 (2) 3 (3) 4
2
163. The average run of a cricketer after 18
1 1
matches was 565. If he made 101 runs and (4) 5 (5) 3
123 runs in 19th and 20th match 2 2
respectively. What is his new average run 168. The respective ratio of two numer is 16 : 21.
after 20th match? If the first number is increased by 30% and
(1) 62.05 (2) 64.45 the second number is decreased by 20%,
(3) 60.45 (4) 61.25 what will be the respective ratio of the first
(5) 63.85 and the second number?
164. Two types of rice (type 1 and type 2) were (1) 32 : 21 (2) 26 : 21
mixed in the respective ratio of 1 : 3. The (3) 25 : 21 (4) 20 : 21
mixture was then sold at the rate of Rs. 75.60 (5) 22 : 21
per kg to gain a profit of 20%. If the price of 169. A bag of fruits was distributed among 4
type 1 rice is Rs. 75 per kg, what is the price studens P, Q, R and S. P took 3/8th of the
of type 2 rice per kg? fruits. Q took 1/5th of the remaining fruits
(1) Rs. 55 (2) Rs. 53 and the remaining fruits were equally
(3) Rs. 59 (4) Rs. 57 distributed among R and S. What fraction of
(5) Rs. 62 fruits did R get?
165. Mr. Shahs monthly income is Rs. 54550. In 1 3
an entire year, he spends 32% of his annual (1) (2)
4 8
salaries on groceries, he spend 12% on
repairs and 10% he pays to his servant. If half 1 5
(3) (4)
of the remaining amount he invests in fixed 8 16
deposits, what is the amount invested by him (5) Other than those given as optons
in fixed deposits?
170. The present population of village P is 2.5
(1) Rs. 150558
times the present population of village Q. If
(2) Rs. 155240
after a year the population of village Q is
(3) Rs. 152610
16537 and has been increased at a rate of
(4) Rs. 158789
15%. What is the present population of
(5) Rs. 154336
village P?
Buy Printed Study Material for IBPS, SBI Bank Exams
http://www.bankexamportal.com/study-kit
Join Online Coaching for IBPS, SBI Bank Exams
http://www.bankexamportal.com/elearning
EXAM PAPER IBPS Clerk Common Written Exam (Held on: 20-12-2015) 18
(1) 34740 (2) 38560 two scheme (A and B) for two years. Scheme
(3) 36820 (4) 35950 A offers compound interest (compouned
(5) 30350 annualy) at the rate of 10% per annum and
Directions (171-175): What will come in place of scheme B offers simple interest at the rate of
question mark (?) in the given number series? 12% per annum. If the total interest earned
by him from both the scheme after two years
171. 29 31 37 49 69 ? is Rs. 3504. How mauch much monty
(1) 108 (2) 99 (principal) did he invested in scheme B?
(3) 94 (4) 103 (1) Rs. 4800 (2) Rs. 4200
(5) 88 (3) Rs. 4600 (4) Rs. 4400
172. 13 13 20 37.5 83 ? (5) Rs. 5200
(1) 233 (2) 216 179. Ravi is older than Simar by 4 yr. Four years
(3) 234 (4) 235 from now, the respective ratio between Ravi's
(5) 239 age and Simar's age will be 9:8. What will be
173. 17 16 30 87 344 ? the Ravi's are (in yr) 15 yr ago?
(1) 1735 (2) 1760 (1) 19 (2) 36
(3) 1660 (4) 1685 (3) 17 (4) 25
(5) 1715 (5) 21
174. 8 9.4 12.2 17.8 29 ? 180. A started a business by investing Rs. 33600.
(1) 53.6 (2) 51.4 After three month B joinede by investing Rs.
(3) 52.1 (4) 48.6 23100. After 3 month of B's investment. C
(5) 49.8 joined them by investing Rs. 18900. If the
175. 26 12 11 15.5 30 ? total profit earned by them is Rs. 26450,
(1) 72 (2) 68 what is C's share of profit?
(3) 74 (4) 82 (1) Rs. 4360 (2) Rs. 4080
(5) 78 (3) Rs. 4260 (4) Rs. 4420
(5) Rs. 4140
176. Arunika brought some articles and sold half
of them at Rs. 22103 thereby making a profit 181. The sum of two numbers is equal to 27 and
of 15%. At what price should sell the rest of their product is equal to 182. What are the
them so as to earn a total profit of 225%? two numbers?
(1) Rs. 25947 (2) Rs. 23528 (1) 15, 12 (2) 11, 16
(3) Rs. 27130 (4) Rs. 24682 (3) 9, 18 (4) 13, 14
(5) Rs. 26240 (5) 19, 8
177. The height of a triangle is equal to the Directions (182-191): What will come in place of
perimeter of a swaure whose diagonal is question mark (?) in the given questions?
9 2 m and the base of the same triangle is 182. 54.2 + 13.52 - 0.52 - 0.5656 - 0.07 = ?
equal to the side of the square whose area is (1) 85.44 (2) 72.12
784m2. What is the area (in m 2) of the (3) 68.32 (4) 76.14
triangle? (5) 66.57
(1) 504 (2) 558 183. 1024 40 + 448 = (?)
3
(3) 478 (4) 522 (1) 8 (2) 14
(5) 496 (3) 16 (4) 12
178. Arunavo invested total sum of Rs. 16000 in (5) 22

Buy Printed Study Material for IBPS, SBI Bank Exams


http://www.bankexamportal.com/study-kit
Join Online Coaching for IBPS, SBI Bank Exams
http://www.bankexamportal.com/elearning
19 EXAM PAPER IBPS Clerk Common Written Exam (Held on: 20-12-2015)
184. (24 16/15 + 32.4)/? = 4
(0.6)3 - (0.4)3
(1) 18 (2) 14.5 191. =?
(3) 12 (4) 16.4 (0.6)3 + (0.4)3
(5) 15.5
19 18 19
185. 255.4 + 542.3 - ? = 1014.3 - 499.4 (1) (2) (3)
38 35 35
(1) 271.5 (2) 290.5
(3) 220.1 (4) 244.8 18 20
(4) (5)
(5) 282.8 37 37
Directions (192-196): Studythe table and answer the
2
1 given questions.
1
2 5
186. 2 2 =? Number of Books Sold by 5
4 Stores During 5 Months
5
Sotres P Q R S T
5 5
(1) (2) Months
14 8
March 213 200 195 253 229
3 8 April 156 208 216 187 175
(3) (4)
10 25 May 177 197 185 181 215
14 June 220 145 235 265 231
(5) July 253 188 278 243 249
20
187. 0.5 5.6 + 2.5 8.5 + 164.85 = ? 192. Number of books sold by store P in May in
(1) 186.95 (2) 188.9 approximately what percent less than the
(3) 182.35 (4) 183.8 number of books sold by store T in July?
(5) 185.6 (1) 35 (2) 25
(3) 21 (4) 29
188. (0.3 + 0.9 + 0.06)(0.1 + 0.4 + 0.04) = ?
(5) 40
(1) 0.936 (2) 0.693
(3) 0.369 (4) 0.963 193. What is the respective ratio between the total
(5) 0.639 number of books sold by store P in April and
June together and total number of boks sold
189. 120% of 675 + 92 = ?% of 1240 + 716
by store T in May and July together?
(1) 20 (2) 15
(1) 49 : 58 (2) 49 : 54
(3) 16 (4) 10 (3) 47 : 58 (4) 43 : 52
(5) 12 (5) 47 : 54
194. If 30% of the total number of books sold by
81 144
25 - 121 store Q, S and T together in April were
Academic books, how many non-academic
190. 1681 =? books were sold by the same stores together
484 in the same month?
(1) 389 (2) 413
(3) 381 (4) 373
1 3 1
(1) (2) (3) (5) 399
25 11 5
195. What is the average number of books sold by
2 1 store R in April, June and July together?
(4) (5)
5 11 (1) 243 (2) 241
Buy Printed Study Material for IBPS, SBI Bank Exams
http://www.bankexamportal.com/study-kit
Join Online Coaching for IBPS, SBI Bank Exams
http://www.bankexamportal.com/elearning
EXAM PAPER IBPS Clerk Common Written Exam (Held on: 20-12-2015) 20
(3) 233 (4) 237 (1) Rs. 38.13
(5) 239 (2) Rs. 25.33
196. What is the difference between total number (3) Rs. 55.08
of books sold by store Q in May and July (4) Rs. 35.30
togehter and total number of books sold by (5) Other than those given as options
store S in March and June togehter? 199. Raju purchases 55o mL of milk everyday. If
(1) 129 (2) 127 cost of 1 L of milk is Rs. 44, how much
(3) 143 (4) 133 amount will he pays in 45 days?
(5) 136 (1) Rs. 1098
197. A train 350 m long takes 36 s to cross a man (2) Rs. 1079
running at a speed of 5 km/h in the direction (3) Rs. 1099 (4) Rs. 1088
opposite to that of train. What is the speed (5) Other than those given as options
of the train? 200. Neha scored 1.2 times as many marks in
(1) 30 km/h (2) 40 km/h Science as in Sanskrit and in Social Science
(3) 24 km/h (4) 34 km/h She scored 20 more marks than Science. If
(5) Other than those given as options she secured 85.5% marks in these three
198. A person invested some money at the rat eof subjects out of a total 600 marks (in the given
6% simle interest. At the end of three years, three subjects only), how much did she
he hot Rs. 900 as SI. If interest is put at the scored in Social Science?
rate of compound interest annually, how (1) 194 (2) 14
much more interest would he got in three (3) 170 (4) 185
years? (5) Other than those given as options

Buy Printed Study Material for IBPS, SBI Bank Exams


http://www.bankexamportal.com/study-kit
Online Coaching for All IBPS Examination
What You will get:
1. All the relevant and required materials of subjects mention in the syllabus like:

100% IBPS Exam Syllabus Covered with MCQs.


Quantitative Aptitude
Reasoning Ability
General English
Hindi Language
General Awareness
Computer Knowledge

2. Special Material based on Current Affair for IBPS Examination

3. Online Tests for Practice will be conducted.

For More Information Click Given below link:


http://bankexamportal.com/online-coaching/ibps-exams
Join Online Coaching for IBPS, SBI Exams
http://www.bankexamportal.com/elearning

IBPS Clerk Online CWE Previous Year Exam Paper - 2014

Subject: Computer Knowledge


1. ......... refers to the unauthorised copying and distibution of software.

(1) Hacking
(2) Software piracy
(3) Software literacy
(4) Cracking
(5) Copyright

2. Which of the following is an operating system?

(1) Linux
(2) Debugger
(3) Mozilla
(4) Google Chrome
(5) Intel 8085

3. A collection of programs that controls how your computer system runs and processes
information is called

(1) operating system


(2) computer
(3) office
(4) compiler
(5) interpreter

4. Which of the following statements is a false conceiving file names?

(1) Every file in the same folder must have a unique name.
(2) The file name comes before the dot (.).
(3) File extension is another name for the type.
(4) The file extension comes before the dot (.) followed by the file name.
(5) Files may share the same name or the same extension, but not both at the same time.

5. Which is not a storage device?

(1) Printer
(2) CD
(3) Hard Disk
(4) Floppy Disk
(5) DVD

Click Here to Buy Printed Study Material for IBPS, SBI Bank Exams
http://www.bankexamportal.com/study-kit Page 1
Join Online Coaching for IBPS, SBI Exams
http://www.bankexamportal.com/elearning

6. ......... tags, placed on animals body can be used to record and truck in a database all of
the animals movements.

(1) POS
(2) RFID
(3) PPS
(4) GPS
(5) Other than those given as options

7. Which of the following cables can transmit data at high speeds?

(1) Flat cables


(2) Coaxial cable
(3) Optic fibre cable
(4) Twisted pair cable
(5) UTP cable

8. The hardware device or software program that sends messages between networks is
known as a

(1) bridge
(2) backbone
(3) router
(4) gateway
(5) Other than those given as options

9. If you are performing Windows 98 operating system to Windows XP you are


performing a(n)

(1) push up
(2) China
(3) patch
(4) pull down
(5) update

10. ......... technology enables you to carry a powerful navigational aid in websites.

(1) Convergence
(2) Locator
(3) Sampling rate
(4) Synchronisation
(5) Global Positioning System

11. Which of the following is not true about RAM?

(1) RAM is the same as hard disk storage.


(2) RAM is a temporary storage area.
(3) RAM is volatile.

Click Here to Buy Printed Study Material for IBPS, SBI Bank Exams
http://www.bankexamportal.com/study-kit Page 2
Join Online Coaching for IBPS, SBI Exams
http://www.bankexamportal.com/elearning

(4) RAM stands for Random Access Memory


(5) Information stored in RAM is gone when you turn the computer off.

12. All of the following are examples of real security and privacy risks EXCEPT

(1) spyware
(2) spam
(3) hackers
(4) identity theft
(5) viruses

13. Which of the following commands in Office 2007, can be used to go to first cell in the
current row?

(1) Tab
(2) Shift + Tab
(3) Esc + Home
(4) Shift + Home
(5) Home

14. Software such as Viruses, Worms and Trojan Horses that has a malicious content, is
known as

(1) malicious software (malware)


(2) adware
(3) scareware
(4) spyware
(5) firewall

15. Chip is a common nickname for a(n)

(1) transistor
(2) resistor
(3) integrated circuit
(4) semi-conductor
(5) other than those given as options

16. What is extension of Microsoft Word document?

(1) doc
(2) ppt
(3) dcc
(4) bd
(5) hml

17. Which of the following enables to determine how often a user visited a website?

Click Here to Buy Printed Study Material for IBPS, SBI Bank Exams
http://www.bankexamportal.com/study-kit Page 3
Join Online Coaching for IBPS, SBI Exams
http://www.bankexamportal.com/elearning

(1) Hackers
(2) Spammers
(3) Phish
(4) Identity thefts
(5) Cookies

18. To move to the bottom of a document while working on MS Word, which command is
used?

(1) Home key


(2) End key
(3) Ctrl + Page Down
(4) Insert key
(5) Ctrl key + End key

19. What is a person called who uses a computer to cause harm to people or destroy
critical systems?

(1) Cyber Terrorist


(2) Black-hat Hacker
(3) Cyber Cracker
(4) Hacktivist
(5) Other than those given as options

20. ........ can interpret voice data into words that can be understood by the computer.

(1) Speech input hardware


(2) Speech recognition software
(3) Word recognition software
(4) Talking software
(5) Other than those given as options

21. Oracle is a(n)

(1) hardware
(2) high level language
(3) operating system
(4) system software
(5) RDBMS

22. Which of the following compay developed the microprocessor?

(1) Intel
(2) IBM
(3) Microsoft
(4) Apple
(5) Google

Click Here to Buy Printed Study Material for IBPS, SBI Bank Exams
http://www.bankexamportal.com/study-kit Page 4
Join Online Coaching for IBPS, SBI Exams
http://www.bankexamportal.com/elearning

23. What does ALU in computing denote?

(1) Application and Logic Unit


(2) Algorithm Logic Unit
(3) Arithmetic Layered Unit
(4) Arithmetic Legal Unit
(5) Arithmetic Logic Unit

24. Integrated Circuit (IC) chips used in computers are made with

(1) copper
(2) aluminium
(3) gold
(4) silicon
(5) silver

25. What does the computer abbreviation MB used for?

(1) Mega Bit


(2) Million Bytes
(3) Mega Bytes
(4) Million Bit
(5) Micro Bytes

26. What is the full form of HTTP?

(1) Hyper Text Transfer Protocol


(2) Hyper Text Transition Protocol
(3) Hyper Text Transfer Program
(4) Hyper Text Transition Program
(5) Hyper Text Trivial Protocol

27. Verification of a login name and password is known as

(1) configuration
(2) accessibility
(3) authentication
(4) logging in
(5) other than those given as options

28. Which of the following is software?

(1) Keyboard
(2) Internet Explorer
(3) Scanner
(4) Mouse
(5) Printer

Click Here to Buy Printed Study Material for IBPS, SBI Bank Exams
http://www.bankexamportal.com/study-kit Page 5
Join Online Coaching for IBPS, SBI Exams
http://www.bankexamportal.com/elearning

29. The term bit is short for

(1) binary digit


(2) binary number
(3) binary language
(4) small digit
(5) one byte

30. What does MICR stand for?

(1) Magnetic Ink Character Register


(2) Magnetic Ink Code Reader
(3) Magnetic Ink Code Register
(4) Magnetic Ink Character Recognition
(5) Magnetic Ink Cases Reader

31. RTGS stands for

(1) Real Time Gross Settlement


(2) Real Time General Settlement
(3) Run Time Gross Settlement
(4) Regular Time General Settlement
(5) Regular Time Gross Settlement

32. A device that converts from decimal binary number is known as

(1) instructor
(2) decoder
(3) AND gate
(4) converter
(5) decoder

33. What does the acronym ASCI stands for?

(1) American Standard Code for Information


(2) American Standard Code for Information interchange
(3) American Standardised Code for information
(4) American Standardisation Code for information
(5) American Standardisation Code for information Interchanger

34. Which of the following is not a valid formula in Microsoft Excel?

(1) = A2 + A1
(2) = A2 + 1
(3) = 1 + A2
(4) = 1A + 2
(5) = A1 + A2

Click Here to Buy Printed Study Material for IBPS, SBI Bank Exams
http://www.bankexamportal.com/study-kit Page 6
Join Online Coaching for IBPS, SBI Exams
http://www.bankexamportal.com/elearning

35. What does the acronym ISP stand for?

(1) Internal Service Provider


(2) International Service Provider
(3) Internal Service Provider
(4) Internet Service Providing
(5) Internet Service Provision

36. What is the full form of WWW?

(1) Work Wide Web


(2) World Wide Web
(3) World With Web
(4) World Wide Wet
(5) World Weak Web

37. Which of the following is hardware?

(1) Assembler
(2) Server
(3) Compiler
(4) Interpreter
(5) Operating System

38. USB in data cables stands for

(1) Unicode Smart Bus


(2) Universal Structural Bus
(3) Unicode Serial Bus
(4) Universal Smart Bus
(5) Universal Serial Bus

39. Which of the following is not a binary number?

(1) 11101
(2) 110
(3) 233
(4) 1001
(5) 100110

40. A ......... is approximately one billion bytes.

(1) megabyte
(2) kilobyte
(3) terabyte
(4) gigabyte
(5) bit

Click Here to Buy Printed Study Material for IBPS, SBI Bank Exams
http://www.bankexamportal.com/study-kit Page 7
Join Online Coaching for IBPS, SBI Exams
http://www.bankexamportal.com/elearning

Subject: Numerical Ability


1. 161. In an examination, a student scores 6 marks for every correct answer and loses 4
marks for every wrong answer. If he attempted 80 questions and obtained 310 marks, how
many questions did he attempted correctly?

(1) 59
(2) 67
(3) 63
(4) 65
(5) 61

2. The diameter of a wheel is 49 m. How many revolutions will it make to cover a distance
of 3200 m?

(1) 17
(2) 27
(3) 24
(4) 22
(5) 18

3. The average run of a cricketer after 18 matches was 56.5. If he made 101 runs and 123
runs in 19th and 20th match respectively. What is his new average run after 20th match?

(1) 62.05
(2) 64.45
(3) 60.75
(4) 61.25
(5) 63.85

4. Two types of rice (type 1 and type 2) were mixed in the respective ratio of 1 : 3. The
mixture was then sold @ 75.60 per kg to gain a profit of 20%. If the price of type 1 rise is
Rs. 75 per kg, what is the price of type 2 price per kg?

(1) Rs. 55
(2) Rs. 53
(3) Rs. 59
(4) Rs. 57
(5) Rs. 62

5. Mr. Shahs monthly income is Rs. 54550. In an entire year, he spends 32% of his annual
salaries on groceries, he spend 12% on repairs and 10% he pays to his servant. If half of
the remaining amount he invests in fixed deposits, what is the amount invested by him in
fixed deposits?

Click Here to Buy Printed Study Material for IBPS, SBI Bank Exams
http://www.bankexamportal.com/study-kit Page 8
Join Online Coaching for IBPS, SBI Exams
http://www.bankexamportal.com/elearning

(1) Rs. 150558


(2) Rs. 155240
(3) Rs. 152610
(4) Rs. 158789
(5) Rs. 154336

6. Two pipes A and B can fill a tank in hours when opened simultaneously. If B alone can
takes 2 hours less than A alone takes to fill the tank completely. How much does A alone
take to fill the tank?

(1) 8 h
(2) 12 h
(3) 4 h
(4) 6 h
(5) 10 h

7. A man can row 10.2 km downstream in 18 minutes. If the speed of the stream is 3.5
km/h, how much time he would take to cover 121.5 km upstream? (in hours)

8. The respective ratio of two numbers is 16 : 21. If the first number is increased by 30%
and the second number is decreased by 20%, what will be the respective ratio of the first
and the second number?

(1) 32 : 21
(2) 26 : 21
(3) 25 : 21
(4) 20 : 21
(5) 22 : 21

9. A bag of fruits was distributed among 4 students P, Q, R and S. P took 3/8th of the
fruits. Q took 1/5th of the remaining fruits and the remaining fruits were equally
distributed among R and S. What fraction of fruits did R get?

Click Here to Buy Printed Study Material for IBPS, SBI Bank Exams
http://www.bankexamportal.com/study-kit Page 9
Join Online Coaching for IBPS, SBI Exams
http://www.bankexamportal.com/elearning

10. The present population of village P is 2.5 time the present population of village Q. If
after a year the population of village Q is 16537 and has been increased at a rate of 15%.
What is the present population of village P?

(1) 34740
(2) 38560
(3) 36820
(4) 35950
(5) 30350

Directions (Q. Nos. 11-15): What will come in place of question mark (?) in the given number series?

11. 29 31 37 49 69 ?

(1) 108
(2) 99
(3) 94
(4) 103
(5) 88

12. 13 13 20 37.5 83 ?

(1) 233
(2) 216
(3) 234
(4) 235
(5) 239

13. 17 16 30 87 344 ?

(1) 1735
(2) 1760
(3) 1660
(4) 1685
(5) 1715

14. 8 94 12.2 17.8 29 ?

Click Here to Buy Printed Study Material for IBPS, SBI Bank Exams
http://www.bankexamportal.com/study-kit Page 10
Join Online Coaching for IBPS, SBI Exams
http://www.bankexamportal.com/elearning

(1) 53.6
(2) 51.4
(3) 52.1
(4) 48.6
(5) 49.8

15. 26 12 11 15.5 30 ?

(1) 72
(2) 68
(3) 74
(4) 82
(5) 78

16. Arunika brought some articles and sold half of them at Rs. 22103 thereby making a
profit of 15%/ At what price should sell the rest of them so as to earn a total profit of
25%?

(1) Rs. 25947


(2) Rs. 23528
(3) Rs. 27130
(4) Rs. 24682
(5) Rs. 26240

17. The height of a triangle is equal to the perimeter of a square whose diagonal is m and
the base of the same triangle is equal to the side of the square whose area is 784 m2. What
is the area of the triangle? (in m2)

(1) 504
(2) 558
(3) 478
(4) 522
(5) 496

18. Arunavo invested total sum of Rs. 16000 in two schemes (A and B) for two years.
Scheme A offers compound interest (compounded annually) at the rate of 10% per annum
and scheme B offers simple interest at the rate of 12% per annum. If the total interest
earned by him from both the schemes after two years is Rs. 3504. How much money
(principle) did he invest in scheme B?

(1) Rs. 4800


(2) Rs. 4200
(3) Rs. 4600
(4) Rs. 4400
(5) Rs. 5200

Click Here to Buy Printed Study Material for IBPS, SBI Bank Exams
http://www.bankexamportal.com/study-kit Page 11
Join Online Coaching for IBPS, SBI Exams
http://www.bankexamportal.com/elearning

19. Ravi is older than Simar by 4 years. Four years from now, the respective ratio between
Ravis age and Simars age will be 9:8. What will be the Ravis age 15 years ago? (in years)

(1) 19
(2) 36
(3) 17
(4) 25
(5) 21

20. A started a business by investing Rs. 33600. After three month B joined him by
investing Rs. 23100. After 3 months of Bs investment, C joined them by investing Rs.
18900. If the total annual profit earned by them is Rs. 26450, what is Cs share of profit?

(1) Rs. 4630


(2) Rs. 4080
(3) Rs. 4260
(4) Rs. 4420
(5) Rs. 4140

21. The sum of two numbers is equal to 27 and their product is equal to 182. What are the
two numbers?

(1) 15, 12
(2) 11, 16
(3) 9, 18
(4) 13, 14
(5) 19, 8

Direction (Q. Nos. 22-31): What will come in place of question mark (?) in the given questions?

22. 54.2 + 13.52 0.52 0.5656 0.07 = ?

(1) 85.44
(2) 72.12
(3) 68.32
(4) 76.14
(5) 66.57

23.

(1) 8
(2) 14
(3) 16
(4) 12
(5) 22

Click Here to Buy Printed Study Material for IBPS, SBI Bank Exams
http://www.bankexamportal.com/study-kit Page 12
Join Online Coaching for IBPS, SBI Exams
http://www.bankexamportal.com/elearning

24. (24 16/15 + 32.4)/? = 4

(1) 18
(2) 14.5
(3) 12
(4) 16.5
(5) 15.5

25. 255.4 + 542.3 - ? = 1014.3 499.4

(1) 271.5
(2) 290.5
(3) 220.10
(4) 244.8
(5) 282.8

26.

27. 0.5 5.6 + 2.5 8.5 + 164.85 = ?

(1) 186.95
(2) 188.9
(3) 182.35
(4) 183.8
(5) 185.6

28. (0.3 + 0.9 + 0.06)(0.4 + 0.4 + 0.05) = ?

(1) 0.936
(2) 0.693
(3) 0.369
(4) 0.963
(5) 0.639

29. 120% of 675 + 92 = ?% of 124444440 + 716

Click Here to Buy Printed Study Material for IBPS, SBI Bank Exams
http://www.bankexamportal.com/study-kit Page 13
Join Online Coaching for IBPS, SBI Exams
http://www.bankexamportal.com/elearning

(1) 20
(2) 15
(3) 16
(4) 10
(5) 12

30.

31.

Directions (32-36): Study the table and answer the given questions.

Click Here to Buy Printed Study Material for IBPS, SBI Bank Exams
http://www.bankexamportal.com/study-kit Page 14
Join Online Coaching for IBPS, SBI Exams
http://www.bankexamportal.com/elearning

32. Number of books sold by store P in May is approximately what percent less than the
number of books sold by store T in July?

(1) 35
(2) 25
(3) 21
(4) 29
(5) 40

33. What is the respective ratio between the total number of books sold by store P in April
and June together and total number of books sold by store T in May and July together?

(1) 49 : 58
(2) 49 : 54
(3) 47 : 58
(4) 43 : 52
(5) 47 : 54

34. If 30% of the total number of books sold by store Q, S and T together in April were
Academic books, how many non-academic books were sold by the same stores together in
the same month?

(1) 389
(2) 413
(3) 381
(4) 373
(5) 399

35. What is the average number of books sold by store R in April, June and July together?

(1) 243
(2) 241
(3) 233
(4) 237
(5) 239

36. What is the difference between total number of books sold by store Q in May and July
together and total number of books sold by store in S in March and June together?

(1) 129
(2) 127
(3) 143
(4) 133
(5) 136

37. A train 350 m long takes 36 seconds to cross a man running at a speed of 5 km/h in the
direction opposite to that of train. What is the speed of the train?

Click Here to Buy Printed Study Material for IBPS, SBI Bank Exams
http://www.bankexamportal.com/study-kit Page 15
Join Online Coaching for IBPS, SBI Exams
http://www.bankexamportal.com/elearning

(1) 30 km/h
(2) 40 km/h
(3) 24 km/h
(4) 34 km/h
(5) Other than those given as options

38. A person invested some money at the rate of 6% simple interest. At the end of three
years, he got Rs. 900 as SI. If interest is put at the rate of compound interest annually, how
much more interest would he got in three years?

(1) Rs. 38.13


(2) Rs. 25.33
(3) Rs. 55.08
(4) Rs. 35.30
(5) Other than those given as options

39. Raju purchases 550 ml of milk everyday. If cost of 1 litre of milk is Rs. 44, how much
amount will he pay in 45 days?

(1) Rs. 1098


(2) Rs. 1079
(3) Rs. 1099
(4) Rs. 1088
(5) Other than those given as options

40. Neha scored 1.2 times as many marks in Science as in Sanskrit and in Social Science.
She scored 20 more marks than Science. If she secured 85.5% marks in these three
subjects out of a total 600 marks (in the given three subjects only), how much did she score
in Social Science?

(1) 194
(2) 174
(3) 170
(4) 185
(5) Other than those given as options

Click Here to Buy Printed Study Material for IBPS, SBI Bank Exams
http://www.bankexamportal.com/study-kit Page 16
Join Online Coaching for IBPS, SBI Exams
http://www.bankexamportal.com/elearning

Subject: General Knowledge


1. India has recently signed a deal to build a $ 100 billion 900 MW hydro-power project on
Arun river with

(1) Bhutan
(2) Bangladesh
(3) Myanmar
(4) Nepal
(5) China

2. Indian player Anirban Lahiri is associated with the game of

(1) rifle shooting


(2) golf
(3) billiards
(4) wrestling
(5) chess

3. The abbreviation ATS stand for

(1) Anti Terrorism Sensor


(2) Anti Terrorist Set-up
(3) Anti Terrorist System
(4) Anti Terrorism Squad
(5) Anti Terrorism Scheme

4. China has recently announced that it has completed a major hydro-power dam in Tibet
over the river Yartung Zangbo known in India as

(1) Indus river


(2) Brahmaputra river
(3) Zanskar river
(4) Ranganadi river
(5) Doyang river

5. According to which of the following acts, a fixed deposit in a bank should not be paid in
cash if it is Rs. 20000 and above?

(1) Wealth Tax Act


(2) RBI Act
(3) Income Tax Act
(4) Banking Regulation Act
(5) Negotiation Instruments Act

Click Here to Buy Printed Study Material for IBPS, SBI Bank Exams
http://www.bankexamportal.com/study-kit Page 17
Join Online Coaching for IBPS, SBI Exams
http://www.bankexamportal.com/elearning

6. The Commonwealth Association for Public Administration and Managements


(CAPAM) International Innovation Awards for 2014 has recently been confered upon the
Indian State.

(1) Uttar Pradesh


(2) Andhra Pradesh
(3) Gujarat
(4) Karnataka
(5) Maharashtra

7. In 201 6, the Olympic Games shall be held in

(1) Beijing (China)


(2) Tokyo (Japan)
(3) Rio de Janeiro (Brazil)
(4) Seoul (South Korea)
(5) Bangkok (Thailand)

8. Banks borrow money from the RBI at which of the following rates?

(1) CRR
(2) Base Rate
(3) Repo Rate
(4) SLR
(5) Reverse Repo Rate

9. Which of the following is not an online travel

(1) Goibibo.com
(2) Thomascook.in
(3) Zomato.com
(4) Arzoo.co
(5) Expedia.co.in

10. Under the PMJDY, the government is loking to open at least ......... basic bank accounts
for unbanked tambles in the country.

(1) 7.5 crore


(2) 1.8 crore
(3) 5.7 crore (4) 3 crore
(5) 4.5 crore

11. In order to achieve financial inclusion goals, RBI has permitted opening of USBs. What
is the full form of USB?

(1) Urban Small Branch


(2) Unique Safety Branch
(3) Ultra Small Branch

Click Here to Buy Printed Study Material for IBPS, SBI Bank Exams
http://www.bankexamportal.com/study-kit Page 18
Join Online Coaching for IBPS, SBI Exams
http://www.bankexamportal.com/elearning

(4) Other than those given as options


(5) United Smaller Branches

12. As per recent press in India, White Label ATMs are installed. What is meant by a
white lebel ATM?

(1) ATMs, which are set-up by non-banking entities.


(2) ATMs, which are white in colour.
(3) ATMs, which are set-up by group of foreign banks operating in India.
(4) ATMs, which are set-up by Reserve Bank of India.
(5) ATMs, which are installed by group of bankers.

13. As per the recent press reports, RBI has recently converyed the in-principle approval
to open new bank. Which of the following entity has received the permission?

(1) Tata Group


(2) LIC Housing Finance
(3) Anil Ambani Group
(4) Videocon Group
(5) IDFC

14. Which of the following is present Chief Economic Advisor of India?

(1) Pranjul Bhandari


(2) Dr. prachi Mishra
(3) Arvind Subramanian
(4) Dr. Kaushik Basu
(5) Supriyo De

15. By the year 2022, the Sardar Patel Urban Housing Mission aims to build as many as

(1) 25 million houses


(2) 20 million houses
(3) 30 million houses
(4) 35 million houses
(5) 15 million houses

16. The private sector banking space witnessed its first consolidation move in four years.
Which of the following bank merger took place recently?

(1) United Western Bank and IDBI Bank


(2) Ganesh Bank of Kurundwad and Federal Bank
(3) ING Vysya Bank and Kotak Mahindra Bank
(4) Bank of Rajasthan and ICICI Bank
(5) State Bank of Indore and State Bank of India

17. Who bears the premium payable to the insurance company under acidental insurance
cover under PMJDY scheme?
Click Here to Buy Printed Study Material for IBPS, SBI Bank Exams
http://www.bankexamportal.com/study-kit Page 19
Join Online Coaching for IBPS, SBI Exams
http://www.bankexamportal.com/elearning

(1) Government of India


(2) 50% by account holder and rest by the bank
(3) NPCI
(4) Account holder
(5) RBI

18. Buy Now Pay Now refers to which of the following?

(1) Affinity Card


(2) Smart Card
(3) Business Card
(4) Credit Card
(5) Debit Card

19. The government of India has decided to observe the birth anniversary of Sardar
Vallabhai Patel on October 31, as

(1) Rashtriya Sankalp Diwas (National Resolve Day)


(2) Rashtriya Shahidi Diwas (National Martyrs Day)
(3) Rashtriya Sadbhavana Diwas (National Solidarity Day)
(4) Rashtriya Ekta Diwas (National Unity Day)
(5) Rashtriya Shiksha Diwas (National Education Day)

20. The international day for Elimination of Violence against Women is observed across
the world on

(1) December 15
(2) August 24
(3) June 24
(4) November 25
(5) September 24

21. Time limit for exchanging of pre-2005 Indian currency notes is

(1) April 1, 2015


(2) December 31, 2014
(3) January 1, 2015
(4) June 1, 2015
(5) September 1, 2015

22. Kangto also known as Kanggardo Rize is a montain in the Eastern Himalays of
India and is highest peak in the State of

(1) Sikkim
(2) Arunachal Pradesh
(3) Jammu and Kashmir
(4) Uttarakhand
(5) Himanchal Pradesh

Click Here to Buy Printed Study Material for IBPS, SBI Bank Exams
http://www.bankexamportal.com/study-kit Page 20
Join Online Coaching for IBPS, SBI Exams
http://www.bankexamportal.com/elearning

23. The Parliamentary Constituency in Uttar Pradesh represented in the 16th Lok Sabh by
Rajnath Singh, the Union Minister of Home Affairs is

(1) Lucknow
(2) Mirzapur
(3) Kanpur
(4) Haidergarh
(5) Chandauli

24. Balance in deposit accounts which are not operated for at least ........ years are to be
transferred to the RBI.

(1) 2
(2) 7
(3) 12
(4) 5
(5) 10

25. The 18th SAARC meet was held in November, 2014 at

(1) Dhaka (Bangladesh)


(2) Kathmandu (Nepal)
(3) Thimpu (Bhutan)
(4) Colombo (Sri Lanka)
(5) New Delhi (India)

26. Many times we read about SHGs in financial newspapers. What is the full form of the
term SHGs?

(1) Self Help Groups


(2) Other than those given as options
(3) Small Help Groups
(4) Small Hope in Growths
(5) Self Hope Groups

27. Which of the following institutions provides secured depository services?

(1) Other than those given as options


(2) RBI
(3) NSDL
(4) BSE
(5) NSE

28. Which Indian women sportsperson has recently been appointed as the UN Goodwil
Ambassador for the South Asian region?

(1) Mary Kom


(2) Sania Mirza

Click Here to Buy Printed Study Material for IBPS, SBI Bank Exams
http://www.bankexamportal.com/study-kit Page 21
Join Online Coaching for IBPS, SBI Exams
http://www.bankexamportal.com/elearning

(3) Sikha Tandon


(4) Jwala Gutta
(5) Dipika Pallikal

29. The United Nations Childrens Fund (UNICEF)

(1) Washington DC (USA)


(2) Vienna (Austria)
(3) Paris (France)
(4) Geneva (Switzerland)
(5) New York City (USA)

30. IRNSS is an independent regional navigation satellite system being developed by

(1) Japan
(2) the USA
(3) Brazil
(4) China
(5) India

31. The first Commonwealth Science Conference was recently held in

(1) Singapore
(2) Bengaluru (India)
(3) Glasgow (Scotland)
(4) Kuala Lumpur (Malaysia)
(5) Colombo (Sri Lanka)

32. Typhoon Rammasun is

(1) China
(2) Singapore
(3) India
(4) Japan
(5) Myanmar

33. The currency of Sultanate of Oman is

(1) Omani Rand


(2) Omani Rial
(3) Omani Dinar
(4) Omani Gulider
(5) Omani Shilling

34. Process of getting shares in an electronic form of holding in a demat account is known
as

Click Here to Buy Printed Study Material for IBPS, SBI Bank Exams
http://www.bankexamportal.com/study-kit Page 22
Join Online Coaching for IBPS, SBI Exams
http://www.bankexamportal.com/elearning

(1) Rematerialisation
(2) Other than those given as options
(3) Dematerialisation
(4) Materialisation
(5) Mutilsation

35. Which of the following is the Capital of the State of Kuwait?

(1) Al Ahmadi
(2) Al Jahra
(3) Kuwait City
(4) Al Salmiya
(5) Hawalli

36. The Davis Cup is the premier international team event in

(1) Mens Tennis


(2) Mens Hockey
(3) Mens Soccer
(4) Womens Hockey
(5) Womens Cricket

37. The Sansad Adarsh Gram Yojana has been launched on October 11, 2014, the birth
anniversary of

(1) Jai Prakah Narayan


(2) Shyam Prasad Mukherji
(3) lala Lajpat Rai
(4) Deen Dayal Upadhyaya
(5) Balraj Madhok

38. The Minister of State (Independent Charge) for Skill Development and
Entrepreneurship in the revamped Union Council of Ministers is

(1) Mahesh Sharma


(2) Rajiv Pratap Rudy
(3) JP Nadda
(4) Birender Singh
(5) Najma Heptulla

39. The theme of the next Pravasi Bhartiya Divas to be held in the year 2015 is based on
the completion of the hundred years of Mahatma Gandhis

(1) return to South Africa from India


(2) marriage with Kasturba Gandhi
(3) Quit India call to the British users
(4) return to India leaving South Africa
(5) founding the Natal India Congress

Click Here to Buy Printed Study Material for IBPS, SBI Bank Exams
http://www.bankexamportal.com/study-kit Page 23
Join Online Coaching for IBPS, SBI Exams
http://www.bankexamportal.com/elearning

40. According to the 2011 census, the highest literacy rate is recorded by

(1) Delhi
(2) Tripura
(3) Mizoram
(4) Kerala
(5) Goa

Click Here to Buy Printed Study Material for IBPS, SBI Bank Exams
http://www.bankexamportal.com/study-kit Page 24
Join Online Coaching for IBPS, SBI Exams
http://www.bankexamportal.com/elearning

Subject: English Language


Directions (Q. Nos. 1-5): Choose the best word that fits in the meaning of the sentences to make
grammatical coherent.

1.

I. Governments in `these countries should create education systems since ......... good schools the bulk of
people entering the world force will not have the skills they need.
II. We cannot process colour, details of rapid changes in our surroundings ......... the cone shaped cells
packed around the centre of the retina.

(1) lack
(2) missing
(3) eficient
(4) without
(5) absence

2.

I. Japan is investing in strengthening bonds with ASEAN countries and Japanese countries recently won a $
370 million contract to start .......... a new underground railway system in Jakarta.

II. The cost of ......... space vehicles is high no doubt, but the costs of complying with legal and regulatory
paperwork and rules is huge too.

(1) construction
(2) manufacture
(3) launch
(4) assembly
(5) building

3.

I. A troubled student and his PhD guide ........... a way for websites to reduce the incidence of spam which
troubled internet users in the 2000s.
II. One of the biggest drawback of concrete is that it often develop cracks and scientists have now ..........
self-healing concrete.

(1) create
(2) devised
(3) develop
(4) thought
(5) invent

4.

Click Here to Buy Printed Study Material for IBPS, SBI Bank Exams
http://www.bankexamportal.com/study-kit Page 25
Join Online Coaching for IBPS, SBI Exams
http://www.bankexamportal.com/elearning

I. Politicians have promised to ........ the goal of eradicating extreme poverty many times but have failed
because they cannot agree about what exactly counts as poverty and how exactly to measure it.
II. Reserve Bank hoped that a cut in its benchmark rate will .......... a reduction in inflation and improve
confidence in the economy.

(1) achieve
(2) get
(3) end
(4) finish
(5) undertake

5.

I. According to some exports, subsidising crop insurance to a large extent may be bad for the environment
since farmers may take .......... such as farming on food plains or sleep hills.
II. In the 1990s, Germany was known as the sick man of Europe and had high unemployment but its
success today is on account of the huge ........ it took in reforming the labour market.

(1) danger
(2) chance
(3) possibility
(4) risks
(5) threats

Directions (Q. Nos. 6-10): Arrange the following five sentences A, B, C, D and E in the proper sequence to
form a meaningful paragraph, then answer the given questions:

A. If you are transitioning from products to services or vice-versa, you have to know and understand these
differences to effectively promote and sell.
B. For example, a product is tangible, which means the customer can touch and see the product before
deciding to make a purchase and a service is intangible.
C. Understanding the different changes in product and service marketing can help you establish the right
approach for this transition.
D. Unless you understand the basic difference of tangibility, it will be a challenge to promote and sell your
product.
E. Companies that are marketing a product face different challenges compared to those that are promoting a
service.

6. Which of the following should be the FIFTH (LAST) sentence after the rearrangement?

(1) A
(2) E
(3) D
(4) C
(5) B

7. Which of the following should be the FIRST sentence after the rearrangement?

Click Here to Buy Printed Study Material for IBPS, SBI Bank Exams
http://www.bankexamportal.com/study-kit Page 26
Join Online Coaching for IBPS, SBI Exams
http://www.bankexamportal.com/elearning

(1) A
(2) B
(3) C
(4) D
(5) E

8. Which of the following should be the SECOND sentence after the rearrangement?

(1) A
(2) B
(3) C
(4) D
(5) E

9. Which of the following should be the THIRD sentence after the rearrangement?

(1) A
(2) B
(3) C
(4) D
(5) E

10. Which of the following should be the FOURTH sentence after the rearrangement?

(1) A
(2) B
(3) C
(4) D
(5) E

Directions (Q. Nos. 11-15): Which of the phrase given against the sentence should replace the word/phrase
given in bold in sentence to make it grammatically correct? If the sentence is given as it is given and no
correction is required, mark No correction required as the answer.

11. In order to look taller, we should use pencil heels instead block heels.

(1) instead of
(2) despite
(3) in spite of
(4) neither
(5) No correction required

12. This move is expected for mitigate the effects of a depleting water by making use of
surface or canal water.

(1) expects of
(2) is expectation of
(3) is expected to

Click Here to Buy Printed Study Material for IBPS, SBI Bank Exams
http://www.bankexamportal.com/study-kit Page 27
Join Online Coaching for IBPS, SBI Exams
http://www.bankexamportal.com/elearning

(4) is expectant to
(5) No correction required

13. Do you know that dressing properly for an interview can earns you those brownie
extra points which will help you get that dream job?

(1) can earn


(2) will earning
(3) earned
(4) well earns
(5) No correction required

14. The craze for private hands has caught the fancy of the citys residents.

(1) catch the fancity


(2) caught fancity
(3) catching fancy
(4) catch fancy
(5) No correction required

15. The trend of wearing boots in something that has most sure come around this season.

(1) most surely


(2) caught fancity
(3) catching fancy
(4) catch fancy
(5) No correction required

Directions (Q. Nos. 16-25): Read the following passage carefully and answer the questions given.

Do you ever feel theres is a greater being inside of you bursting to get out? It is the voice that encourages
you to really make something of your life. When you act congruently with that voice, its like your are a
whole new person. You are bold and courageous. You are strong. You are unstoppable. But, then reality sets
in, and soon those moments are history. It is not hard to put youself temporarily into an emotionally
motivated state. Just listen to that motivational song for that matter. However, this motivation does not stay
forever. Your great ideas seem impractical. How many times have you been temporarily inspired with a idea
like, I want to start my own business. And then a week later its forgotten? You come up with inspiring
ideas when you are motivated. But you fail to maintain that motivation through the action phase.

The problem we ask ourselves is, why does this happen? You can listen to hundereds of motivational
speakers and experience an emotional yo-yo effect, but it does not fast. The problem is that as we are
intellectually guided, we try to find logic in emotional motivation and as we fail to find logic eventually
phases out. I used to get frustrated when my emotional motivation fizzled out after a while. Eventually, I
realised that being guided by intellect, was not such a bad thing after all. I just had to learn to use my mind
as an effective motivational tool. I figured that if I was not feeling motivated to go after a particular goal,
may be there was a logical reason for it. I noted that when I had strong intellectual reasons for doing
something. I usually did not have trouble taking action.

Click Here to Buy Printed Study Material for IBPS, SBI Bank Exams
http://www.bankexamportal.com/study-kit Page 28
Join Online Coaching for IBPS, SBI Exams
http://www.bankexamportal.com/elearning

But when my mind thinks a goal is wrong on some level. I usually feel blocked. I eventually realised that
this was my minds way of telling me the goal was a mistake to begin with. Sometimes a goal seem to make
sense on one level but when you look further upstream, it becomes clear that the goal is ill advised. Suppose
you work in sales, and you get a goal to increase your income by 20% by becoming a more effective
salesperson. That seems like a reasonable and intelligent goal. But may be you are surprised to find yourself
encountering all sorts of internal blocks when you try to pursue it. You should feel motivated, but you just
dont. The problem may be that on a deeper level your mind knows you dont want to be working in sales at
all. You really want to be a musician. Matter how hard you push yourself in sales career, it will always be a
motivational dead end.
Further when you set goals, that are too small and too timid, you suffer a perpetual lack of motivation. You
just need to summon the courage to acknowledge your true desires. Then you will have to deal with the self-
doubt and fear thats been making you think too small. Ironically, the real key to motivation is to set the
goals that scare you. You are letting fears, excuses and limiting beliefs hold you back. Your subconscious
mind knows you are strong, so it wont provide any motivational fuel until. You step up, face your fears, and
acknowledge your hearts desire. Once you finally decide to face your tears and drop the excuses, then you
will find your motivation turning on full blast.

16. What does the author want to convey when he says, When you look further upstream,
it becomes clear that the goal is ill advised.?

(1) When you analyse your goal closely you realise that it is not what you thought it would be.
(2) When you work towards dealing with your problems eventually they disappear
(3) When you inspect the problem you realise that the solution is within our reach
(4) When you devise a method of motivating self you must keep analysing it periodically
(5) When you face a problem you realise your incapability of making good decisions.

17. Which of the following is/are true in the context of the passage?

A. Many a time, although we are motivated at the start, we fail to keep up the motivation while working
towards the goal.
B. We can learn to use our mind as a motivational tool.
C. Being guided by intellect is very bad.
(1) A and B
(2) B and C
(3) Only A
(4) A and C
(5) All of these

18. According to the passage, the key to motivation is

A. setting challenging goals.


B. not letting our fears pull us back.
C. changing our goals periodically.

(1) A and B
(2) Only C
(3) All of these

Click Here to Buy Printed Study Material for IBPS, SBI Bank Exams
http://www.bankexamportal.com/study-kit Page 29
Join Online Coaching for IBPS, SBI Exams
http://www.bankexamportal.com/elearning

(4) A and C
(5) Only B

19. Which of the following is most nearly the same in meaning to the word timid as used in
the passage?

(1) Sudden
(2) Scared
(3) Humble
(4) Distant
(5) Egoistic

20. Which of the following is/are not true about the context of the passage?

A. Although a goal may look intellectual, it may not actually work for us.
B. Our subconscious mind will not motivate us unless we face our fears.
C. The only way to keep yourself motivated throughout is by listening to many motivational speakers.

(1) B and C
(2) Only C
(3) Only A
(4) A and B
(5) Only B

21. According to the passage, the author found it difficult to be motivated because

A. he was not intellectually motivated.


B. he wanted to take the easy way out
C. he did not push himself hard enough

(1) A and B
(2) Only C
(3) Only A
(4) A and C
(5) Only B

22. Which of the following is most nearly the same in meaning to the word congruently as
used in the passage?

(1) Periodically
(2) In delusion
(3) In addition
(4) Progressively
(5) In agreement

23. What does the author mean when he says, It eventually phases out.?

Click Here to Buy Printed Study Material for IBPS, SBI Bank Exams
http://www.bankexamportal.com/study-kit Page 30
Join Online Coaching for IBPS, SBI Exams
http://www.bankexamportal.com/elearning

(1) We need continuous practice in phases to retain it.


(2) It reduces because of lack of practice
(3) We become self-motivated with time
(4) It goes after a period of time passes
(5) It improves after a certain period passes.

24. Which of the following can be an appropriate title for the passage?

(1) Living Life with Success


(2) Why Dont We Feel Motivated?
(3) Our Only Aim-Success
(4) How to Achieve Intellectual Success
(5) Feel Motivated-Feel Superior

25. According to the passage, what does the author say about emotional motivation?

A. It tends to be temporary
B. It promises our negativity
C. It has the potential to inspire us.

(1) A and B (2) Only B


(3) Only A (4) A and C
(5) All of these

Directions (Q. Nos. 26-30): Read the following sentences to find out whether there is any grammatical
mistakes/error to it. The error, if any

will be in one part of the sentence. Mark the part with the error as you answer. If there is no error, mark No
error as your answer. (Ignore the errors of punctuation, if any)

26. More than scoring points for/choosing the colour, the actress creating a fluffed/with the
mini cape that she wore/to match her dress.

(1) More than scoring points for


(2) Choosing the colour, the actress creating a fluffed
(3) With the mini cape that she wore
(4) To match her dress
(5) No error

27. Winter is/the best/season to/explored the outdoors.

(1) Winter is
(2) the best
(3) Season to
(4) explored the outdoors
(5) No error

Click Here to Buy Printed Study Material for IBPS, SBI Bank Exams
http://www.bankexamportal.com/study-kit Page 31
Join Online Coaching for IBPS, SBI Exams
http://www.bankexamportal.com/elearning

28. The technology senses / how a handshakes / and makes instant adjustments / to stay
balanced.

(1) The technology senses


(2) How a handshakes
(3) and makes instant adjustments
(4) to stay balanced
(5) No error

29. With the water project moving / a step closer to feasibility, / city residents can soon /
expect for sufficient water supply.

(1) with the water project moving


(2) a step closer to feasibility
(3) city residents can soon
(4) expect for sufficient water supply
(5) No error

30. We are in the final stages/ of resolve the issue / which has been pending / for over two
years.

(1) We are in the final stage


(2) of resolve the issue
(3) which has been pending
(4) for over two years
(5) No error

Directions (Q. Nos. 31-40): In the given passage, there are blanks, each of which has been numbered.
Against each, fieve words are suggested, one of which fits the blank appropriately. Find out the appropriate
word in each case.

The emergence of a cloud based banking will affect banks big and small. Banks are expected to spend
almost $ 180 billion on IT this year. At present, cloud based services make up a (31) fraction of this amount
but some estimates (32) by financial services firms on the cloud will (33) $ 26 billion in 2015. This increase
should (34) barriers to entry for newcomers which can (35) modern IT infrastructure at monthly fees of less
than $ 10000 (36) having to invest tens of millions of dollars upfront (37) build their own secure data entries
and it should (38) enable big banks to become much more cost of (39). Small firms without traditional
computer systems to maintain are the fastest movers. (40) can type documents, run spreadsheets and read e-
mails in the cloud. Keeping track of clients, payments and loans can be done on a cloud computing platform
using a specially banking software.

31.

(1) largely
(2) tiny
(3) bit
(4) part
(5) less

Click Here to Buy Printed Study Material for IBPS, SBI Bank Exams
http://www.bankexamportal.com/study-kit Page 32
Join Online Coaching for IBPS, SBI Exams
http://www.bankexamportal.com/elearning

32.

(1) paying
(2) offering
(3) buying
(4) purchase
(5) spending

33.

(1) sum
(2) come
(3) account
(4) costs
(5) total

34.

(1) fall
(2) dropped
(3) lower (4) sank
(5) maintain

35.

(1) rent
(2) sold
(3) hired
(4) leased
(5) used

36.

(1) other
(2) rather
(3) more
(4) further
(5) compared

37.

(1) that
(2) for
(3) to
(4) try
(5) on

38.

Click Here to Buy Printed Study Material for IBPS, SBI Bank Exams
http://www.bankexamportal.com/study-kit Page 33
Join Online Coaching for IBPS, SBI Exams
http://www.bankexamportal.com/elearning

(1) both
(2) include
(3) always
(4) beside
(5) also

39.

(1) effect
(2) efficient
(3) price
(4) ceiling
(5) subsidy

40.

(1) Employees
(2) Who
(3) How
(4) Worker
(5) Sub-ordinates

Click Here to Buy Printed Study Material for IBPS, SBI Bank Exams
http://www.bankexamportal.com/study-kit Page 34
Printed Study Material, for Bank, SBI, IBPS - PO, Clerk Exams

IBPS Specialist Officer Exam Study Material - 100% Syllabus Covered


http://bankexamportal.com/study-kit/ibps-specialist-officer

Printed Study Material for SBI PO Exam 2015 - 100% Syllabus


Covered
http://bankexamportal.com/study-kit/sbi-po

Printed Study Materials for IBPS-PO (Probationary Officer) Exam


http://bankexamportal.com/study-kit/ibps-po

IBPS Clerk Study Kit


http://bankexamportal.com/study-kit/ibps-clerk

Essay Writing Skills Improvement Programme (EWSIP)


http://iasexamportal.com/civilservices/study-kit/essay-mains

For More Information Click Given below link:


http://bankexamportal.com/study-kit
Join Online Coaching for IBPS, SBI Exams
http://www.bankexamportal.com/elearning

Subject: Reasoning
Directions (Q. Nos. 1-5): Study the following arrangement carefully and answer the given questions.

W2XT3*ZbU4OP9 $QG D5#WEJ6&8K@7+

1. If all the symbols are dropped from the arrangement, then which will be the eleventh
element from the right end of the given arrangement?

(1) 9
(2) G
(3) D
(4) 5
(5) P

2. How many such symbols are there in the given arrangment each of which is immediately
followed by a letter and also immediately preceded by a number?

(1) More than three


(2) Two
(3) Three
(4) None
(5) One

3. Which of the following is sixth to the left of the fourteenth from the left end of the given
arrangement?

(1) b
(2) *
(3) U
(4) Z
(5) 4

4. What should come in place of the question mark(?) in the following series based on the
given arrangement?

WXT 3ZU 4PQ G5J ?

(1) W2b
(2) 6%8
(3) 6%+
(4) WXZ
(5) &8+

5. Four of the following five are alike in a certain way based on their positions in the given
arrangement and so form a group. Which is the one that does not belong to the group?

Click Here to Buy Printed Study Material for IBPS, SBI Bank Exams
http://www.bankexamportal.com/study-kit Page 35
Join Online Coaching for IBPS, SBI Exams
http://www.bankexamportal.com/elearning

(1) TW3
(2) @%7
(3) G9D
(4) b3U
(5) $9Q

Directions (Q. Nos. 6-10): The questions is based on the five three-digit numbers given below.

476 538 289 814 753

6. Which of the following is the second digit of the three digit number obtained by
subtracting the lowest number from the highest number?

(1) 2
(2) 3
(3) 4
(4) 6
(5) 7

7. If 1 is added to the first digit in each number and then the position of the first and the
third digits are interchanged. Which of the following will be the third digit of the second
highest number thus formed?

(1) 6
(2) 4
(3) 2
(4) 8
(5) 7

8. In which of these digits, the sum of all the three digits is an even number?

(1) 753
(2) 538
(3) 269
(4) 476
(5) 814

9. Which of the following is the sum of the second and third digits of the second lowest
number?

(1) 12
(2) 15
(3) 91
(4) 11
(5) 13

Click Here to Buy Printed Study Material for IBPS, SBI Bank Exams
http://www.bankexamportal.com/study-kit Page 36
Join Online Coaching for IBPS, SBI Exams
http://www.bankexamportal.com/elearning

10. If 1 is subtracted from third digit in the each number and the position of the first and
third digits are interchanged. Which of the following will be the first digit of the third
highest number thus formed?

(1) 3
(2) 5
(3) 7
(4) 8
(5) 9

Directions (Q. Nos. 11-15): In these questions, the symbol @, %, , $ and # are used with the following
meaning as illustrated below:

P @ Q means P is neither smaller than nor equal to Q.


P % Q means P is neither greater than nor equal to Q.
P Q means P is not greater than Q.
P $ Q means P is not smaller than Q.
P # Q means P is neither smaller nor greater than Q.

Assuming the given statements to be true, find which conclusion is definitely true.

11. Statements

H @ K, K % M, M D

Conclusions

I. H @ D II. K % D

(1) Either conclusion I or II is true.


(2) Only conclusion I is true.
(3) Neither conclusion I nor II is true.
(4) Only conclusion II is true.
(5) Both conclusions I and II are ture.

12. Statements

R % H, H T, T @ K

Conclusions

I. T R

II. K % H

(1) Either conclusion I or II is true.


(2) Only conclusion I is true.
(3) Neither conclusion I nor II is true.

Click Here to Buy Printed Study Material for IBPS, SBI Bank Exams
http://www.bankexamportal.com/study-kit Page 37
Join Online Coaching for IBPS, SBI Exams
http://www.bankexamportal.com/elearning

(4) Only conclusion II is true.


(5) Both conclusions I and II are ture.

13. Statements

R D, D $ M, M # J

Conclusions

I. J # D II. J % D

(1) Either conclusion I or II is true.


(2) Only conclusion I is true.
(3) Neither conclusion I nor II is true.
(4) Only conclusion II is true.
(5) Both conclusions I and II are ture.

14. Statements

W # D, Z B, B $ H

Conclusions

I. H # Z II. B % W

(1) Either conclusion I or II is true.


(2) Only conclusion I is true.
(3) Neither conclusion I nor II is true.
(4) Only conclusion II is true.
(5) Both conclusions I and II are ture.

15. Statements

F $ N, N @ D, D % B

Conclusions

I. F @ D II. B @ N

(1) Either conclusion I or II is true.


(2) Only conclusion I is true.
(3) Neither conclusion I nor II is true.
(4) Only conclusion II is true.
(5) Both conclusions I and II are ture.

Directions (Q. Nos. 16-20): In these questions, two statements followed by two conclusions numbered I and
II have been given. Decide which of the given conclusions logically follows the given statements
disregarding commonly known facts.

Click Here to Buy Printed Study Material for IBPS, SBI Bank Exams
http://www.bankexamportal.com/study-kit Page 38
Join Online Coaching for IBPS, SBI Exams
http://www.bankexamportal.com/elearning

16. Statements

All buses are cars.


All Scooters are buses.

Conclusions

I. No scooter is a bus.
II. All cars are buses.

(1) Either conclusion I or II is true.


(2) Only conclusion I is true.
(3) Neither conclusion I nor II is true.
(4) Only conclusion II is true.
(5) Both conclusions I and II are ture.

17. Statements

No auditorium is hall.
All theatres are halls.

Conclusions

I. No auditorium is a theatre.
II. All halls are theatres.

(1) Either conclusion I or II is true.


(2) Only conclusion I is true.
(3) Neither conclusion I nor II is true.
(4) Only conclusion II is true.
(5) Both conclusions I and II are ture.

18. Statements

Some drugs are medicines.


No medicine is a treatment.

Conclusions

I. All treatments being drugs is a possibility.


II. All drugs can never be treatments.

(1) Either conclusion I or II is true.


(2) Only conclusion I is true.
(3) Neither conclusion I nor II is true.
(4) Only conclusion II is true.
(5) Both conclusions I and II are ture.

Click Here to Buy Printed Study Material for IBPS, SBI Bank Exams
http://www.bankexamportal.com/study-kit Page 39
Join Online Coaching for IBPS, SBI Exams
http://www.bankexamportal.com/elearning

19. Statements

Some cameras are photos.


All cameras are snaps.

Conclusions

I. All snaps are photos.


II. Some snaps are photos.
(1) Either conclusion I or II is true.
(2) Only conclusion I is true.
(3) Neither conclusion I nor II is true.
(4) Only conclusion II is true.
(5) Both conclusions I and II are ture.

20. Statements

Some computers are tablets.


Some laptops are computers.

Conclusions

I. No laptop is a tablet.
II. All tablets being computers is a possibility.
(1) Either conclusion I or II is true.
(2) Only conclusion I is true.
(3) Neither conclusion I nor II is true.
(4) Only conclusion II is true.
(5) Both conclusions I and II are ture.

Directions (Q. Nos. 21-25): Study the following information carefully and answer the given questions:

Eight friends L, M, N, O, P, Q, R and S are sitting around a square table in such a way that four of them sit
at four corners of the square while four sit in the middle of each of the four sides. The ones who sit at the
four sides face outside while those who sit in the middle of the sides face the centre.

Only one person sits between L and Q. L sits at middle of one of the series.
O sits third to the right of Q.
Both R and S are immediate neighbours of L.
M sits second to left of N.
N is neither an immediate neighbour of Q nor S.

21. Four of the following five are alike in a certain way and so form a group. Which is the
one that does not belong to that group?

(1) L
(2) R
(3) S

Click Here to Buy Printed Study Material for IBPS, SBI Bank Exams
http://www.bankexamportal.com/study-kit Page 40
Join Online Coaching for IBPS, SBI Exams
http://www.bankexamportal.com/elearning

(4) O
(5) P

22. Who sits third to the left of Q?

(1) M
(2) S
(3) R
(4) N
(5) P

23. How many people sit between M and S when counted from the right hand side of S?

(1) One
(2) Three
(3) None
(4) Two
(5) Four

24. Which of the following is true regarding P?

(1) P sits exactly between M and N.


(2) O sits second to right of P.
(3) None of the given options is true.
(4) L sits immediate right of P.
(5) Q is an immediate neighbour of P.

25. What is the position of P with respect to S?

(1) Fourth to the left


(2) Second to the right
(3) Second to the left
(4) Third to the right
(5) Third to the left

Directions (Q. Nos. 26-30): Study the information and answer the given questions.
In a certain code language

work never goes waste is written as rb mk ni tj


never waste your time is written as ni ap sy rb
focus on your work is written as mk ap cn or
focus goes with time is written as sy tj cn ke

(All the codes are two letter codes only)

26. In the given code language, what does the code tj stand for?

Click Here to Buy Printed Study Material for IBPS, SBI Bank Exams
http://www.bankexamportal.com/study-kit Page 41
Join Online Coaching for IBPS, SBI Exams
http://www.bankexamportal.com/elearning

(1) never
(2) goes
(3) on
(4) work
(5) waste

27. What is the code for focus in the given code language?

(1) ap
(2) ni
(3) sy
(4) cn
(5) mk

28. What is the code for time in the given code language?

(1) tj
(2) sy
(3) ni
(4) cn
(5) rb

29. Which of the possibly means work on projects?

(1) sy cn tj
(2) gt cn or
(3) mk gt or
(4) mk cn gt
(5) mk or sy

30. In the given code language, what does the code rb stand for?

(1) Either never or waste


(2) goes
(3) your
(4) work
(5) time

Directions (Q. Nos. 31-35): Study the following information to answer the given questions.

Ten people are sitting in two parallel rows having five people each, in such a way that there is an equal
distance between adjacent person. In row 1-V, W, X, Y and Z are seated (but not necessarily in the same
order) and all of them are facing North. In row 2-F, G, H, I and J are seated (but not necessarily in the same
order) and all of the are facing South. Therefore, in the given seating arrangement, each member seated in a
row faces another member of the other row.

Y sits third to the left of W. The one who faces Y sits second to the right of F.
Only one person sits between F and I.

Click Here to Buy Printed Study Material for IBPS, SBI Bank Exams
http://www.bankexamportal.com/study-kit Page 42
Join Online Coaching for IBPS, SBI Exams
http://www.bankexamportal.com/elearning

H and J are immediate neighbours of each other. J does not sit at any of the extreme ends of the line.
The one faces G sits to the immediate right of Z.
X is not an immediate neighbour of Z.

31. Who amongst the following faces H?

(1) Y
(2) V
(3) Z
(4) W
(5) X

32. Who amongst the following sits to the immediate left of the person who sits exactly in
the middle of row-2?

(1) J
(2) H
(3) I
(4) G
(5) F

33. Four of the following five are alike in a certain way based on the given seating
arrangement and thus form a group. Which is the one that does not belong to that group?

(1) H
(2) I
(3) W
(4) Y
(5) X

34. Who amongst the following sits third to the right of the person who faces X?

(1) G
(2) F
(3) J
(4) I
(5) H

35. Which of the following is true regarding V?

(1) None of the given options is true.


(2) An immediate neighbour of V faces F.
(3) X is an immediate neighbour of V.
(4) W sits to immediate right of V.
(5) V faces I.

Click Here to Buy Printed Study Material for IBPS, SBI Bank Exams
http://www.bankexamportal.com/study-kit Page 43
Join Online Coaching for IBPS, SBI Exams
http://www.bankexamportal.com/elearning

Directions (Q. Nos. 36-40): These questions consist of a question and two statements numbered I and II
below it. You have to decide whether the data given in the statements are sufficient to answer the questions.
Read the statements and choose the most appropriate option.

36. In a straight line of eight people (all facing North), what is the position of R from the
left end?

I. Y stands fourth from the right end of the line. Only two people stand between Y and Z. R stands to the
immediate right of Z.
II. W stands fourth from the left end of the line. R is an immediate neighbour of W.

(1) The data in statement I alone are sufficient to answer the question while the data in statement II are not
sufficient to answer the question.
(2) The data in both statements I and II together are necessary to answer the question.
(3) The data even in both statements I and II together are not sufficient to answer the question.
(4) The data either in statement I or in statement II alone are sufficient to answer the question.
(5) The data in statement II alone are sufficient to answer the question while the data in statement I are not
sufficient to answer the question.

37. Among four friends W, X, Y and Z (each having different number of cookies), who has
the most number of cookies?

I. W has lesser number of cookies than Z. Y does not have the most number of cookies.
II. W has more cookies than Y. X does not have the most number of cookies.

(1) The data in statement I alone are sufficient to answer the question while the data in statement II are not
sufficient to answer the question.
(2) The data in both statements I and II together are necessary to answer the question.
(3) The data even in both statements I and II together are not sufficient to answer the question.
(4) The data either in statement I or in statement II alone are sufficient to answer the question.
(5) The data in statement II alone are sufficient to answer the question while the data in statement I are not
sufficient to answer the question.

38. In a code language my dear family is coded as 624. Which number stands for
dear?

I. In the same code language my small family is coded as 256.


II. In the same code language, dear family friend is coded as 647?

(1) The data in statement I alone are sufficient to answer the question while the data in statement II are not
sufficient to answer the question.
(2) The data in both statements I and II together are necessary to answer the question.
(3) The data even in both statements I and II together are not sufficient to answer the question.
(4) The data either in statement I or in statement II alone are sufficient to answer the question.
(5) The data in statement II alone are sufficient to answer the question while the data in statement I are not
sufficient to answer the question.

39. How is P related to Q?

Click Here to Buy Printed Study Material for IBPS, SBI Bank Exams
http://www.bankexamportal.com/study-kit Page 44
Join Online Coaching for IBPS, SBI Exams
http://www.bankexamportal.com/elearning

I. Q is the mother of T. M is the only sibling of T. H is the daughter of M and P.


II. M is married to P. T is the brother of M. Q is the brother of T.

(1) The data in statement I alone are sufficient to answer the question while the data in statement II are not
sufficient to answer the question.
(2) The data in both statements I and II together are necessary to answer the question.
(3) The data even in both statements I and II together are not sufficient to answer the question.
(4) The data either in statement I or in statement II alone are sufficient to answer the question.
(5) The data in statement II alone are sufficient to answer the question while the data in statement I are not
sufficient to answer the question.

40. How many students attended the cultural fair of the college?

I. The number of students attending the cultural fair was twice the number of female students.
II. The number of female students attending the cultural fair was 25 more than that in the previous year.

(1) The data in statement I alone are sufficient to answer the question while the data in statement II are not
sufficient to answer the question.
(2) The data in both statements I and II together are necessary to answer the question.
(3) The data even in both statements I and II together are not sufficient to answer the question.
(4) The data either in statement I or in statement II alone are sufficient to answer the question.
(5) The data in statement II alone are sufficient to answer the question while the data in statement I are not
sufficient to answer the question.

Click Here to Buy Printed Study Material for IBPS, SBI Bank Exams
http://www.bankexamportal.com/study-kit Page 45
Join Online Coaching for IBPS, SBI Exams
http://www.bankexamportal.com/elearning

IBPS Clerk Online CWE Previous Year Exam Paper - 2013

Subject: Numerical Ability


1. The ratio of monthly salaries of two persons. A and B is 8:7. If the salary of A is
increased by 20% and that of B by 11% the new ratio becomes 96: 77. What is As salary?

(1) Rs.800
(2) Rs.700
(3) Rs.750
(4) Cannot be determined
(5) None of these

2. Simple interest on a certain sum at 7 p.c. p.a for four years is Rs. 3584. What will be the
compound interest on the same principal at 4 p.c. p.a. in two years?

(1) Rs. 1054.48


(2) Rs. 1044.48
(3) Rs. 1044.84
(4) Rs. 1064.84
(5) None of these

3. The cost of pure milk is Rs. 16 per litre. On adding water the mixture is sold at Rs.
15/litre. In this way the milkman earns 25% profit. What is the ratio of milk and water in
the mixture?

(1) 25 : 7
(2) 7 : 25
(3) 15 : 1
(4) 1 : 15
(5) None of these

4. Four persons - M. N, O and P-distributed a sum of Rs. 44352 among themselves. M got
th of total amount. N got th part of the remaining amount. Thereafter, the remaining
amount was divided between O and P in teh ratio 3 : 2. The amount received by P is

(1) Rs. 1648


(2) Rs. 1848
(3) Rs. 1884
(4) Rs. 1684
(5) Rs.1448

5. Three persons A. B and C start a business with Rs. 12800. Rs. 16800 and Rs. 9600
respectively. At the end of the year. B received Rs. 13125 as share in total profit. What is
the share of Mr. C in the profit?

Click Here to Buy Printed Study Material for IBPS, SBI Bank Exams
http://www.bankexamportal.com/study-kit Page 46
Join Online Coaching for IBPS, SBI Exams
http://www.bankexamportal.com/elearning

(1) Rs.7850
(2) Rs.7550
(3) Rs.7500
(4) Rs.8500
(5) None of these

6. One-third of a diagonal of a square is units. What is the measure or the side or the
square?

(1) 6 units
(2) 3 units
(3) 18 units
(4) 9 units
(5) None of these

7. 56 men can do a job in 14 days. How many additional men are required to do the same
job in 8 days?

(1) 42
(2) 24
(3) 52
(4) 25
(5) None of these

8. A shopkeeper has goods of worth Rs. 6000. He sold hall or the goods at a gain of 12%.
At what profit percent should he sell the remaining half of the stock so that he gets 18%
profit on the whole?

(1) 25%
(2) 24%
(3) 18%
(4) 21%
(5) None of these

9. A 210 metre long train crosses a man running at 9 kmph in opposite direction in 6
seconds. Find the speed or the train.

(1) 98 kmph
(2) 97 kmph
(3) 107 kmph
(4) 117 kmph
(5) None of these

10. Average score of a cricketer in 13 matches is 42 runs. If the average score in the first
five matches be 54 runs what is the average score in the last eight matches?

Click Here to Buy Printed Study Material for IBPS, SBI Bank Exams
http://www.bankexamportal.com/study-kit Page 47
Join Online Coaching for IBPS, SBI Exams
http://www.bankexamportal.com/elearning

(1) 36.5
(2) 34.5
(3) 35.4
(4) 38.5
(5) None of these

11. The perimeter- of a rectangle whose length is 6 metre more than its breadth is 84
metre. What is the area of the triangle whose base is equal to the diagonal of the rectangle
and height is equal to the length of the rectangle?

(1) 360 sq. metre


(2) 360 sq. metre
(3) 380 metre
(4) 400 sq. metre
(5) None of these

12. Due to decrease of 10% in the price of tea per kg. a consumer buys 250gm of tea more
than before for Rs. 270. What is the Original price of tea ?

(1) Rs. 130/kg


(2) Rs. 120/kg
(3) Rs. 115/kg
(4) Rs. 140/kg
(5) None of these

Directions (93-107): What will come in place of the question mark (?) in the following questions?

13.(34.5 14 42) 2.8 = ?

(1) 7445
(2) 7425
(3) 7245
(4) 7435
(5) None of these

14. -676.76 + 1237.87 + 897.34 - ? = 1294.25

(1) 168.2
(2) 164.2
(3) 154.2
(4) 164.8
(5) None of these

15. 3/8 of {4624 (564 - 428)} = ?

(1) 11.75
(2) 12.57
(3) 21.75

Click Here to Buy Printed Study Material for IBPS, SBI Bank Exams
http://www.bankexamportal.com/study-kit Page 48
Join Online Coaching for IBPS, SBI Exams
http://www.bankexamportal.com/elearning

(4) 12.75
(5) None of these

16.

(1) 18
(2) 17
(3) 28
(4) 19
(5) None of these

17. (216) 4 (36) 4 x 6 5 = (6)?

(1) 8
(2) 9
(3) 7
(4) 10
(5) 11

18. 456 24 38 - 958 + 364 = ?

(1) 228
(2) 124
(3) 128
(4) 138
(5) 145

19.

Click Here to Buy Printed Study Material for IBPS, SBI Bank Exams
http://www.bankexamportal.com/study-kit Page 49
Join Online Coaching for IBPS, SBI Exams
http://www.bankexamportal.com/elearning

20. (973 14) 5 11 = ?

(1) 152.2
(2) 152.9
(3) 159.2
(4) 195.2
(5) None of these

21.

(1) 196
(2) 125
(3) 169
(4) 225
(5) 81

22. (43) 2 + 841 = (?) 2 + 1465

(1) 15
(2) 65
(3) 45
(4) 35
(5) 25

23.

24.

Click Here to Buy Printed Study Material for IBPS, SBI Bank Exams
http://www.bankexamportal.com/study-kit Page 50
Join Online Coaching for IBPS, SBI Exams
http://www.bankexamportal.com/elearning

(1) 5.75
(2) 6.75
(3) 7.75
(4) 6.25
(5) 6.50

25. (1097.63 + 2197.36 - 2607.24) 3.5 = ?

(1) 196.5
(2) 186.5
(3) 196.75
(4) 200.5
(5) 136.5

26.

(1) 121
(2) 11
(3) 9
(4) 12
(5) 17

27.

(1) 16
(2) 12
(3) 14
(4) 18
(5) 17

Directions (28-32): What will come in place of the question mark (7) In the following number series 7

28. 960 839 758 709 ? 675

(1) 698
(2) 694
(3) 684

Click Here to Buy Printed Study Material for IBPS, SBI Bank Exams
http://www.bankexamportal.com/study-kit Page 51
Join Online Coaching for IBPS, SBI Exams
http://www.bankexamportal.com/elearning

(4) 648
(5) 680

29. 3 ? 14 55 274 1643

(1) 5
(2) 6
(3) 7
(4) 8
(5) None of these

30. 36 38.8 42.8 ? 54.4 62

(1) 46
(2) 48
(3) 45
(4) 48.2
(5) 49.4

31. 37 ? 103 169 257 367

(1) 49
(2) 46
(3) 56
(4) 59
(5) 69

32. 4 6 12 ? 90 315

(1) 25
(2) 27
(3) 30
(4) 45
(5) None of these

Directions (33-37): In the following table the number of pages printed by 6 printers in 5 weeks has
been given. Read the following table carefully and answer the questions.

Click Here to Buy Printed Study Material for IBPS, SBI Bank Exams
http://www.bankexamportal.com/study-kit Page 52
Join Online Coaching for IBPS, SBI Exams
http://www.bankexamportal.com/elearning

33. What is the average number of pages printed by all printers in the first week 7

(1) 586.5
(2) 540
(3) 545.4
(4) 546.5
(5) 548

34. What is the average number of pages printed by printer C taking all the weeks
together ?

(1) 529
(2) 519
(3) 591
(4) 592
(5) 539

35. In which week was the number of printed pages maximum?

(1) First
(2) Second
(3) Third
(4) Fourth
(5) Fifth

36. The difference between the pages printed by printers A and E in the second week is

(1) 46
(2) 42
(3) 62
(4) 52
(5) None of these

Click Here to Buy Printed Study Material for IBPS, SBI Bank Exams
http://www.bankexamportal.com/study-kit Page 53
Join Online Coaching for IBPS, SBI Exams
http://www.bankexamportal.com/elearning

37. The difference between the total pages printed by printers B and C (all weeks taken
together) is

(1) 65
(2) 70
(3) 66
(4) 77
(5) None of these

38. Pipe A can fill a tank in 8 Hours while another pipe B can fill it in 16 hours. A third
pipe C can empty the full tank in 32 hours. All three pipes are opened simultaneously. In
what time will an enmity tank be filled?

(1) 5.5 hours


(2) 6 hours
(3) 6.4 hours
(4) 7 hours
(5) 7.2 hours

39. A and B are two numbers. Six times square of B is 540 more than square of A. The
ratio of A and B is 3 : 2. Find the number B?

(1) 12
(2) 18
(3) 14
(4) 21
(5) None of these

40. Eight years ago, the age of Vishal was four times that of Shekhars. After 8 years,
VishaJs age will be twice of Shekhars age. Vishals present age is

(1) 24years
(2) 28years
(3) 30years
(4) 32years
(5) None of these

Click Here to Buy Printed Study Material for IBPS, SBI Bank Exams
http://www.bankexamportal.com/study-kit Page 54
Study Kits for Staff Selection Commission Exams
Study Kit for SSC CGL EXAM (Tier-I)
http://sscportal.in/community/study-kit/cgl

Study Kit for SSC Combined Graduate Level Examination (Tier - II)
http://sscportal.in/community/study-kit/cgl-tier-2

Study Kit for Combined Higher Secondary Level (10+2) Examination


http://sscportal.in/community/study-kit/chsle

( :-1)
http://sscportal.in/community/study-kit/cgl/tier-1-hindi

Study Kit For SSC MTS Examination


http://sscportal.in/community/study-kit/mts

Study Kit For Prasar Bharti Examination


http://sscportal.in/community/study-kit/prasar-bharti-exam

Study Kit for Central Armed Police Forces (CAPFs) Examination


http://sscportal.in/community/study-kit/capfs

Study Kit For Sub Inspector in Delhi Police & CAPFs, CISF (Paper-1)
http://sscportal.in/community/study-kit/si-delhi-police-capf

Study Kit of English Language & Comprehension For All SSC Exam
http://sscportal.in/community/study-kit/english-language-comprehension-for-all-
ssc-exam

Study Kit For Postal Assistant Examination


http://sscportal.in/community/study-kit/postal-assistant

For More Information Click Given below link:


http://sscportal.in/community/study-kit
Join Online Coaching for IBPS, SBI Exams
http://www.bankexamportal.com/elearning

Subject: General Awareness


1. Derivatives Contract which gives the buyer/holder of the contract the right (but not the
obligation) to buy/sell the underlying asset at a predetermined price within or at end of a
specified period is known as

(1) Futures Contract


(2) Option Contract
(3) Index Futures contract
(4) Mini Derivative Contract
(5) None of these

2. The first Yash Chopra Memorial Award has been given to

(1) A. R. Rahman
(2) Lata Mangeshkar
(3) Amitabh Bachchan
(4) Rishi Kapoor
(5) Gulzar

3. CASA ratio of bank indicate the

(1) Banks deposits


(2) Banks Non Performing Assets
(3) Banks liabilities
(4) Banks base rate
(5) Banks total assets

4. The process by which LIC holder can transfer all rights, title and interest under a policy
contract to any third person is known as

(1) Subrogation
(2) Escrow
(3) Mortgage
(4) Assignment
(5) Nomination

5. In order to promote no frills accounts as part of basic banking, the Reserve Bank of
India, in August 2012, advised banks to rechristen it as

(1) Basic Saving Account


(2) Basic Deposit Account
(3) Basic Account
(4) Basic Savings Bank Deposit Account
(5) None of these

Click Here to Buy Printed Study Material for IBPS, SBI Bank Exams
http://www.bankexamportal.com/study-kit Page 55
Join Online Coaching for IBPS, SBI Exams
http://www.bankexamportal.com/elearning

6. The UIDAI has set a target of enrolment for how many people for Aadhaar Numbers till
end of2014?

(1) 25 crore people


(2) 48 crore people
(3) 50 crore people
(4) 60 crore people
(5) 72 crore people

7. Equity share is also known as

(1) Shareholder equity


(2) Stockholders equity
(3) Share capital
(4) Net worth
(5) All of these

8. Fixed Deposit is also referred to as

(1) Time Deposit


(2) Recurring Deposit
(3) Demand Deposit
(4) Static Deposit
(5) All of the above

9. W ith which sports is Deepak Lathore associated?

(1) Basketball
(2) Athletics
(3) Weightlifting
(4) Squash
(5) Wrestling

10. What is the minimum age required to become a member of the Rajya Sabha?

(1) 21 years
(2) 25 years
(3) 30 years
(4) 32 years
(5) 35years

11. Microcredit concentrates on

(1) Small loans


(2) Women
(3) Rural areas
(4) Poor people
(5) All of these

Click Here to Buy Printed Study Material for IBPS, SBI Bank Exams
http://www.bankexamportal.com/study-kit Page 56
Join Online Coaching for IBPS, SBI Exams
http://www.bankexamportal.com/elearning

12. Which of the following is a receipt, declaring ownership of shares of a foreign


company, which can be listed In India and traded in rupees?

(1) ADR
( 2) GDR
(3) IDR
(4) EDR
(5) None of these

13. NABARD is responsible for regulating and supervising

(1) Securities market in India


(2) Bank rates of scheduled commercial banks in India
(3) Cooperatives
(4) Regional Rural Banks
(5) Only 3 and 4

14. Arrangement of bank selling insurance product acting as agent of respective companies
is called

(1) Bancassurance
(2) Outsourcing
(3) Delegation
(4) Proxy Insurance
(5) Shadow Insurance

15. BCSBI aims to plan, evolve, prepare, develop, promote and publish comprehensive
Codes and Standards for banks, for providing for fair treatment to their customers. Which
of the following codes has been evolved by it?

(1) Code of Banks Commitment to Customers


(2) Code of Banks Commitment to Micro and Small Enterprises
(3) Code of Banks Commitment to Financial Inclusion
(4) Both 1 and 2
(5) Both 2 and 3

16. World Food Day is observed on

(1) 5 September
(2) 21 September
(3) 16 October
(4) 19 November
(5) 3 December

17. The Adhaar-enabled payment systems (AEPS) is a bank-led model that facilitates
banking facilities by allowing transactions at Point of Sale through the business

Click Here to Buy Printed Study Material for IBPS, SBI Bank Exams
http://www.bankexamportal.com/study-kit Page 57
Join Online Coaching for IBPS, SBI Exams
http://www.bankexamportal.com/elearning

correspondent (BC) using the Aadhaar authentication number. Adhaar-enabled basic


types of banking transactions do not include

(1) Balance enquiry


(2) Cash withdrawal
(3) Online payment
(4) Cash deposit
(5) Aadhaar to Aadhaar funds transfer

18. Which of the following is Asias largest residential university?

(1) Nalanda University


(2) University of Delhi
(3) Banaras Hindu University
(4) Jamia Milia Islamia
(5) Jawaharlal Nehru University

19. Which of the following banks/ ministries/bodies is the Lender of last resort in India?

(1) World Bank


(2) Reserve Bank of India
(3) Union Ministry of Finance
(4) Planning Commission
(5) International Monetary Fund

20. For expanding access to banking services, the Reserve Bank of India has advised
banks to open branches with minimum infrastructure support of 8 to 10 BC units at a.
reasonable 3-4 km. Such branches are known as

(1) White Label ATMs


(2) Ultra Small Branches
(3) Banking Kiosks
(4) CBS Terminals
(5) ICT Hubs

21. According to the Income Tax Act of 1961, the age of Super Senior Citizens should be

(1) 62years
(2) 70years
(3) 80years
(4) 88years
(5) 95years

22. Who has authored the book, titled The Lowland?

(1) Khushwant Singh


(2) Vikram Seth
(3) Chetan Bhagat

Click Here to Buy Printed Study Material for IBPS, SBI Bank Exams
http://www.bankexamportal.com/study-kit Page 58
Join Online Coaching for IBPS, SBI Exams
http://www.bankexamportal.com/elearning

(4) Jhumpa Lahiri


(5) Jeet Thayil

23. Who, at present, is the Chief Election Commissioner of India?

3(1) V.S. Sampath


(2) S.Y. Qureshi
(3) Navin Chawla
(4) N. Gopalaswamy
(5) B.B. Tandon

24. When Criminals use an innocent persons details to open or use an account to carry out
financial transactions, it is known a

(1) Money Laundering


(2) Phishing
(3) Identity Theft
(4) Real Time gross Settlement
(5) Insinuation

25. Where is the headquarters of the United Nations Organization located?

(1) Rome
(2) Brussels
(3) Geneva
(4) New York
(5) Paris

26. As per the announcements made in Union Budget 2013-14, a person taking a loan for
his first home from a bank or a housing finance corporation (HFC) up to Rs 25 lakh from
April 1, 2013 to March 31, 2014, will be entitled to an additional deduction of interest of

(1) Rs. 75,000


(2) Rs. 1 lakh
(3) Rs. 1.5 lakh
(4) Rs. 2 lakhs
(5) Rs. 2.6 lakh

27. Where was the 8lh National Conference on Krishi Vlgyan Kendra 20 13 held?

(1) Chandigarh
(2) Bangalore
(3) Kolkata
(4) Patna
(5) Jodhpur

28. The Reserve Bank of India has made it mandatory for banks to lend at least 40per cent
of their credit to select sectors such as agriculture, Micro and Small Enterprises other
Click Here to Buy Printed Study Material for IBPS, SBI Bank Exams
http://www.bankexamportal.com/study-kit Page 59
Join Online Coaching for IBPS, SBI Exams
http://www.bankexamportal.com/elearning

weaker sections, etc. Which banks have to lend at least 40 per cent of their total net credit
to priority sectors?

(1) Scheduled Commercial Banks


(2) Foreign Banks
(3) NABARD
(4) IRDA
(5) All of the above

29. What is the term for a bank without any branch network that offers its services
remotely?

(1) Virtual Bank


(2) Direct Bank
(3) Lending Institution
(4) Indirect Bank
(5) Online Bank

30. With reference to Indian Banking sector which of the following is the most appropriate
expanded form of EMI?

(1) Estimated Monthly Installment


(2) Extended Matching Items
(3) Equated Monthly Installment
(4) Enterprise Manufacturing Intelligence
(5) Easy Monthly Installment

31. Which term is used for banking in which banking institutions execute transactions
directly with consumers rather than corporations or other banks?.

(1) Merchant Banking


(2) Collateral Banking
(3) Retail Banking
(4) Bulk Banking
(5) Direct Banking

32. As per the 2011 Census which state of India has the lowest density?

(1) Mizoram
(2) Jammu and Kashmir
(3) Arunachal Pradesh
(4) Tripura
(5) Nagaland

33. With which state of India is the Cheraw Dance associated?

(1) Assam
(2) Tripura

Click Here to Buy Printed Study Material for IBPS, SBI Bank Exams
http://www.bankexamportal.com/study-kit Page 60
Join Online Coaching for IBPS, SBI Exams
http://www.bankexamportal.com/elearning

(3) Meghalaya
(4) Mizoram
(5) Rajasthan

34. The Direct Benefit Transfer Scheme is targeted at the people

(1) Affected by Uttrakhand landslides


(2) Affected by communal riots
(3) Living below the poverty line
(4) Suffering from acute starvation
(5) Belonging to Other Backward Classes

35. Which among the following, is not identified as a minority community in India?

(1) Buddhists
(2) Jains
(3) Muslims
(4) Zoroastrians
(5) Sikhs

36. Which of the following bodies recently expressed concern over unregistered investment
advisors spread across the country and brought out the Investment Advisers Regulations
2013?

(1) Reserve Bank of India


(2) IRDA
(3) SEBI
(4) NABARD
(5) CII

37. EFTPOS (electronic funds transfer at point of sale) is based on

(1) SMS Alerts


(2) Debit Cards
(3) Credit Cards
(4) Both 1 and 2
(5) Both 2 and 3

38. In a circular issued by the Reserve Bank of India on 9 April 2010. Banks were advised
to switch over to which system for calculation of their lending rates with effect from July
1.2010?

(1) Prime Lending Rate System


(2) Base Rate System
(3) Marginal Liquidity Facility System
(4) Statutory Liquidity Ratio system
(5) Benchmark Rate System

Click Here to Buy Printed Study Material for IBPS, SBI Bank Exams
http://www.bankexamportal.com/study-kit Page 61
Join Online Coaching for IBPS, SBI Exams
http://www.bankexamportal.com/elearning

39. Which Bollywood actress in October 2013 was honoured in the House of Commons for
her contribution to the global entertainment?

(1) Aishwarya Rat Bachchan


(2) Sharmila Tagore
(3) Kareena Kapoor
(4) Waheeda Rehman
(5) Shabana Azmi

40. Part of companys earning or profit which is paid out to share holders is known as

(1) Premium
(2) Dividend
(3) Bonus
(4) Sum Assured
(5) Return

Click Here to Buy Printed Study Material for IBPS, SBI Bank Exams
http://www.bankexamportal.com/study-kit Page 62
Join Online Coaching for IBPS, SBI Exams
http://www.bankexamportal.com/elearning

Subject: Computer Knowledge


1. The advantage of a LAN is

(1) Sharing peripherals


(2) Backing up your data
(3) Saving all your data
(4) Accessing the Web
(5) Automatic printing of data

2. Of the 5 words listed below which one is the odd one out?

(1) Applications
(2) Peripherals
(3) Programs
(4) Software
(5) Operating System

3. What type of device is a computer printer?

(1) Input
(2) Input / Output
(3) Software
(4) Storage
(5) Output

4. What is the biggest number you can get with 8 bits?

(1) 256
(2) 128
(3) 1000
(4) 255
(5) 1024

5. What type of device is a digital camera?

(1) Input
(2) Output
(3) Software
(4) Storage
(5) Input / Output

6. Which of the following domains is used by for-profit businesses?

(1) .com
(2) .edu
(3) .mil

Click Here to Buy Printed Study Material for IBPS, SBI Bank Exams
http://www.bankexamportal.com/study-kit Page 63
Join Online Coaching for IBPS, SBI Exams
http://www.bankexamportal.com/elearning

(4) .net
(5) .org

7. WAN stands for __

(1) Wired Area Network


(2) Wide Area Network
(3) Wide Array Net
(4) Wireless Area Network
(5) Wanted Area Network

8. What are lists of programs waiting to be run called?

(1) Shells
(2) The background
(3) Queues
(4) Page frames
(5) Lists

9. An example of peripheral equipment is the-

(1) Printer
(2) CPU
(3) spreadsheet
(4) microcomputer
(5) ALU

10. Which of the following commercial software products are examples of operating
system software and application software respectively?

(1) Microsoft Windows XP and Microsoft. Word


(2) Microsoft Office XP and Microsoft Windows XP
(3) MS DOS and Microsoft Windows XP
(4) UNIX and LINUX
(5) UNIX and Java

11. __ are attempts by individuals to obtain confidential information from you by


falsifying their identity.

(1) Phishing trips


(2) Computer viruses
(3) Spyware scams
(4) Viruses
(5) Phishing scams

12. Why is it unethical to share copyrighted files with your friends?

Click Here to Buy Printed Study Material for IBPS, SBI Bank Exams
http://www.bankexamportal.com/study-kit Page 64
Join Online Coaching for IBPS, SBI Exams
http://www.bankexamportal.com/elearning

(1) It is not unethical, because it is legal.


(2) It is unethical because the files are being given for free.
(3) Sharing copyrighted files without permission breaks copyright laws.
(4) It is not unethical because the files are being given for free.
(5) It. is not unethical - any- one can access a computer

13. Which of the following can be used to select the entire document?

(1) CTRL + A
(2) ALT + F5
(3) SHIFT + A
(4) CTRL + K
(6) CTRL + H

14. The system unit-

(1) coordinates input and output devices


(2) is the container that houses electronic components
(3) is a combination of hardware and software
(4) controls and manipulates data
(5) does the arithmetic operations

15. The simultaneous processing of two or more programs by multiple processors is __

(1) multiprogramming
(2) multitasking
(3) time-sharing
(4) multiprocessing
(5) None of these

16. The device which helps you to communicate with computer is called:

(1) Input device


(2) Output device
(3) Software device
(4) Storage device
(5) None of these

17. In order to avoid memorizing e-mail address you should use

(1) Browser
(2) Search engine
(3) List of Birth date
(4) Phonebook
(5) Address book

18. The devices which store information and that are used by computer for its functioning
are called:

Click Here to Buy Printed Study Material for IBPS, SBI Bank Exams
http://www.bankexamportal.com/study-kit Page 65
Join Online Coaching for IBPS, SBI Exams
http://www.bankexamportal.com/elearning

(1) Input devices


(2) Output devices
(3) Software devices
(4) Storage devices
(5) None of these

19. Which among the following is not such an operation which can be carried out on
objects in graphic programmed?

(1) Spell check


(2) Change size
(3) Move
(4) Delete
(5) None of these

20. A hardware device which converts data into meaningful information is called

(1) Protector
(2) Output device
(3) Input device
(4) Program
(5) Processor

21. Which process checks to ensure the components of the computer are operating and
connected properly?

(1) Booting
(2) Processing
(3) Saving
(4) Editing
(5) Starting

22. A means of capturing an image (drawing or photo) so that it can be stored on a


computer is

(1) Modem
(2) Software
(3) Scanner
(4) Keyboard
(5) Mouse

23. An error in a computer program

(1) Crash
(2) Power Failure
(3) Bug
(4) Virus
(5) Fatal error

Click Here to Buy Printed Study Material for IBPS, SBI Bank Exams
http://www.bankexamportal.com/study-kit Page 66
Join Online Coaching for IBPS, SBI Exams
http://www.bankexamportal.com/elearning

24. Access control based on a persons fingerprints is an example of

(1) biometric identification


(2) characteristic identification
(3) characteristic security
(4) fingerprint security
(5) logistics

25. The patterns of printed lines on most products are called

(1) prices
(2) striping
(3) scanners
(4) OCR
(5) barcodes

26. Most mail programs automatically complete the following two parts in an e-mail

(1) From: and Body:


(2) From: and Date:
(3) From: and To:
(4) From: and Subject:
(5) None of these

27. The computers capability of distinguishing spoken words is called

(1) voice analysts


(2) speech acknowledgement
(3) voice recognition
(4) speech interpretation
(5) vocalization

28. Which of the following is an advantage of mounting an application on the Web?

(1) the possibility of 24-hour access for users


(2) creating a system that can extend globally
(3) standardizing the design of the interface
(4) All of these
(5) None of these

29. The first page of a Web site is called the

(1) Homepage
(2) Index
(3) Java Script
(4) Book mark
(5) Intro Page

Click Here to Buy Printed Study Material for IBPS, SBI Bank Exams
http://www.bankexamportal.com/study-kit Page 67
Join Online Coaching for IBPS, SBI Exams
http://www.bankexamportal.com/elearning

30. ERP is an acronym for

(1) Enterprise Retirement Planning


(2) Enterprise Relationship Planning
(3) Enterprise Resource Planning
(4) Enterprise Reorder Planning
(5) Enterprise Retention Planning

31. Origin of internet can be tracked from

(1) ARPAnet
(2) Radio networks
(3) Satellite networks
(4) Indian army networks
(5) Air Force networks

32. To connect networks of similar protocols. __ are used

(1) Routers
(2) Bridges
(3) Gateways
(4) Dial-up routers
(5) None of these

33. Decimal equivalent of (11l1)2

(1) 11
(2) 10
(3) 1
(4) 15
(5) 13

34. _ is one reason for problems of data integrity

(1) Data availability constraints


(2) Data inconsistency
(3) Security constraints
(4) Unauthorized access of data
(5) Data redundancy

35. Dr. E.F. Codd represented____ rules that a database must obey if it has to be
considered truly relational

(1) 10
(2) 8
(3) 12
(4) 6
(5) 5

Click Here to Buy Printed Study Material for IBPS, SBI Bank Exams
http://www.bankexamportal.com/study-kit Page 68
Join Online Coaching for IBPS, SBI Exams
http://www.bankexamportal.com/elearning

36. > symbol in DOS commands is use to

(1) Compare two values


(2) Redirect input
(3) Redirect output
(4) Filter data
(5) None of these

37. System proposal is prepared in ____ phase of SDLC

(1) Conception
(2) Initiation
(3) Analysis
(4) Design
(5) construction

38. A data warehouse

(1) Contains numerous naming conventions and formats


(2) Is organized around important subject areas
(3) Contains only current data
(4) Can be updated by end users
(5) Explains some observed event or condition

39. Which of these is considered intelligent CASE tool

(1) Toolkit
(2) Methodology companion
(3) Workbench
(4) Upper CASE
(5) Lower CASE

40. Super computer developed by Indian scientists

(1) Param
(2) Super301
(3) Compaq Presario
(4) CrayYMP
(5) Blue Gene

Click Here to Buy Printed Study Material for IBPS, SBI Bank Exams
http://www.bankexamportal.com/study-kit Page 69
Join Online Coaching for IBPS, SBI Exams
http://www.bankexamportal.com/elearning

Subject: Reasoning
Directions (1-5): Study the following information carefully and answer the questions given below:

Seven persons A, B, C, D, E, F and G - are standing in a straight line facing north at equal distances
but not necessarily in the same order. Each of them is a different professional viz. Actor. Reporter,
Doctor, Engineer, Lawyer. Teacher and Painter but not necessarily in the same order.

G is standing at the fifth position to the left of C. Reporter is standing at the third position to the right
of G. F is standing at the fifth position to the right of A. E is standing second to the left of B. Engineer
is standing the second position to the left of D. Three persons are sitting between Engineer and
Painter. Doctor is to the immediate left of Engineer. Lawyer is to the immediate right of teacher.

1. How many persons are there to the left of Reporter?

(1) None
(2) One
(3) Two
(4) Three
(5) More than three

2. Which o f the following pairs of persons are sitting at the extreme ends?

(1) A and Actor


(2) Engineer and C
(3) Doctor and F
(4) F and Lawyer
(5) Teacher and Doctor

3. Who among the following is Sitting exactly in the middle of the row?

(1) Doctor
(2) F
(3) Lawyer
(4) B (5) Teacher

4. Who among the following is sitting second to the right of Teacher?

(1) Painter
(2) B
(3) A
(4) Actor
(5) Lawyer

5. Who among the following are the immediate neighbours of Painter?

(1) Actor and Teacher


(2) B and Lawyer

Click Here to Buy Printed Study Material for IBPS, SBI Bank Exams
http://www.bankexamportal.com/study-kit Page 70
Join Online Coaching for IBPS, SBI Exams
http://www.bankexamportal.com/elearning

(3) B and Engineer


(4) Reporter and C
(5) Doctor and Lawyer

Directions (6-7): Study the following information carefully and answer the questions given below:
Point A is 11 metre to the north of point B. Point C is 11 metre to the east of point B. Point D is 5
metre to the north of point C. Point E is 7 metre to the west of the point D. Point F is 9 metre to the
north of point E. Point G is 4 metre to the west of point F.

6. Point D is in which direction with respect to point F?

(1) North-East
(2) South-East
(3) South
(4) North-West
(5) East

7. Which of the following three points lie in a straight line?

(1) A, E and D
(2) F. E and C
(3) G. F and B
(4) G. A and B
(5) None of these

Directions (8-10): Study the following information carefully and answer the questions given below:
Among six persons K, L, M, N, O and P - each lives on a different floor of a building having six
floors numbered one to six (the ground floor is numbered 1. the floor above it is numbered 2 and so on
and the topmost floor is numbered 6).
L lives on an even numbered floor. L lives on a floor immediately below Ks floor and immediately
above Ms floor, P lives on a floor immediately above Ns floor. P lives on an even numbered floor. O
does not live on floor number 4.

8. Who amongst the following live on the floors exactly between K and P?

(1) O and L
(2) Land N
(3) Land M
(4) M and N
(5) M and O

9. On which floor does 0 live?

(1) 6th
(2) 2nd
(3) 3rd
(4) 5th
(5) Cannot be determined

Click Here to Buy Printed Study Material for IBPS, SBI Bank Exams
http://www.bankexamportal.com/study-kit Page 71
Join Online Coaching for IBPS, SBI Exams
http://www.bankexamportal.com/elearning

10. Who amongst the following does live on floor?

(1) O
(2) M
(3) N
(4) K
(5) Cannot be determined

Directions (11 - 15): Each of the questions below consists of a question and two statements numbered I and
II given below It. You have to decide whether the data provided in the statements are sufficient to answer
the question. Read both the statements and-

Give answer (1) if the data in statement I alone are sufficient to answer the question while the data in
statement II alone are not sufficient to answer the question.

Give answer (2) if the data in statement II alone are sufficient to answer the question while the data in
statement I alone are not sufficient to answer the question.

Give answer (3) if the data either in statement I alone or in statement II alone are sufficient to answer
the question.

Give answer (4) if the data given in both the statements I and II together are not sufficient to answer
the question and

Give answer (5) if the data in both the statements I and II together are necessary to answer the
question

11. Is D the mother of S?

I. L is the husband of D. L has only three children.


II. N is the brother of Sand P. P is the daughter of L.

12. How many students are there in the class?

I. There are more than 20 but less than 27 students in the class.
II. There are more than 24 but less than 31 students in the class. When the students are divided into groups
each group contains five students.

13. Among J, K, L M and N. each has different height. Who amongst them is the second
tallest?

I. N is taller than M and K. K is shorter than M.


II. L is taller than N. J is not the tallest.

14. Five persons A, B, C, D and E- are sitting in a circle facing the centre. Who is sitting
to the immediate left of D?

Click Here to Buy Printed Study Material for IBPS, SBI Bank Exams
http://www.bankexamportal.com/study-kit Page 72
Join Online Coaching for IBPS, SBI Exams
http://www.bankexamportal.com/elearning

I. C is sitting second to the left of A. Band 0 are immediate neighbours of each other.
II. D is sitting to the immediate left of B. E is not an immediate neighbour of D and B.

15. How is cost written in a code language?

I. tell me the cost is written as @ 0 # 9 and cost was very high is written as & 6 # 1 in that code
language.
II. some cost was discount is written as 1 8 7 # and some people like discount is written as 8 7 5 % in
that code language.

Directions (16-20): In these questions relationship between different elements is shown in the statements.
These statements are followed by two conclusions.

Mark answer if

(1) Only conclusion I follows.


(2) Only conclusion II follows.
(3) Either conclusion I or II follows.
(4) Neither conclusion I nor II follows.
(5) Both conclusions and II follow.

16. Statements

K>ITE:
O<R<K

Conclusions

I. R < E
II. O < T

17. Statements

B>A> S< I> C>L< Y

Conclusions

I. B > L
II. A > Y

18. Statements

C<L<O= U= DS> y

Conclusions

I. O > Y
II. C < D

Click Here to Buy Printed Study Material for IBPS, SBI Bank Exams
http://www.bankexamportal.com/study-kit Page 73
Join Online Coaching for IBPS, SBI Exams
http://www.bankexamportal.com/elearning

19. Statements

B>R>E>A>K
H > A >S

Conclusions

I. H > K
II. S < B

20. Statements

J=A; CKSA

Conclusions

I. C > J
II. C = J

Directions (21-25): In each of the questions/set of questions below are given two statements followed by
two conclusions numbered I and II. You have to assume everything in the statements to be true even if they
seem to be at variance from commonly known facts and then decide which of the two given conclusions
logically follows from the information given in the statement,

Give answer (1) if only conclusion I follows.


Give answer (2) if only conclusion II follows.
Give answer (3) if either conclusion I or conclusion II follows.
Give answer (4) if neither conclusion I nor conclusion II follows.
Give answer (5) if both conclusions I and II follow,

21. Statements

All alphabets are numbers.


Some alphabets are digits.

Conclusions

I. At least some digits are numbers.


II. No digit is a number.

22. Statements

Some squares are circles.


Some circles are rectangles

Conclusions

Click Here to Buy Printed Study Material for IBPS, SBI Bank Exams
http://www.bankexamportal.com/study-kit Page 74
Join Online Coaching for IBPS, SBI Exams
http://www.bankexamportal.com/elearning

I. At least some rectangles are squares.


II. No rectangle is a square.

23. Statements

No office is a palace.
All colleges are places.

Conclusions

I. All palaces are colleges.


II. No college is an office.

24. Statements

All mountains are rivers.


All rivers are lakes.

Conclusions

I. All mountains are lakes.


II. At least some lakes are rivers.

25. Statements

Some wins are losses.


All trophies are losses.

Conclusions

I. All trophies are wins.


II. All losses are trophies.

Directions (26-30): Study the following information carefully and answer the questions given below:
In a certain code language very large risk associated is written as nu ta ro gi

risk is very low is written as gi se nu mi


is that also associated is written as ta mi po Iu
inherent risk also damaging is written as fu nu di yu

26. Which of the following is the code of damaging?

(1) di
(2) yu
(3) nu
(4) either di or yu
(5) None of these

Click Here to Buy Printed Study Material for IBPS, SBI Bank Exams
http://www.bankexamportal.com/study-kit Page 75
Join Online Coaching for IBPS, SBI Exams
http://www.bankexamportal.com/elearning

27. Which of the following may represent risk is very large?

(1) gi mi nu ro
(2) nu gi ta se
(3) nu ro se yu
(4) ro gi di nu
(5) None of these

28. Which of the following is the code of associated?

(1) nu
(2) po
(3) ta
(4) fu
(5) gi

29. What would be the code for inherent large risk?

(1) yu ro nu
(2) di ro nu
(3) yu fu ro
(4) di ta se
(5) Cannot be determined

30. Which of the following may represent low risk associated industry ?

(1) ta nu gi ro
(2) ta hi nu se
(3) mi ta se fu
(4) di gi ta po
(5) po gi se di

Directions (31-35): Study the following information carefully answer the questions given below:

CEBACDBCDACEDE DCABADACEDUBANBD

31. How many such Ds are there in the above arrangement each of which is immediately
preceded by a consonant and also immediately followed by a vowel?

(1) None
(2) One
(3) Two
(4) Three
(5) More than three

32. How many Ds are there in the above arrangement?

Click Here to Buy Printed Study Material for IBPS, SBI Bank Exams
http://www.bankexamportal.com/study-kit Page 76
Join Online Coaching for IBPS, SBI Exams
http://www.bankexamportal.com/elearning

(1) Four
(2) Three
(3) Five
(4) Seven
(5) Six

33. If all the Bs are deleted from the above arrangement which of the following will be
eleventh from the left end?

(1) D
(2) C
(3) E
(4) A
(5) None of these

34. How many such As are there in the above arrangement each of which is immediately
preceded by B and also immediately followed by a consonant?

(1) None
(2) One
(3) Two
(4) Three
(5) More than three

35. Which of the following is ninth to the right of the 22nd from the right end of the above
arrangement?

(1) A
(2) C
(3) B
(4) D
(5) E

Directions (36-40): Study the following information carefully and answer the questions given below:

Eight people - S, T, U, V, W, X, Y and Z - are sitting around a circle but not necessarily in the same
order. Two of them S and Tare facing towards the centre while other people are facing towards the
outside. Y sits second to the left of W. S sits second to the left of Y. Only one person sits between Sand
Z. T sits to the immediate right of S. T is not an immediate neighbour of Y. V is not an immediate
neighbour of Y. Both the immediate neighbours of X face towards the outside.

36. What is the position of X with respect to S?

(1) Third to the right


(2) Fourth to the left
(3) Third to the left
(4) Fourth to the right
(5) Second to the right

Click Here to Buy Printed Study Material for IBPS, SBI Bank Exams
http://www.bankexamportal.com/study-kit Page 77
Join Online Coaching for IBPS, SBI Exams
http://www.bankexamportal.com/elearning

37. Who sits to the immediate left of Z?

(1) T
(2) W
(3) S
(4) X
(5) V

38. Which of the following pairs represents the immediate neighbours of W ?

(1) V and X
(2) V and Z
(3) X and Y
(4) Z and T
(5) S and U

39. What is the position of U with respect to W ?

(1) Second to the left


(2) Fifth to the right
(3) Sixth to the left
(4) Third to the right
(5) Fifth to the left

40. How many people sit between U and V?

(1) Two
(2) Four
(3) One
(4) Three
(5) Five

Click Here to Buy Printed Study Material for IBPS, SBI Bank Exams
http://www.bankexamportal.com/study-kit Page 78
SSC CGL Exams Online Coaching
(Starting at just Rs.100 per month) - English & Hindi
Medium

Online Crash Course for SSC CGL Tier-1 Exam


http://sscportal.in/community/courses/crash-course-for-cgl-tier-1

SSC CGL (Tier-1)


http://sscportal.in/community/courses/ssc-cgl-tier-1-hindi

For More Information Click Given below link:


http://sscportal.in/community/courses
Join Online Coaching for IBPS, SBI Exams
http://www.bankexamportal.com/elearning

Subject: English Language


Directions (1-5): In each of these questions, two sentences (I) and (II) are given. Each sentence has a blank
in it. Five words (1), (2), (3), (4) and (5) are suggested. Out of these. only one fits at both the places in the
context of each sentence. Number of that word is the answer.

1. I. Future generations will be left with a __ of pollution and destruction.

II. They each received a __ of $ 10000.

(1) legacy
(2) sum
(3) fortune
(4) amount
(5) inheritable

2. I. He was in bringing about an end to the Conflict.

II. Mr. Hart Prasad Chaurasia is an exponent of __ music.

(1) instrument
(2) instrumental
(3) keen
(4) instructor
(5) popular

3. I. Many __ men remarry and have second families.

II. When he was depressed. he felt utterly __ from reality.

(1) divided
(2) separate
(3) disturbed
(4) divorced
(5) destined

4. I. Most of our employees get __ abroad at some stage.

II. The aircraft and its crew were __ missing.

(1) announced
(2) arrested
(3) sent
(4) posted
(5) stationed

5. I. He admired his __ in the mirror.

Click Here to Buy Printed Study Material for IBPS, SBI Bank Exams
http://www.bankexamportal.com/study-kit Page 79
Join Online Coaching for IBPS, SBI Exams
http://www.bankexamportal.com/elearning

II. The article is an accurate __ of even is that day.

(1) reflection
(2) description
(3) imagery
(4) photograph
(5) mirage

Directions (6-10): In each question below, four words printed in bold type are given. These are numbered
(1), (2), (3) and (4). One of these words printed in bold may either be wrongly spelt or inappropriate in the
context of the sentence. Find out the word that is inappropriate or wrongly spelt, if any. The number of that
word is your answer. If all the words printed in bold are correctly spelt and appropriate in the context of the
sentence then mark (5) i.e. All Correct as your answer.

6. Only the golkeepers (1)/ reflexes (2) / stopped the ball (3)/ from going in. (4)/ All
Correct (5)

7. He has (1)/ refrain (2)/ from criticizing (3) / the government (4)/ in the public. All
Correct (5)

8. To save his own skull (1)/ he lied and blamed (2)/the accident (3)/ on his friend. (4)/ All
Correct (5)

9. He was badly (1)/ shaken (2)/ by the news (3)/ of her dead. (4)/ All Correct (5)

10. We lost (1)/ our way (2)/in the laybarinth (3)/ of streets. (4)/ All Correct (5)

Directions (11-15): Read each sentence to find out whether there is any grammatical error in it. The error if
any will be in one part of the sentence, the number of that part will be the answer. If there is no error, the
answer is (5). i.e. No error. (Ignore the errors of punctuation. if any.)

11. Lets wait and see (1)/ how the land (2)/ laid before (3)/ we do anything. (4)/ No error
(5)

12. In ancient Greece (1)/ a messenger ran from Marathon (2)/ to Athens with the news of
(3) / a victory of the persians. (4)/ No error (5)

13. Grammar information enables (1)/ students to map a (2)/ structure of a foreign (3)/
language onto their own. (4)/ No error (5)

14. Their wedding (1)/ turned out to be (2)/ quite (3)/ an occasion. (4)/ No error (5)

15. I may be stated(I) / the obvious but (2)/ without more-money the (3)/ project cannot
survive. (4)/ No error (5)

Click Here to Buy Printed Study Material for IBPS, SBI Bank Exams
http://www.bankexamportal.com/study-kit Page 80
Join Online Coaching for IBPS, SBI Exams
http://www.bankexamportal.com/elearning

Directions (16 - 20): Rearrange the following six sentences/ group of sentences (A), (B), (C), (D), (E)
and (F) in the proper sequence to form a meaningful paragraph; then answer the questions given
below them.

(A) The storm-given the name Hercules-closed major roads with snowdrifts.
(B) A fierce winter storm brought dangerous glacial temperature in New York on Friday.
(C) Weather experts said the wind chill temperature would plummet to -25 in New York.
(D) With more than 24 inches of snow falling in the city, a state of emergency was declared.
(E) More than 4000 international and domestic flights were also cancelled.
(F) Indeed storms are disastrous.

16. Which of the following should be the FIRST sentence after the rearrangement?

(1) E
(2) B
(3) D
(4) C
(5) F

17. Which of the following should be the FOURTH sentence after the rearrangement?

(1) D
(2) F
(3) B
(4) E
(5) C

18. Which of the following should be the FIFTH sentence after the rearrangement?

(1) E
(2) D
(3) B
(4) C
(5) F

19. Which of the following should be the SIXTH (LAST) sentence after the
rearrangement?

(1) A
(2) D
(3) F
(4) E
(5) C

20. Which of the following should be the SECOND sentence after the rearrangement?

(1) A
(2) D

Click Here to Buy Printed Study Material for IBPS, SBI Bank Exams
http://www.bankexamportal.com/study-kit Page 81
Join Online Coaching for IBPS, SBI Exams
http://www.bankexamportal.com/elearning

(3) F
(4) B
(5) C

Directions (21-30): Read the following passage carefully and answer the questions given below it. Certain
words have been printed in bold to help you locate them while answering some of the questions.

Internet banking is the term used for new age banking system. Internet banking is also called as
online banking and it is an outgrowth of PC banking. Internet banking uses the internet as the
delivery channel by which to conduct banking activity for example, transferring funds, paying bills,
viewing checking and savings account balances, paying mortgages and purchasing financial
instruments and certificates of deposits. Internet banking is a result of explored possibility to use
internet application in one of the various domains of commerce, It is difficult to infer whether the
internet tool has been applied for convenience of bankers or for the customers convenience. But
ultimately it contributes in increasing the efficiency of the banking operation as well providing more
convenience to customers. Without even interacting with the bankers, customers transact from one
corner of the country to another corner.

There are many advantages of online Banking. It is convenient, it isnt bound by operational timings,
there are no geographical barriers and the services can be offered at a minuscule cost. Electronic
banking has experienced explosive growth and has transformed traditional practices in banking.

Private Banks in India were the first to implement internet banking services in the banking industry.
Private Banks due to late entry into the industry understood that the establishing network in remote
comers of the country is a very difficult task, It was clear to them that the only way to stay connected
to the customers at any place and at any time is through internet applications. They took the internet
applications as a weapon of competitive advantage to comer the great monoliths like State Bank of
India. Indian Bank etc. Private Banks are pioneer in India to explore the versatility of internet
applications in delivering services to customers.

Several studies have attempted to assess the relative importance of B2B and B2C business domains.
There is wide difference in estimates of volume of business transacted over Internet and its
components under B2C and B2B. However. most studies agree that volume of transactions in B2B
domain far exceeds that in B2C. This is expected result. There is also a growing opinion that the
future of e-business lies in B2B domain as compared to B2C. This has several reasons like low
penetration of PCs to households low bandwidth availability etc.. in a large part of the world. The
success of B2C ventures depends to a large extent on the shopping habits of people in different parts
of the world. A survey sponsored jointly by Confederation of Indian Industries and Infrastructure
Leasing and Financial Services on e-commerce in India in 2010 the following observations. 62% of PC
owners and 75% of PC non-owners but who have access to Internet would not buy through the net as
they were not sure of the product offered. The same study estimated the size of B2B business in India
by the year 2011 to be varying between Rs. 1250 billion to Rs. 1500 billion. In a recent study done by
Arthur Anderson, it has been estimated that 84% of total e-business revenue is generated from B2B
segment and the growth prospects in this segment are substantial. It has estimated the revenues to be
anywhere between US $ 8.1 trillion to over US $ 21 trillion within the next three years (2014).

21. Which bank(s) is/are pioneer in India to explore the versatility of internet banking in
serving customers?

Click Here to Buy Printed Study Material for IBPS, SBI Bank Exams
http://www.bankexamportal.com/study-kit Page 82
Join Online Coaching for IBPS, SBI Exams
http://www.bankexamportal.com/elearning

(1) State Bank of India


(2) Indian Bank
(3) Public Sector Banks
(4) Private Banks
(5) None of these

22. Which of the following is not an advantage of online banking?

(1) It is convenient.
(2) It is bound by operational timings.
(3) The services can be offered at a minimum cost.
(4) There is no geographical barrier.
(5) None of these

23. What percentage of PC non- owners but who have access to Internet would not prefer
to buy through the net, as they are not sure of the product offered?

(1) 75%
(2) 62%
(3) 84%
(4) 76%
(5) None of these

24. Which type of activities are performed by internet banking?

(1) Paying bills


(2) Transferring funds
(3) Paying mortgages
(4) Purchasing financial instruments and certificates of deposits
(5) None of these

25. What estimate was made by confederation of Indian Industries regarding the size of
B2B business in India by the year 2011

(1) Between Rs. 250 billion to Rs. 500 billion


(2) Between Rs. 1250 billion to Rs. 1500 billion
(3) Between Rs. 850 billion to Rs. 1050 billion
(4) Between $ 8.7 trillion to $ 21 trillion
(5) None of these

Directions (26-28): Choose the word which is most similar in meaning to the word printed in bold as used
in the passage.

26. Versatility

(1) multi-utility
(2) vesicle
(3) dullness

Click Here to Buy Printed Study Material for IBPS, SBI Bank Exams
http://www.bankexamportal.com/study-kit Page 83
Join Online Coaching for IBPS, SBI Exams
http://www.bankexamportal.com/elearning

(4) necessity
(5) meanness

27. Monoliths

(1) large blocks of stone


(2) large organizations
(3) monopoly
(4) dwarfs
(5) niche

28. TRANSACT

(1) do business
(2) tranquillize
(3) transcend
(4) exceed
(5) transfer

Directions (29 30): Choose the word which is most opposite in meaning to the word printed in bold as used
in the passage.

29. Substantia

(1) meagre
(2) considerable
(3) large
(4) submissive
(5) sufficient

30. Minuscule

(1) small
(2) minimum
(3) minute
(6) large
(7) maximum

Directions (31-40): In the following passage there are blanks each of which has been numbered. These
numbers are printed below the passage and against each five words are suggested one of which fits the blank
appropriately. Find out the appropriate word in each case.

Trust is (31) relationship concept that needs clarification because researchers (32) disciplines have
defined it in so many different ways. A typology of trust types would make it easier to (33) and
communicate results and would be especially valuable if the types of trust related to one another. The
typology should be interdisciplinary (34) many disciplines research e-commerce. This paper (35) a
parsimonious interdisciplinary typology and relates trust constructs to e-commerce consumer actions
defining both conceptual-level and operational-level trust constructs. Conceptual-level constructs

Click Here to Buy Printed Study Material for IBPS, SBI Bank Exams
http://www.bankexamportal.com/study-kit Page 84
Join Online Coaching for IBPS, SBI Exams
http://www.bankexamportal.com/elearning

consist (36) disposition to trust (primarily from psychology) institution-based trust (from sociology)
and trusting (37) and trusting intentions (primarily from social psychology). Each construct is (38)
into measurable sub constructs, and the typology shows how trust constructs relate (39) already
existing Internet relationship constructs. The effects of Web vendor interventions on consumer
behaviors are posited to be(40) mediated by consumer trusting beliefs and trusting intentions in the e-
vendor.

31.

(1) vital (2) close


(3) important (4) low
(5) proximate

32.

(1) along
(2) around
(3) across
(4) above
(5) over and above

33.

(l) compose
(2) compare
(3) comparison
(4) comparing
(5) spare

(1) but
(2) that
(3) which
(4) because
(5) as if

35.

(1) justifies
(2) clarifies
(3) denies
(4) regards
(5) registers

36.

Click Here to Buy Printed Study Material for IBPS, SBI Bank Exams
http://www.bankexamportal.com/study-kit Page 85
Join Online Coaching for IBPS, SBI Exams
http://www.bankexamportal.com/elearning

(1) in
(2) of
(3) for
(4) with
(5) by

37.

(1) disbeliefs
(2) beliefs
(3) ways
(4) means
(5) measures

38.

(1) decomposed
(2) decomposing
(3) deposed
(4) divided
(5) distributed

39.

(1) with
(2) in
(3) by
(4) of
(5) to

40.

(1) partially
(2) legally
(3) generally
(4) illegally
(5) gradually

Click Here to Buy Printed Study Material for IBPS, SBI Bank Exams
http://www.bankexamportal.com/study-kit Page 86
Join Online Coaching for IBPS, SBI Exams
http://www.bankexamportal.com/elearning

IBPS Clerk Online CWE Previous Year Exam Paper - 2012

Subject: Reasoning Ability


Directions (1-5): Each of the questions given below is based on the given diagram. The diagram shows
three circles each representing Doctors, Experienced Hospital Employee and Post Graduates.

1. Which of the following represents such people who are Experienced- Hospital
Employees but are not doctors?

(1) Only A
(2) Only C
(3) A and C
(4) F and C
(5) G and A

2. Which of the following represents such people who are Doctors and Post Graduates but
not Experienced- Hospital Employees?

(1) Only D
2) B and F
(3) Only E
(4) Only B
(5) B and E

3. Which of the following represents such Doctors who are also Experienced- Hospital
Employees?

(1) G and F
(2) D and G
(3) Only A
(4) Only G
(5) Band F

4. Which of the following represents Experienced-Hospital Employees who are Doctors but
are not Post Graduates?

Click Here to Buy Printed Study Material for IBPS, SBI Bank Exams
http://www.bankexamportal.com/study-kit Page 87
Join Online Coaching for IBPS, SBI Exams
http://www.bankexamportal.com/elearning

(1) Only F
(2) F and G
(3) B, F and G
(4) A and G
(5) Only G

5. Which of the following represents all such people who are Doctors?

(1) Only D
(2) B, D, F and G
(3) Only F
(4) B, D and G
(5) S, F and G

Directions (6-10): Study the following arrangement carefully and answer the questions given below:
QSRUIVPJLGNFMTOKEAH DBC

6. How many such vowels are there in the above arrangement each of which is immediately
preceded by a Consonant and also immediately followed by a Vowel?

(1) None
(2) Three
(3) Four
(4) One
(5) Two

7. How many such pairs of alphabets are there in the series of alphabets given in Bold in the
above arrangement each of which has as many letters between them (in both forward and
backward directions) as they have between them in the English alphabetical series?

(1) None
(2) One
(3) Two
(4) Three
(5) More than three

8. On the basis of above series of alphabets DECK is written as BABE and BARS is
written as CHSR, then how will JUMP be written?

(1) LIFJ
(2) PRTV
(3) LRFV
(4) PITJ
(5) None of these

9. On the basis of above series of alphabets if PJ is related to GI, EA is related to DO, then
NF is related to-

Click Here to Buy Printed Study Material for IBPS, SBI Bank Exams
http://www.bankexamportal.com/study-kit Page 88
Join Online Coaching for IBPS, SBI Exams
http://www.bankexamportal.com/elearning

(1) JL
(2) OL
(3) TL
(4) TO
(5) None of these

10. Which of the following is the fourth to the left of the ninth from the left end of the
above arrangement?

(1) V
(2) I
(3) U
(4) M
(5) None of these

Directions (11-15): Study the following information carefully and answer the questions given below:

J, K. L, M, N, 0, P and Q are sitting around a circular table facing the centre but not necessarily in the
same order. 0 is immediate neighbour of both K and Q. Only one person sits between K and J. Land
Mare neighbours but are not immediate neighbours of

Q. Two persons sit between Mana P. P does not sit to the immediate right of K.

11. What is the position or P with respect to Q ?

(1) Fourth to the left


(2) Third to the left
(3) Third to the right
(4) Second to the left
(5) Second to the right

12. Who amongst the following is sitting to the right of J?

(1) N
(2) M
(3) L
(4) P
(5) K

13. Who amongst the following sits exactly between K and J ?

(1) P
(2) O
(3) Q
(4) L
(5) None of these

14. Who amongst the following sits fourth to the left of M ?

Click Here to Buy Printed Study Material for IBPS, SBI Bank Exams
http://www.bankexamportal.com/study-kit Page 89
Join Online Coaching for IBPS, SBI Exams
http://www.bankexamportal.com/elearning

(1) N
(2) J
(3) P
(4) O
(5) None of these

15. Four of the following five are alike in a certain way and thus form a group. Which is
the one that does not belong to that group?

(1) JLL
(2) NQ
(3) ML
(4) OK
(5) PJ

Directions (16-18): Study the following information to answer the given questions.
In a six storeyed building the ground floor is numbered one, the floor above it is numbered two and so
on such that the topmost floor is numbered six. One out of six people viz, A, B, C, D, E and F lives on
each floor.

No one lives between C and F. There are two floors between the floors on which A and D live. A lives
on floor above the floor on which D lives. E lives on odd numbered floor. B does not live on a floor
immediately above or below Es floor.

16. Who lives on the ground floor?

(1) E
(2) B
(3) D
(4) A
(5) Cannot be determined

17. Where does A live?

(1) Numbered 2
(2) Numbered 3
(3) Numbered 5
(4) Numbered 4
(5) None of these

18. Who lives immediately above Ds floor?

(1) A
(2) C
(3) F
(4) B
(5) Cannot be determined

Click Here to Buy Printed Study Material for IBPS, SBI Bank Exams
http://www.bankexamportal.com/study-kit Page 90
Join Online Coaching for IBPS, SBI Exams
http://www.bankexamportal.com/elearning

Directions (19-23): In each question below are two statements followed by two conclusions numbered I
and II. You have to take the two given statements to be true even if they seem to be at variance from
known facts and then decide which of the given conclusions logically commonly follows from the given
statements disregarding commonly known facts.

Give answer (1) if only conclusion I follows.


Give answer (2) if only conclusion II follows.
Give answer (3) if either conclusion I or II follows.
Give answer (4) if neither conclusion I nor II follows.
Give answer (5) if both conclusions I and II follow.

19. Statements:

All buses are cars.


Some buses are trucks.

Conclusions:

I. Some buses are definitely not trucks.


II. At least some trucks are cars.

20. Statements:

No leaf is a root.
All plants are roots.

Conclusions:

I. No leaf is a plant.
II. Some plants are leaves.

21. Statements:

Some chapters are units.


Some units are topics.

Conclusions:

I. At least some topics are chapters.


II. Some topics are definitely not units.

22. Statements :

All spoons are bowls.


All dishes are bowls.

Conclusions:

Click Here to Buy Printed Study Material for IBPS, SBI Bank Exams
http://www.bankexamportal.com/study-kit Page 91
Join Online Coaching for IBPS, SBI Exams
http://www.bankexamportal.com/elearning

I. Some spoons are dishes.


II. All bowls are spoons.

23. Statements:

Some gates are doors.


No gate is window.

Conclusions:

I. No door is window.
II. Some doors are definitely not windows.

Directions (24-28): In these questions relationships between different elements is shown in the
statements. These statements are followed by two conclusions.

Mark answer (1) if only Conclusion I follows


Mark answer (2) if only Conclusion II follows
Mark answer (3) if either Conclusion I or II follows
Mark answer (4) if neither Conclusion I nor II follows
Mark answer (5) if both Conclusions I and II follow

24. Statement: A F T = E R

Conclusions:

I. A < F
II. R F

25. Statement: R I < G H >T

Conclusions:

I. T < I
II. H > I

26. Statement: T = A K > E = S

Conclusions:

I. K T
II. S < K

27. Statement: P < O = I N > T

Conclusions:

Click Here to Buy Printed Study Material for IBPS, SBI Bank Exams
http://www.bankexamportal.com/study-kit Page 92
Join Online Coaching for IBPS, SBI Exams
http://www.bankexamportal.com/elearning

I. P < N
II. O > T

28. Statement: D > E L > A Y

Conclusions:

I. Y D
II. A E

Directions (29-30): Study the following information carefully and answer the questions given below:
Among five friends P, Q, R, S and T each having different height. T is the second tallest. P is taller than
only S, R and T are taller than Q. P is taller than S but shorter than Q.

29. Who among the following is the tallest in the group?

(1) R or T
(2) T
(3) R
(4) Q
(5) None of these

30. Who among the following is taller than T?

(1) R
(2) Q
(3) P
(4) S
(5) None of these

Directions (31-35): Each of the following questions below consists or a question and two statements
numbered I and II given below it. You have to decide whether the data provided in the statements are
sufficient to answer the question. Read both the statements and-

Give answer (1) If the data in Statement I alone are sufficient to answer the question while the data in
/ statement II alone are not sufficient to answer the question.

Give answer (2) If the data in Statement II alone are sufficient to answer the question while the data
in statement I alone are not sufficient to answer the question.
Give answer (3) If the data in either Statement I alone or statement II alone are sufficient to answer
the question.
Give answer (4) If the data in both the Statements I and II together are not sufficient to answer the
question.
Give answer (5) If the data in both the Statements I and II are together necessary to answer the
question.

31. What is the total number of students in Course A in College X ?

Click Here to Buy Printed Study Material for IBPS, SBI Bank Exams
http://www.bankexamportal.com/study-kit Page 93
Join Online Coaching for IBPS, SBI Exams
http://www.bankexamportal.com/elearning

I. The respective ratio of girls and boys is 2 : 3.


II. The number of students has grown by 5 per cent this year as compared to 4 per cent last year from the
number 1000 which it was year before last.

32. What was the grand total of Falcon Team of College X?

I. Mayank correctly remembers that Falcon Team scored a grand total of above 82 but below 91.
II. Animesh correctly remembers that Falcon Team scored a grand total of above 77 and below 84.

33. Which direction is Ajay facing at the moment?

I. After walking 5 metres early morning from point P, Ajay is facing the opposite direction of the Sun.
II. Ajay took two consecutive right turns after covering a distance of 6 metres to reach the point P.

34. How is M related to H?

I. H is the daughter of P and sister of K.


II. S is the son of M and husband of P.

35. How is healthy coded in the language?

I. In that code language eat and drink healthy is written as se ta pa me and drink hot beverages is
written as ta nu fa.
II. In that code language eat hot meal daily is written as fa me la du and cold and hot is written as pa fa
ga.

36. How many such pairs of letters are there in the word FINANCIAL, each of which has as
many letters between them in the word as in the English alphabetical series (in both forward
and backward directions)?

(1) None
(2) One
(3) Two
(4) Three
(5) More than three

37. Cause: The Government has recently increased the taxes on retail items by about 10
per cent. Which of the following can be a possible effect of the above cause?

(1) The retailers will reduce the prices of retail items by about 10 per cent
(2) The retailers will dose their shops till the taxes are rolled back by the Government
(3) The retailers will increase the prices of retail items by about 20 per cent
(4) The retailers will most probably increase the prices of retail items by about 10 percent or less than 10
percent
(5) None of these

Click Here to Buy Printed Study Material for IBPS, SBI Bank Exams
http://www.bankexamportal.com/study-kit Page 94
Join Online Coaching for IBPS, SBI Exams
http://www.bankexamportal.com/elearning

38. Statement: Many students died in a collision of their bus and a truck near the school
premise because the driver of truck lost his balance owing to high speed of the truck.

Courses of Action:

(A) The Government should immediately cancel the licenses of all the trucks operating in the city.
(B) The Government should prohibit the movement of all the vehicles near the school premise.
(C) The Government should set up a high level task force to suggest measures to prevent such
incidents in the future.

A course of action is a step or administrative decision to be taken for improvement, follow-up or further
action in regard to the problem, policy etc. On the basis of the information given in the statement, decide
which of the suggested courses of action logically follow(s) for pursuing.

(1) Only A
(2) Only B
(3) Only C
(4) Only A and C
(5) None

Directions (39-40): Study the following information carefully and answer the questions given below:

The Government has decided to continue providing subsidy to consumers for cooking gas for three
more years. This is not good news from the point of view of reining in the fiscal deficit. Mounting
subvention for subsidies means diversion of savings by the government from investment to
consumption, raising the cost of capital in the process. The Government must cut expenditure on
subsidies to create more fiscal space for investments in both physical and social infrastructure. It should
outline a plan for comprehensive reform in major subsidies including petroleum, food and fertilizers
and set a final deadline.

39. Which of the following is a Conclusion which can be drawn from the facts stated in the
above paragraph?

(1) Government cannot withdraw subsidies provided to various items.


(2) Government subsidy on cooking gas is purely a political decision
(3) Government can compensate the expenditure incurred on subsidy by raising the various taxes
(4) Subsidy provided by the government under various heads to the citizens increases the cost of capital
(5) None of these

40. Which of the following is an assumption which is implicit in the facts stated in the
above paragraph?

(1) It is not possible to create more infrastructural abilities if the present level of subsidy continues even for
short duration
(2) The Government should sought assistance from international financial organizations for developing
infrastructural projects
(3) The people of India may not be able to pay more for cooking gas
(4) India is no longer a developing country and many people in the country are rich enough to buy petroleum

Click Here to Buy Printed Study Material for IBPS, SBI Bank Exams
http://www.bankexamportal.com/study-kit Page 95
Join Online Coaching for IBPS, SBI Exams
http://www.bankexamportal.com/elearning

products market cost


(5) None of these

Subject: English Language


Directions (1-5): Which of the phrases (1), (2), (3) and (4) given below should replace the phrase given
in bold in the following sentences to make the sentence grammatically correct? If the sentence is correct
as it is and there is no correction required mark (5) i.e. No correction required as the answer.

1. During the recession many companies will be forced to lay oil workers.

(1) have the force to


(2) be forced into
(3) forcibly have
(4) forcefully
(5) No correction required

2. He wanted nothing else expecting to sleep after a stressful day at work.

(1) nothing better than


(2) anything else unless
(3) nothing but having
(4) nothing else than
(5) No correction required

3. Ramesh took charge of the project, within a few days of having appointed?

(1) having an appointment


(2) being appointed
(3) after being appointed
(4) appointing
(5) No correction required

4. It is difficult to work with him because he is one of those persons who think he is always
right.

(1) think they are always


(2) always thinks he is
(3) is always thinking they are
(4) always think his
(5) No correction required

5. Foreign businesses in developing countries have usually problems with lack of


infrastructure and rigid laws.

(1) usual problems as


(2) usually problems on

Click Here to Buy Printed Study Material for IBPS, SBI Bank Exams
http://www.bankexamportal.com/study-kit Page 96
Join Online Coaching for IBPS, SBI Exams
http://www.bankexamportal.com/elearning

(3) as usual problems like


(4) the usual problems of
(5) No correction required

Directions (46-50): Pick out the most effective word from the given words to fill in the blank in each
sentence to make the sentence meaningfully complete.

6. They work hard not because of the __ , but because of their inner urge.

(1) desire
(2) drive
(3) energy
(4) incentive
(5) motivation

7. His __ background has made him so docile.

(1) famous
(2) lucrative
(3) rich
(4) advanced
(5) humble

8. It is __ for everyone to abide by the laws of the land.

(1) expected
(2) obligatory
(3) meant
(4) optional
(5) recommended

9. he is a hard worker, his quality of work is not of a desirable level.

(1) Despite
(2) Because
(3) Although
(4) Somehow
(5) However

10. In spite of repeated instructions, he __ the some mistakes.

(1) commits
(2) detects
(3) corrects
(4) imitates
(5) exhibits

Click Here to Buy Printed Study Material for IBPS, SBI Bank Exams
http://www.bankexamportal.com/study-kit Page 97
Join Online Coaching for IBPS, SBI Exams
http://www.bankexamportal.com/elearning

Directions (11-20): Read the following passage carefully and answer the questions given below it.
Certain words have been printed in bold to help you locate them while answering some of the question.

Political ploys initially hailed as master-strokes often end up as flops. The Rs. 60,000 crore farm loan
waiver announced in the budget writes off 100% of overdues of small and marginal farmers holding
upto two hectares, and 25% of overdues of larger farmers. While India has enjoyed 8%-9% GDP
growth for the past few years, the boom has bypassed many rural areas and farmer distress and suicides
have made newspaper headlines. Various attempts to provide relief (employment guarantee scheme,
public distribution system) have made little impact, thanks. to huge leakages from the governments
lousy delivery systems. So, many economists think the loan waiver is a worthwhile alternative to provide
relief.

However the poorest rural folk are landless labourers, who get neither farm loans nor waivers. Half of
the small and marginal farmers get no loans from banks and depend entirely on money-lenders, and
will not benefit. Besides, rural India is full of the family holdings rather than individual holdings and
family holdings will typically be much larger than two hectares even for dirt-poor farmers, who will,
therefore, be denied the 100% waiver. It will thus fail in both economic and political objectives. IRDP
loans to the rural poor in the 1980s demonstrated that crooked bank officials demand bribes amounting
to one-third the intended benefits. Very few of the intended beneficiaries who merited relief received it.
After the last farm loan waiver will Similarly slow down fresh loans to deserving farmers. While
overdues to cooperatives may be higher, economist Surjit Shalla says less than 5% of farmer loans to
banks are overdue i.e. overdues exist for only 2.25 million out of 90 million farmers. If so, then the 95%
who have repaid loans will not benefit. They will be angry at being penalised for honesty.

The budget thus grossly overestimates the number of beneficiaries, It also underestimates the negative
effects of the waiver-encouraging willful default in the future and discouraging fresh bank lending for
some years. nstead of trying to reach the needy, through a plethora of leaky schemes we should transfer
cash directly to the needy using new technology like biometric smart cards, which are now being used
in many countries, and mobile phones bank accounts. Then benefits can go directly to phone accounts
operable only by those with biometric cards, ending the massive leakages of current schemes.

The political benefits of the loan waiver have also been exaggerated since if only a small fraction of farm
families benefit, and many of these have to pay bribes to get the actual benefit, will the waiver really be
a massive vote- winner? Members of joint families will feel aggrieved that, despite having less than one
hectare per head, their family holding is too large. Lo qualify for the 100% waiver. Alliance ministers,
of central or state governments, give away freebies in their last budgets, hoping to win electoral regards,
Yet, four-fifth of all incumbent governments are voted out. This shows that beneficiaries of favours are
not notably grateful, while those not so favoured may feel aggrieved, and vote for the opposition. That
seems to be why election budgets constantly fail to win elections in India and the loan waiver will not
change that pattern.

11. Why do economists feel that loan waivers will benefit farmers in distress?

(1) It will improve the standard of living of those farmers who can afford to repay their loans but are
exempted,
(2) Other government relief measures have proved ineffective.
(3) Suicide rates of farmers have declined after the announcement of the waiver.
(4) Farmers will be motivated to increase the size of their family holdings not individual holdings.
(5) The government will be forced to reexamine and Improve the public distribution system.

Click Here to Buy Printed Study Material for IBPS, SBI Bank Exams
http://www.bankexamportal.com/study-kit Page 98
Join Online Coaching for IBPS, SBI Exams
http://www.bankexamportal.com/elearning

12. What message will the loan waiver send to farmers who have repaid loans?

(1) The Government will readily provide them with loans in the future.
(2) As opposed to money lenders banks are a safer and more reliable source of credit.
(3) Honesty is the best policy.
(4) It is beneficial to take loans from co-operatives since their rates of interest are lower.
(5) They will be angry at being penalised for honesty

13. What is the authors suggestion to provide aid to farmers?

(1) Families should split their joint holding to take advantage of the loan waiver.
(2) The government should increase the reach of the employment guarantee scheme.
(3) Loans should be disbursed directly into bank accounts of the farmers using the latest technology.
(4) Government should ensure that loans waivers can be implemented over the number of years.
(5) Rural infrastructure can be improved using schemes which were successful abroad.

14. What was the outcome of IRDP loans to the rural poor?

(1) The percentage of bank loan sanctioned to family owned farms increased.
(2) The loans benefited dishonest money-lenders not landless labourers.
(3) Corrupt bank officials were the unintended beneficiaries of the loans.
(4) It resulted in the Government sanctioning thrice the amount for the current loan waiver.
(5) None of these.

15. What are the terms of the loan waiver?

(A) One-fourth of the overdue loans of landless labourers will be written off.
(B) The Rs. 60,000 crore loan waiver has been sanctioned for 2.25 million marginal farmers.
(C) Any farmer with between 26 per cent to 100 per cent of their loan repayments overdue will be
penalised.

(1) Only(A)
(2) Only (B)
(3) Both (B) and (C)
(4) All (A), (B) and (C)
(5) None of these

16. What is the authors view of the loan waiver?

(1) It will have an adverse psychological impact on those who cannot avail of the waiver.
(2) It is a justified measure in view of the high suicide rate among landless labourers.
(3) It makes sound economic and political sense in the existing scenario.
(4) It will ensure that the benefits of Indias high GDP are felt by the rural poor.
(5) None of these

17. Which of the following cannot be said about loan waiver?

Click Here to Buy Printed Study Material for IBPS, SBI Bank Exams
http://www.bankexamportal.com/study-kit Page 99
Join Online Coaching for IBPS, SBI Exams
http://www.bankexamportal.com/elearning

(A) Small and marginal farmers will benefit the most.


(B) The loan waiver penalizes deserving farmers.
(C) A large percentage i.e.. ninety- five per cent of distressed farmers will benefit.

(1) Only(C)
(2) Both (A) and (C)
(3) Only (A)
(4) Both (B) and (C)
(5) None of these

18. Which of the following will definitely be an impact of Joan waivers?

(A) Family holdings will be split into individual holdings not exceeding one hectare.
(B) The public distribution system will be revamped.
(C) Opposition will definitely win the election.

(1) None
(2) Only (A)
(3) Both (A) and (B)
(4) Only (C)
(5) All (A), (B) and ( C)

29. What impact will the loan waiver have on banks?

(1) Banks have to bear the entire brunt of the write off.
(2) Loss of trust in banks by big farmers.
(3) Corruption among bank staff will increase.
(4) Farmers will make it a habit to default on loans
(5) None of these

20. According to the author what is the governments motive in sanctioning the loan
waiver?

(1) To encourage farmers to opt for bank loans from money- lenders.
(2) To raise 90 million farmers out of indebtedness.
(3) To provide relief to those marginal farmers who have the means to but have not repaid their loans
(4) To ensure they will be re-elected
(5) None of these

Directions (21-23): Choose the word which is most nearly the SAME in meaning to the word printed
in bold as used in the passage.

21. Incumbent

(1) mandatory
(2) present
(3) incapable

Click Here to Buy Printed Study Material for IBPS, SBI Bank Exams
http://www.bankexamportal.com/study-kit Page 100
Join Online Coaching for IBPS, SBI Exams
http://www.bankexamportal.com/elearning

(4) lazy
(5) officious

22. ploys

(1) surveys
(2) entreaties
(3) ruses
(4) lazy
(5) assurances

23. aggrieved

(1) vindicated
(2) intimidated
(3) offensive
(4) wronged
(5) disputed

Directions (24-25): Choose the word which is most OPPOSITE in meaning to the word printed in bold
as used in the passage.

24. plethora

(1) dearth
(2) missing
(3) superfluous
(4) sufficient
(5) least

25. merited

(1) ranked
(2) unqualified for
(3) lacked
(4) inept at
(5) unworthy of

Directions (26-30): Read each sentence to find out whether there is any grammatical error in it.

The error, if any, will be in one part of the sentence. The number of that part is the answer. If there is no
error, the answer is (5). (Ignore errors of punctuation, if any)

26. It is more better(l)/ if one of the parents(2)/ stays at home(3)/ to look after the
children(4)/ No error(5)

27. With a fresh coat(l)/ of paint(2)/ the school can(3)/ look much nice(4)/ No error(5)

Click Here to Buy Printed Study Material for IBPS, SBI Bank Exams
http://www.bankexamportal.com/study-kit Page 101
Join Online Coaching for IBPS, SBI Exams
http://www.bankexamportal.com/elearning

28. I asked the salesman(1) / If I could exchange(2)/ the faulty camera(3)/ with another
one(4)/ No error(5)

29. I took me(l)/ almost a hour(2)/ to fill the(3)/ application form(4)/ No error(5).

30. She insists (1)/ you stay(2)/ until her husband (3)/ comes home(4)/ No error (5)

Directions (31-40): In the following passage, there are blanks, each of which has been numbered.

These numbers are printed below the passage and against each, five words are suggested one of which fits
the blank appropriately. Find out the appropriate word in each case.

Mankind has seen rapid (31) in the last 150 years because of the mass manufacturing techniques (32) in
western nations and later taken to new levels of efficiency by Japan. Mass production and production
for the masses became the bases of new business strategies. Large scale consumption by all with the
social benefit of (33) poverty became the dominant economic strategy. The advent of electricity and its
large-scale application to lighting, heating and operating machines added a fresh dimension to
manufacturing. By the 1950s came (34) in electronics and transistor devices to be followed by
innovations in microelectronics, computers and various forms of sensors all of which (35) altered the
manufacturing scene. It is now no longer necessary to make prototypes in a factory or a laboratory to
study a new product. Many new products can be (36) on computers and their behaviour simulated on
them. By choosing an optimum design through such simulations, computer programmes can directly
(37) the manufacturing processes. These processes are generally called Computer Aided Design (CAD)
and Computer Assisted Manufacturing (CAM). These capabilities are leading to newer forms of (38)
by customers. Each customer can be offered several special options. Customised product design or (39)
manufacturing are other popular techniques currently in (40) in many developed countries.

31.

(1) havoc
(2) transformation
(3) destruction
(4) violence
(5) deforestation

32.

(1) discarded
(2) resorted
(3) indulged
(4) perfected
(5) designated

33.

(1) removing
(2) nurturing
(3) appeasing

Click Here to Buy Printed Study Material for IBPS, SBI Bank Exams
http://www.bankexamportal.com/study-kit Page 102
Join Online Coaching for IBPS, SBI Exams
http://www.bankexamportal.com/elearning

(4) cajoling
(5) mastering

34.

(1) additions
(2) gadgets
(3) modifications
(4) variety
(5) invention

35.

(1) immediately
(2) precisely
(3) irreversibly
(4) indefinitely
(5) measurably

36.

(1) designed
(2) produced
(3) manufactured
(4) sold
(5) purchased

37.

(1) inspire
(2) cultivate
(3) visualise
(4) drive
(5) curtail

38.

(1) uses
(2) demands
(3) advertisements
(4) consumption
(5) goods

39.

(1) visible
(2) secure

Click Here to Buy Printed Study Material for IBPS, SBI Bank Exams
http://www.bankexamportal.com/study-kit Page 103
Join Online Coaching for IBPS, SBI Exams
http://www.bankexamportal.com/elearning

(3) fundamental
(4) overt
(5) flexible

40.

(1) view
(3) vogue
(5) order
(2) wings
(4) isolation

Click Here to Buy Printed Study Material for IBPS, SBI Bank Exams
http://www.bankexamportal.com/study-kit Page 104
Study Kit for Railway Recruitment Board (RRB) Exams

What you will get:


100% Syllabus Covered in printed format.

4 Booklets

950+ Pages

One Year Current Affairs (PDF Copy)

Guidance & Support from Our Experts (via Call and Email)

Our Objectives:
Firstly to cover 100% syllabus of the Examination.

Secondly to compile all the required study materials in a single place, So to save the precious
time of the aspirants.

For More Information Click Given below link:


http://www.rrbportal.com/study-kit
Join Online Coaching for IBPS, SBI Exams
http://www.bankexamportal.com/elearning

Subject: General Awareness


1. Which of the following is not correct regarding the JICA President Award?

(1) It is given to an individual


(2) It is given to an organization
(3) It is given for making contributions in the field of development
(4) The J in JICA denotes Jordan
(5) Delhi Metro is the recipient of the 2012 Award

2. Who among the following was never the Governor of Reserve Bank of India?

(1) Osborne Smith


(2) James Braid Taylor
(3) Montek Singh Ahluwalia
(4) C.D. Deshmukh
(5) D. Subbarao

3. Which one of the following is correct regarding the Note Issuing Authority of the
Reserve Bank of India?

(1) Apart from the government it has the authority / monopoly to issue currency notes other than one rupee
notes/ coins
(2) One rupee notes/coins and coins of smaller denominations are put into circulation by the Central
government
(3) At present the RBI issues notes in seven denominations
(4) The functions of note issue and currency management is discharged by the RBI through its head office in
Mumbai
(5) None of these

4. Which of the following Is not correct regarding the National Integration Day?

(1) It aims to bring unity, peace, and spreading of love and brotherhood among fellow Indians
(2) It is also celebrated as Quami Ekata Divas
(3) It is celebrated in remembrance of Indira Gandhi
(4) It is celebrated on October 31
(5) National Integration Day celebration ends with Childrens Day

5. In banking, a Pay in Slip-

(1) Means a slip through which a person can deposit money in his account
(2) Means a slip through which a person can issue cheque from his account
(3) Is also called a withdrawal slip
(4) Must essentially contain IFSC Code
(5) None of these

Click Here to Buy Printed Study Material for IBPS, SBI Bank Exams
http://www.bankexamportal.com/study-kit Page 105
Join Online Coaching for IBPS, SBI Exams
http://www.bankexamportal.com/elearning

6. The Reserve Bank of India reduced which of the following rates by 25 basis points in
October 2012?

(1) Repo rate


(2) Cash Reserve Ratio
(3) Statutory Liquidity Ratio and Repo Rate
(4) Repo Rate and Cash Reserve Ratio
(5) None of these

7. Who among the following has authored the book, titled Saga of Struggle and
Sacrifice?

(1) Manmohan Singh


(2) A.P.J. Abdul Kalam
(3) Hamid Ansari
(4) Pranab Mukherjee
(5) Amartya Sen

8. Prime Minister Manmohan Singh effected a reshuffle of his council of ministers in


October 2012. Which of the following pairs is not correct with regard to the new Union
Cabinet Ministers?

(1) Manmohan Singh: Planning


(2) Salman Khurshid: External Affairs
(3) Kapil Sibal: Communications
(4) Anand Sharma: Information Technology
(5) Ajay Maken: Housing

9. A unique function of the bank is to create credit. Which of the following factors do not
determine the credit creation power of banks?

(1) Amount of Cash Reserve in the Country


(2) Cash Reserve Ratio
(3) Special Drawing Rights
(4) Monetary Policy of the Central Bank
(5) Willingness of Customers to Borrow

10. For which of the following debt instruments, not having a fixed rate of interest over the
life of the instrument, can Floating Interest Rate be applied?

(1) Aloan
(2) A bond
(3) A mortgage
(4) A credit
(5) All of these

11. Which of the following is not correct regarding SMS Banking?

Click Here to Buy Printed Study Material for IBPS, SBI Bank Exams
http://www.bankexamportal.com/study-kit Page 106
Join Online Coaching for IBPS, SBI Exams
http://www.bankexamportal.com/elearning

(1) It is operated through Pull messages


(2) One-Time password (OTP) is a type of pull message
(3) Most SMS banking solutions are add-on products
(4) The lack of encryption on SMS messages is an area of concern
(5) None of these

12. In banking, Mobile Banking refers to the performance of which of the following
services through a mobile device?

(1) Balance checks


(2) Account transactions
(3) Payments
(4) Credit applications
(5) All of these

13. Which among the following subsidiaries of the State Bank of India, which were
nationalized in 1960, has now been merged with SBI?

(1) State Bank of Bikaner and Jaipur


(2) State Bank of Hyderabad
(3) State Bank of Patiala
(4) State Bank of Saurashtra
(5) State Bank of Travancore

14. The Goods and Services Tax (GST) is to be implemented in India. Which of the
following taxes will not be replaced by It?

(1) Excise duty


(2) Service tax on the Centres front
(3) Value Added Tax (VAT) at states end
(4) Cesses, surcharges and local levies
(5) Wealth Tax

15. Service Tax act/Rules in India has provided exemption to small scale service providers
from service tax up to aggregate value of taxable services provided up to how much in a
Financial Year?

(1) Rs. 1 lakh


(2) Rs. 2.5 lakhs
(3) Rs. 5 lakhs
(4) Rs. 10lakhs
(5) None of these

16. Which of the following does not aptly describe Demat Account in India?

(1) It refers to a dematerialized account for individual Indian citizens to trade In listed stocks or debentures
in paper form
(2) It is required for investors by the Securities and Exchange Board of India (SEBI)

Click Here to Buy Printed Study Material for IBPS, SBI Bank Exams
http://www.bankexamportal.com/study-kit Page 107
Join Online Coaching for IBPS, SBI Exams
http://www.bankexamportal.com/elearning

(3) In a Demat account, shares and securities are held electronically


(4) This account is opened by the Investor while registering with an investment broker (or sub-broker)
(5) Access to the Demat account requires an internet password and a transaction password

17. The Securities and Exchange Board of India (SEBI) has introduced a new type of
Demat Account called Basic Services De-mat Account (BSDA). Which of the following is
one of its features?

(1) Individuals are eligible to have multiple BSDA account across all depositories
(2) Investors will receive transaction statement on annual base
(3) Investors can avail at least two delivery instruction slip during account opening
(4) Investors can avail SMS alert facility for credit transactions
(5) If investors account have zero balance and no transactions during the year then will receive no annual
statement

18. Which of the following is not one of the major charges usually levied on a Demat
account?

(1) Account opening fee


(2) Annual maintenance fee
(3) Annual fee for spot conversions of one currency into another
(4) Custodian fee
(5) Transaction fee

19. Who among the following has authored the famous novel titled Joseph Anton?

(1) Jeet Thayil


(2) Salman Rushdie
(3) Shobha De
(4) V.S. Nalpaul
(5) Chetan Bhagat

20. Which of the following are the essential facilities/benefits provided by a Demat account
to an individual to trade the securities in the stock market?

(1) It reduces brokerage charges


(2) It makes pledging/hypothecation of shares easier
(3) It enables quick ownership of securities on settlement reo sulting in increased liquidity
(4) It avoids confusion in the ownership title of securities
(5) All of these

21. Which one of the following can be described as a secondary function of commercial
banks in India?

(1) Overdraft facilities


(2) Discounting Bills of Exchange
(3) Money at Call

Click Here to Buy Printed Study Material for IBPS, SBI Bank Exams
http://www.bankexamportal.com/study-kit Page 108
Join Online Coaching for IBPS, SBI Exams
http://www.bankexamportal.com/elearning

(4) Purchase and Sale of Securities


(5) Remittance of Funds

22. The balance sheet of a bank comprises of two sides; the assets side and the liabilities
side. Which one of the following items comes under the side of Assets?

(1) Subscribed capital


(2) Acceptance and endorsements
(3) Money at call and short notice
(4) Bills for collection
(5) Paid-up Capital

23. In the reshuffle of Union Council of Ministers, undertaken by Prime Manmohan Singh
in October 2012, who among the following is a Cabinet Minister (excluding Ministers of
State and Ministers of Stare with Independent Charge)?

(1) Pawan Kumar Bansal


(2) Shashi Tharoor
(3) Jyotiraditya Madhavrao Scindia
(4) Sachin Pilot
(5) Chiranjeevi

24. The first ever India Biodiversity Awards were presented in October 2012. In which of
the following categories were these awards not given?

(1) Community Stewardship


(2) Decentralized Governance
(3) Environmental sensitization
(4) Co-management
(5) Protected Area

25. Which of the following Central Armed Police Forces of India has recently celebrated
its golden jubilee?

(1) Sash astra Seema Bal (SSB)


(2) Border Security Force (BSF)
(3) Central Reserve Police Force (CRPF)
(4) Indo-Tibetan Border Police (ITBP)
(5) Central Industrial Security Force (CISF)

26. Which of the following is not correct regarding cheque in banking?

(1) It is a type of bill of exchange


(2) It is a non-negotiable instrument
(3) There are four main items on a cheque
(4) Parties to regular cheques generally include a drawer and a payee
(5) Cheques have been in decline for some years both for point of sale transactions and for third party
payments

Click Here to Buy Printed Study Material for IBPS, SBI Bank Exams
http://www.bankexamportal.com/study-kit Page 109
Join Online Coaching for IBPS, SBI Exams
http://www.bankexamportal.com/elearning

27. Which of the following does not aptly describe a Demand Draft in India?

(1) It is a type of cheque


(2) It is an Account payee instrument
(3) In case of Demand drafts, the drawer is a bank
(4) The principle of Cheque Clearing does not apply to it
(5) None of these

28. India has taken a pledge towards strengthening the institutional mechanism for
biodiversity conservation in the country with a sum of $ 10 million announced by Prime
Minister Manmohan Singh in this regard. The name of this pledge is

(1) Rio Pledge


(2) Doha Pledge
(3) Hyderabad Pledge
(4) Delhi Pledge
(5) Kyoto Pledge

29. Which of the following is not a technique/tool/Instrument of money/credit control by


the Reserve Bank of India?

(1) Open Market Operations


(2) Bank Rate
(3) Liquidity Adjustment Facility
(4) Refinance
(5) Monopoly to issue currency notes

30. Which of the following is a type of indirect security?

(1) Mutual Funds


(2) Ordinary Shares
(3) Debentures
(4) Preference shares
(5) Innovative Debt Instruments

31. Europe recently produced the greatest comeback in Ryder Cup to retain the trophy.
With which sports is the Ryder Cup associated?

(1) Golf
(2) Tennis
(3) Baseball
(4) Basketball
(5) Ice Hockey

32. The Cheque was first introduced in India by which of the following banks?

(1) Bank of Hindustan


(2) State Bank of India

Click Here to Buy Printed Study Material for IBPS, SBI Bank Exams
http://www.bankexamportal.com/study-kit Page 110
Join Online Coaching for IBPS, SBI Exams
http://www.bankexamportal.com/elearning

(3) Bank of India


(4) Punjab National Bank
(5) None of these

33. United Nations secretary general Ban Ki-Moon has announced that every year 10
November will be celebrated as

(1) Malala Day


(2) Earth Day
(3) Development Day
(4) Wilmut Day
(5) Jobs Day

34. Which of the following is a quantitative credit control policy of the Reserve Bank of
India?

(1) Margin requirements


(2) Moral suasion
(3) Bank rate
(4) RBI guidelines
(5) Direct action

35. What is the term for inflation when the general price level increases at the rate of about
5-6 per cent per annum?

(1) Creeping inflation


(2) Walking inflation
(3) Running inflation
(4) Hyper inflation
(5) Galloping inflation

36. Which of the following cannot be termed as non-traditional functions of a commercial


bank in India?

(1) ATM facility


(2) Issuing credit cards
(3) Venture capital financing
(4) Creating deposits
(5) None of these

37. Who among the following is a Deputy Governor of the Reserve Bank of India?

(1) Abhijit Sen


(2) Mihir Shah
(3) K Kasturirangan
(4) H.R. Khan
(5) Arun Mall-a

Click Here to Buy Printed Study Material for IBPS, SBI Bank Exams
http://www.bankexamportal.com/study-kit Page 111
Join Online Coaching for IBPS, SBI Exams
http://www.bankexamportal.com/elearning

38. Which Union Budget first announced the introduction of the Goods and Services Tax
(GST) from April 1, 2010?

(1) Budget 2002-03


(2) Budget 2005-07
(3) Budget 2007 -08
(4) Budget 2009- 10
(5) None of these

39. The National Capital Region (NCR) in India is a name for the conurbation or
metropolitan area which encompasses the entire Delhi as well as urban areas around it in
neighbouring states. Which of the following is not that neighbouring state?

(1) Punjab
(2) Rajasthan
(3) Uttarakhand
(4) Uttar Pradesh
(5) None of these

40. Which one of the following banks was nationalized in the second wave of
nationalization in 1980?

(1) Central Bank of India


(2) Bank of India
(3) Syndicate Bank
(4) Corporation Bank
(5) Bank of Maharashtra

Click Here to Buy Printed Study Material for IBPS, SBI Bank Exams
http://www.bankexamportal.com/study-kit Page 112
Join Online Coaching for IBPS, SBI Exams
http://www.bankexamportal.com/elearning

Subject: Numerical Ability


1. In an urn there are 4 red balls and 3 blue balls. If two balls are drawn at random, find
the probability that none is red.

2. On annual day of a school some chocolates were to be distributed equally among 420
children. But on that particular day, due to some reason 140 more children of another
school joined them: hence each child got 1 chocolate less. How many chocolates were
originally supposed to be distributed among the children?

(1) 1640
(2) 1680
(3) 1690
(4) 1600
(5) None of these

3. Raj Kumar got the result of his 8th class. Each subject consists of a maximum of 140
marks. If he scores 98 marks in science; 129 marks in Sanskrit; 131 marks in Maths, 110
marks in English and 120 marks in Hindi, what was his percentage of marks in all the five
subject?

(1) 84%
(2) 82%
(3) 77%
(4) 79%
(5) None of these

4. If Suresh sells an article at a price of Rs. 9300, he incurs a loss of Rs. 3100. At what price
should he sell the article so that he gets a profit of 25%?

(1) Rs. 7250


(2) Rs. 7350
(3) Rs. 7650
(4) Rs. 7750
(5) None of these

5. There is a circular ground whose area is 246400 sq. metre. If a person runs at the speed
of 14.08 m/sec, then how much time will he take to complete the circle?

Click Here to Buy Printed Study Material for IBPS, SBI Bank Exams
http://www.bankexamportal.com/study-kit Page 113
Join Online Coaching for IBPS, SBI Exams
http://www.bankexamportal.com/elearning

(1) 125 sec.


(2) 130 sec.
(3) 100 sec.
(4) 120 sec.
(5) None of these

Directions (6-10): What should come in place of the question mark (?) in the following number series?

6. 2 4 16 96 768 ? 92160

(1) 7680
(2) 7580
(3) 7608
(4) 7090
(5) 7860

7. 14 36 ? 300 897 2676 8022

(1) 101
(2) 102
(3) 103
(4) 104
(5) None of these

8. 5 8 13 20 ? 44 61

(1) 29
(2) 30
(3) 31
(4) 32
(5) 37

9. 11 16 31 56 91 136 ?

(1) 171
(2) 181
(3) 185
(4) 191
(5) 197

10. 3 4 12 45 196 ?

(1) 985
(2) 990
(3) 995
(4) 1000
(5) 1005

Click Here to Buy Printed Study Material for IBPS, SBI Bank Exams
http://www.bankexamportal.com/study-kit Page 114
Join Online Coaching for IBPS, SBI Exams
http://www.bankexamportal.com/elearning

11. The simple interest accrued in 2 years on a principal of Rs. 24000 is one-eighth of the
principal. What is the rate of simple interest p.c.p.a. ?

(1) 5
(2) 4.5
(3) 6.25
(4) 7.25
(5) None of these

12. If a person runs 14.35 km in five weeks, then what distance does he travel everyday?

(1) 400 m
(2) 410 m
(3) 405 m
(4) 415 m
(5) None of these

13. If a train 280 metre long runs at the speed of 7.4 m/ second how much time will it take
to cross a platform 460 metre long?

(1) 95 sec.
(2) 96 sec.
(3) 98 sec.
(4) 99 sec.
(5) 100 sec.

14. If a trader sells his stock of oranges at Rs. 18270, he gains 45 per cent. What is the cost
price of total stock of oranges?

(1) Rs. 12600


(2) Rs. 13600
(3) Rs. 12650.
(4) Rs. 13650
(5) None of these

15. If the numerator of a fraction is increased by 20% and its denominator by 25%. then
the fraction so obtained is . What is the original fraction ?

Directions (16-24): What will come in place of the question mark (?) in the following questions?

Click Here to Buy Printed Study Material for IBPS, SBI Bank Exams
http://www.bankexamportal.com/study-kit Page 115
Join Online Coaching for IBPS, SBI Exams
http://www.bankexamportal.com/elearning

16.

17. 97. 4 566 5 + 24.2 - 36 = (?)2

(1) 20
(2) 21
(3) 22
(4) 23
(5) 25

18. 5252 + 2525 = ? 25

(1) 310.8
(2) 311.8
(3) 311.08
(4) 312.8
(5) 312.08

19. 8 ? = 4888 4

(1) 150.75
(2) 125.75
(3) 125.05
(4) l52.75
(5) None of these

20. 39254 + 5217 - 2286 =? 50

(1) 813.7
(2) 843.7
(3) 834.7
(4) 943.77
(5) None of these

21. (62.5 14 5) 25 + 41 = (?)3

(1) 4
(2) 5

Click Here to Buy Printed Study Material for IBPS, SBI Bank Exams
http://www.bankexamportal.com/study-kit Page 116
Join Online Coaching for IBPS, SBI Exams
http://www.bankexamportal.com/elearning

(3) 9
(4) 8
(5) 6

22. (23 23 23 23 23 23)5 (23 23)2 (23)2 = (23)?

(1) 32
(2) 30
(3) 9
(4) 7
(5) 11

23. 27% of 510 = 266.3

(1) 182.6
(2) 122.6
(3) 123.6
(4) 128.6
(5) None of these

24.

25. 5/8 of 4/9 of 3/5 of 222 = ?

(1) 42
(2) 43
(3) 39
(4) 37
(5) None of these

26. A car covers a distance from town A to town B at the speed of 58 kmph and covers the
distance from town B to town A at the speed of 52 kmph What is the approximate average
speed of the car?

(1) 55 kmph
(2) 52 kmph
(3) 48 kmph
(4) 50kmph
(5) 60kmph

Click Here to Buy Printed Study Material for IBPS, SBI Bank Exams
http://www.bankexamportal.com/study-kit Page 117
Join Online Coaching for IBPS, SBI Exams
http://www.bankexamportal.com/elearning

27. Mr. Phanse invests an amount of Rs. 24,200 at the rate of 4 p.c.p.a. for 6 years to obtain
a simple interest. Later he invests the principal amount as well as the amount obtained as
simple interest for another 4 years at the same rate of interest. What amount of simple
interest will he obtain at the end of the last 4 years?

(1) Rs. 4,800


(2) Rs. 4,850.32
(3) Rs. 4,801.28
(4) Rs. 4,700
(5) None of these

28. In a sale perfumes are available at a discount of 25% on the selling price. If a perfume
costs Rs. 5.895 in the sale what is the selling price of the perfume?

(1) Rs. 6.020


(2) Rs. 7.860
(3) Rs. 7,680
(4) Cannot be determined
(5) None of these

29. What approximate value should come in place of the question mark (?) in the following
question?

(1) 294
(2) 276
(3) 265
(4) 300
(5) 288

30. The cost of 15 digital cameras and 21 handy cameras is Rs. 3.54.900. What is the cost of
5 digital cameras and 7 handy cameras?

(1) Rs. 1.25.500


(2) Rs. 1.18.300
(3) Rs. 2.15.100
(4) Cannot be determined
(5) None of these

31. A canteen requires 56 kgs. of rice for seven days. How many kgs. of rice will It require
for the months of April and May together?

(1) 496
(2) 480
(3) 498

Click Here to Buy Printed Study Material for IBPS, SBI Bank Exams
http://www.bankexamportal.com/study-kit Page 118
Join Online Coaching for IBPS, SBI Exams
http://www.bankexamportal.com/elearning

(4) 488
(5) None of these

32. How much part of a day is 45 minutes?

33. The total number of students in a school is 31700. If the ratio 01 boys to the girls in the
school is 743:842 respectively, what is the total number of girls in the school?

(1) 14860
(2) 16480
(3) 15340
(4) Cannot be determined
(5) None of these

34. The sum of five consecutive even numbers A, B, C, D and E is 130. What is the product
of A and E?

(1) 720
(2) 616
(3) 660
(4) 672
(5) None of these

35. If the square of a number is subtracted from 4052 and the difference is multiplied by
15, the answer so obtained is 41340. What is the number?

(1) 36
(2) 1024
(3) 32
(4) 1296
(5) None of these

36. 20% of the total cost of a plot with an area of 395 sq.ft. is Rs. 78,210. What is the rate
of per sq. ft. of the plot?

(1) Rs. 1,020/-


(2) Rs. 999/-
(3) Rs. 1,000/-

Click Here to Buy Printed Study Material for IBPS, SBI Bank Exams
http://www.bankexamportal.com/study-kit Page 119
Join Online Coaching for IBPS, SBI Exams
http://www.bankexamportal.com/elearning

(4) Rs. 995/-


(5) None of these

37. The owner of a furniture shop charges his customers 15% more than the cost price. If a
customer paid Rs. 9,039/- for a sofa set, then what is the cost price of the sofa set?

(1) Rs. 7,680/-


(2) Rs.7.860/-
(3) Rs. 7.880/-
(4) Rs. 7,660/-
(5) None of these

38. Each child from a certain school can make 5 items of handicraft in a day. If 1125
handicraft items are to be displayed in an exhibition then in how many days can 25
children make these items?

(1) 6 days
(2) 9 days
(3) 8 days
(4) 7 days
(5) None of these

39. The product of two successive positive integers is 462. Which is the smaller integer?

(1) 20
(2) 22
(3) 21
(4) 23
(5) None of these

40. While withdrawing an amount of Rs. 49.350/- a customer by mistake collects Rs.
48.150/-. The remaining amount is deposited back to his account by the bank, which shows
the balance of Rs. 25.376/-. What will be the customers balance after depositing the
remaining amount?

(1) Rs. 26,576/-


(2) Rs. 26,676/-
(3) Rs. 26,586/-
(4) Rs. 26,686/-
(5) None of these

Click Here to Buy Printed Study Material for IBPS, SBI Bank Exams
http://www.bankexamportal.com/study-kit Page 120
Join Online Coaching for IBPS, SBI Exams
http://www.bankexamportal.com/elearning

Subject: Computer Knowledge


1. The system unit

(1) coordinates input and output devices


(2) is the container that houses electronic components
(3) is a combination of hardware and software
(4) controls and manipulates data
(5) does the arithmetic operations

2. In Word you can force a page break

(1) By positioning your cursor at the appropriate place and pressing the F1 key
(2) By positioning your cursor at the appropriate place and pressing Ctrl + Enter
(3) By using the Insert/Section Break
(4) By changing the font size of your document
(5) None of these

3. The basic unit of a worksheet into which your enter data in Excel is called a _____

(1) tab
(2) cell
(3) box
(4) range
(5) None of these

4. To reload a Web page, press the _____ button.

(1) Redo
(2) Reload
(3) Restore
(4) Ctrl
(5) Refresh

5. Which of the following is a programming language for creating special programs like
applets?

(1) Java
(2) Cable
(3) Domain name
(4) Net
(5) COBOL

6. Which part of the computer is directly involved in executing the instructions of the
computer program?

Click Here to Buy Printed Study Material for IBPS, SBI Bank Exams
http://www.bankexamportal.com/study-kit Page 121
Join Online Coaching for IBPS, SBI Exams
http://www.bankexamportal.com/elearning

(1) The scanner


(2) The main storage
(3) The secondary storage
(4) The printer
(5) The Processor

7. In order to create column data in Word you need to

(1) Tab consecutively until your cursor reaches the desired place
(2) Set tabs or user the Table menu
(3) You need to use Excel
(4) Press the space bar until your cursor reaches the desired place
(5) None of these

8. When the computer is switched on. the booting process performs the

(1) integrity Test


(2) Power-On Self-Test
(3) Correct Functioning Test
(4) Reliability Test
(5) Shut-down

9. Press the __ key to move the insertion point to the first cell in a row in Excel.

(1) Page Up
(2) Page Down
(3) Home
(4) Tab
(5) None of these

10. How many margins are on a page?

(1) Two (header and footer)


(2) Four (top, bottom. right, left)
(3) Two (landscape and portrait)
(4) Two (top and bottom)
(5) None of these

11. Language used in a computer that is similar to the languages of humans and is easy to
understand is referred to as

(1) Source Code


(2) Machine Language
(3) High Level Language
(4) Object Code
(5) Assembly Language

Click Here to Buy Printed Study Material for IBPS, SBI Bank Exams
http://www.bankexamportal.com/study-kit Page 122
Join Online Coaching for IBPS, SBI Exams
http://www.bankexamportal.com/elearning

12. Which of the following should be used to move a paragraph from one place to another
in a Word document?

(1) Copy and Paste


(2) Cut and Paste
(3) Delete and Retype
(4) Find and Replace
(5) None of these

13. Excel documents are stored as files called ________

(1) workforce
(2) worksheets
(3) worktables
(4) workgroups
(5) workbooks

14. A computer programmer

(1) does all the thinking for a computer


(2) can enter input data quickly
(3) can operate all types of computer equipments
(4) can draw only flowchart
(5) is not a useful person

15. A cookie ______

(1) stores information about the users web activity


(2) stores software developed by the user
(3) stores the password of the user
(4) stores the commands used by the user
(5) None of these

16. Which of the following is not true about an assembler?

(1) Trartslates Instructions of assembly language into machine language


(2) It does not translate a C program
(3) It is involved in programs execution
(4) Is a translating program
(5) It does not translate a BASIC program

17. A web site is a collection of ______

(1) graphics
(2) programs
(3) algorithms
(4) web pages
(5) charts

Click Here to Buy Printed Study Material for IBPS, SBI Bank Exams
http://www.bankexamportal.com/study-kit Page 123
Join Online Coaching for IBPS, SBI Exams
http://www.bankexamportal.com/elearning

18. Which contents are lost when the computer turns off?

(1) storage
(2) input
(3) output
(4) memory
(5) None of these

19. Which of the following is hardware and not software?

(1) Excel
(2) Printer driver
(3) Operating System
(4) Power Point
(5) CPU

20. To be able to boot, the computer must have a(n)

(1) Compiler
(2) Loader
(3) Operating System
(4) Assembler
(5) None of these

21. In Excel this is a prerecorded formula that provides a shortcut for complex
calculations -

(1) Value
(2) Data Series
(3) Function
(4) Field
(5) None of these

22. In word, replace option is available on-

(1) File Menu


(2) View Menu
(3) Edit Menu
(4) Format Menu
(5) None of these

23. A word gets selected by clicking it to select a word in Word

(1) once
(2) twice
(3) three times
(4) four times
(5) None of these

Click Here to Buy Printed Study Material for IBPS, SBI Bank Exams
http://www.bankexamportal.com/study-kit Page 124
Join Online Coaching for IBPS, SBI Exams
http://www.bankexamportal.com/elearning

24. The process of trading goods over the Internet is known as

(1) e-selling-n-buying
(2) e- trading
(3) e-finance
(4) e-salesmanship
(5) e-commerce

25. In word, when you indent a paragraph, you-

(1) push the text in with respect to the margin


(2) change the margins on the page
(3) move the text up by one line
(4) move the text down by one line
(5) None of these

26. The operation of combining two cells into a single cell in Excel is referred to as

(1) Join Cells


(2) Merge Cells
(3) Merge Table
(4) Join Table
(5) None of these

27. What do you see when you click the right mouse button?

(1) The same effect as the left click


(2) A special menu
(3) No effect
(4) A mouse cannot be right clicked
(5) Computer goes to sleep mode

28. In order to delete a sentence from a document you would use __

(1) highlight and copy


(2) cut and paste
(3) copy and paste
(4) highlight and delete
(5) select and paste

29. When machine instructions are being executed by a computer the instruction phase
followed by the execution phase is referred to as __

(1) program cycle


(2) machine instruction
(3) execution cycle
(4) task cycle
(5) machine cycle

Click Here to Buy Printed Study Material for IBPS, SBI Bank Exams
http://www.bankexamportal.com/study-kit Page 125
Join Online Coaching for IBPS, SBI Exams
http://www.bankexamportal.com/elearning

30. Which of the following refers to the process of a computer receiving information from
a server on the Internet?

(1) gathering
(2) uploading
(3) inputting
(4) outputting
(5) downloading

31. Which of the following is not a storage medium?

(1) hard disk


(2) flash drive
(3) DVD
(4) CD
(5) monitor

32. _______ is the part of the computer that does the arithmetical calculations.

(1) OS
(2) ALU
(3) CPU
(4) Memory
(5) Printer

33. User can use ______ commands to search for and correct words in a document.

(1) Print and Print Preview


(2) Header and Footer
(3) Find and Replace
(4) Spelling and Grammar
(5) Copy and Paste

34. Which of the following will be used if a sender of e-mail wants to bold, italics etc. the
text message?

(1) Reach Signature


(2) Reach Text
(3) Reach Format
(4) Plain Format
(5) Plain Text

35. Which of the following justification align the text on both the sides-left and right -of
margin?

(1) Right
(2) Justify
(3) both Sides

Click Here to Buy Printed Study Material for IBPS, SBI Bank Exams
http://www.bankexamportal.com/study-kit Page 126
Join Online Coaching for IBPS, SBI Exams
http://www.bankexamportal.com/elearning

(4) Balanced
(5) None of these

36. Which of the following characteristics is used to compute dynamically the results from
Excel data?

(1) Goto
(2) Table
(3) Chart
(4) Diagram
(5) Formula and Function

37. What is included in an e-mail address?

(1) Domain Name followed by Users Name


(2) Users Name followed by Domain Name
(3) Users Name followed by Postal address
(4) Users Name followed by street address
(5) None of these

38. The other name of motherboard is

(1) Mouse
(2) Computer Board
(3) System Device
(4) Central Board
(5) System Board

39. In Power Point the Header & Footer button can be found on the Insert tab in what
group?

(1) Illustrations group


(2) Object group
(3) Text group
(4) Tables group
(5) None of these

40. What does CO stands in COBOL

(1) Common Object


(2) Common
(3) Common Operating
(4) Computer Oriented
(5) None of these

Click Here to Buy Printed Study Material for IBPS, SBI Bank Exams
http://www.bankexamportal.com/study-kit Page 127
Online Coaching for RRB Recruitment Exams
What you will get:
1. All the relevant and required materials of subjects mention in the RRB syllabus like:

100% RRB Exam Syllabus Covered with MCQs.

Special Current Affairs.


2. Home assignment
3. Important current affairs materials for RRB Examination will be provided
4. Online Tests will be conducted after the end of each subject.
5. At the end of your course, five comprehensive test will be conducted to evaluate your performance.

Our Objectives:
Firstly to cover 100% syllabus of the Examination.

Secondly to compile all the required study materials in a single place, So to save the precious time of
the aspirants.

Our Strategy:
Content of every section of the syllabus is developed after a exhaustive research of last year
Question Papers.

Every section is covered with practice set.

For More Information Click Given below link:


http://rrbportal.com/online-coaching

You might also like